Top Banner
Combined remembered Qs 2020 Decembery RQS – NBDE II December: trismus in which space The hallmark of a masticatory space infection is trismus or infection in anterior compartment of lateral pharyngeal space results in trismus. If these infections are unchecked, can spread to various facial spaces of the head and neck and lead to serious complications such as cervical cellulitis or mediastinitis propanolol interacts with- LA or epi to cause bradycardia If a patient on a nonselective beta-blocker receives a systemic dose of epinephrine, however, the beta-blocker prevents the vasodilation, leaving unopposed alpha vasoconstriction. The resulting hypertensive reaction can be large, with systolic pressure well over 200 mm Hg. (I actually surprisingly didn't get a lot of pharmacology on both days which was kind of disappointing because I prepared the most for it) recent studies cannot prove perio due to- smoking or vit deficit trephination is? (definition) surgical creation of a fistula by puncturing the soft tissue and bone overlying the root apex to provide drainage of infectious materials. Also called apicostomy . Surgical creation of a drainage tract with a bur or sharp instrument in the soft tissue or bone overlying a tooth root apex. osteotomy/tectomy (definition) alveoloplasty performed immediate or after extractions xylitol best form? gum/spray/patch Xylitol is widely used as a sugar substitute and in "sugar-free" chewing gums , mints, and other candies. calculations sensitvity specify 2 qs
230

Decembery RQS – NBDE II

Jun 10, 2022

Download

Documents

dariahiddleston
Welcome message from author
This document is posted to help you gain knowledge. Please leave a comment to let me know what you think about it! Share it to your friends and learn new things together.
Transcript
Page 1: Decembery RQS – NBDE II

Combined remembered Qs 2020

Decembery RQS – NBDE II 

December: • trismus in which space

The hallmark of a masticatory space infection is trismus or infection in anterior compartment of lateral pharyngeal space results in trismus. If these infections are unchecked, can spread to various facial spaces of the head and neck and lead to serious complications such as cervical cellulitis or mediastinitis

• propanolol interacts with- LA or epi to cause bradycardia If a patient on a nonselective beta-blocker receives a systemic dose of epinephrine, however, the beta-blocker prevents the vasodilation, leaving unopposed alpha vasoconstriction. The resulting hypertensive reaction can be large, with systolic pressure well over 200 mm Hg.

• (I actually surprisingly didn't get a lot of pharmacology on both days which was kind of disappointing because I prepared the most for it)

• recent studies cannot prove perio due to- smoking or vit deficit • trephination is? (definition)

surgical creation of a fistula by puncturing the soft tissue and bone overlying the root apex to provide drainage of infectious materials. Also called apicostomy. Surgical creation of a drainage tract with a bur or sharp instrument in the soft tissue or bone overlying a tooth root apex.

• osteotomy/tectomy (definition)

• alveoloplasty performed immediate or after extractions

● xylitol best form? gum/spray/patch

Xylitol is widely used as a sugar substitute and in "sugar-free" chewing gums, mints, and other candies.

• calculations sensitvity specify 2 qs

Page 2: Decembery RQS – NBDE II

Combined remembered Qs 2020 • Specificity calculator to evaluate the chances of a person being affected with diseases, calculated

based on the present health conditions. Negative cases are classified as true negatives (healthy people correctly identified as healthy) whereas false negative (sick people incorrectly identified as healthy).

Specificity = True Negative / (False Positive + True Negative) x 100 Sensitivity (true positive rate or probability of detection) is a statistical method that correctly identifies a person with a disease.

Sensitivity = True Positive / (True Positive + False Negative) x 100

• ohl which papilla interdental papilla

• cementoblastoma pic

• calcified carotid? xray pic

Page 3: Decembery RQS – NBDE II

Combined remembered Qs 2020

• boggy gingiva diffuse ewings or leukemia young pt pic. pt had bright red spongy gums

leukemia • best anti seizure in kids? diazepam post trauma seizure • which prep first in class 3? which filled first?prep large cavity first ,fill the small cavity

first. • pic with 4 dots over gums of centrals I put sinus • aggressive perio bacteria action

LAP is localised to first molar or incisor interproximal attachment loss, whereas GAP is the interproximal attachment loss affecting at least three permanent teeth other than incisors and first molar.

65-75% of bacteria are Gram-negative bacilli

rapid loss of periodontal attachment associated with Actinobacillus actinomycetemcomitans, Capnocytophaga spp and Porphyromonas gingivalis

The inflammatory exudate in the gingival tissues and gingival crevicular fluid is mostly polymorph neutrophils but also includes B cells and plasma cells.

The plasma cells produce specific antibodies in response to the periodontal pathogens, IgG

increased amounts IL-1α and IL-1β which cause osteoclastic bone resorption

The main distinction between the localized and generalized form of AgP lies in the number of teeth affected. GAP brings about attachment loss involving more than 30% of sites on teeth;[1] effectively being at least three permanent teeth other than the first molars or incisors.[16]

• best way to differentiate lesions cyst granuloma- biopsy/histollogical evaluation • incisional bipsy excisional biopsy measurements. questions were like, best to use

which for diagnosis Incisional biopsy is often used in lesions larger than 1 cm in size, as opposed to excisional biopsy, which is often used in lesions less than 1 cm.

• first pass metabolism absorbed in intestine or enterohepatic circulation First pass effect: After a drug is swallowed, it is absorbed by the digestive system and enters the portal circulation. The absorbed drug is carried through the portal vein into the liver. The liver is responsible for metabolizing many drugs.

Page 4: Decembery RQS – NBDE II

Combined remembered Qs 2020 • day 2 • asa classifications. I had probably 3 questions on this current

ASA I Normal healthy patient non smoking, no or minimal alcohol use

ASA II mild systemic disease smoker, social alcohol drinker, pregnancy, obesity, (30<BMI<40), well controlled DM/HTN, mild lung disease

ASA III severe systemic disease one or more moderate to severe diseases. poorly controlled DM or HTN, COPD, morbid obesity (BMI>40), active hepatitis, alcohol dependence or abuse, implanted pacemaker, history of MI (> 3 mos), TIA, CVA

ASA IV severe systemic disease that is constant threat to life

recent MI (< 3 mos), CVA, TIA, or CAD/stent, ongoing cardiac ischemia or severe valve dysfunction, severe reduction of ejection fraction, sepsis, DIC, ARD or ESRD not undergoing regularly scheduled dialysis

• qs about teeth not even in arch so always check ALLLL the pics and don't get

confused. • worst perio prognosis? furcation/mobility/7mm pocket • levothyroxine moa?

Levothyroxine is a synthetic form of thyroxine (T4), an endogenous hormone secreted by the thyroid gland, which is converted to its active metabolite, L-triiodothyronine (T3). T4 and T3 bind to thyroid receptor proteins in the cell nucleus and caubse metabolic effects through the control of DNA transcription and protein synthesis.

Page 5: Decembery RQS – NBDE II

Combined remembered Qs 2020 December 6&7 RQ’s: 1. Best cement for temporary crowns- -Polycarboxylate temporary luting cements present low postoperative sensitivity, adequate retention, and easy cleanup. Examples of this category are: Cling2 (CLINICIAN'S CHOICE) and Hy-Bond (Shofu Dental). -ZOE temporary luting cements are commonly used because of their sedative effect on sensitive teeth. It is well documented that eugenol is able to penetrate and diffuse throughout the dentin and can affect the bond strengths provided by resin materials used for definitive restorations. It presents an excellent antibacterial effect. -ZONE temporary luting cements are designed to replace eugenol with various types of carboxylic acids that do not interfere with definitive cementation. They have the characteristics of being compatible with resin provisional materials, are compatible with permanent resin cements, and show greater retention compared to ZOE cements but have no sedative effect on the pulp. -Resin temporary luting cements present high strength, excellent retention, better aesthetics, and easy cleanup. However, these temporary cements have a higher incidence of microleakage, discoloration, and odor associated with their use. 2. Pt came with diastema between two centrals and one of the central is rotated initial treatment for this patient for correcting his diastema (derotating his centrals)

3. 3 questions on efficacy and potency by asking which one of the above drug is showing more efficacy (I remembered efficacy as taller,potency as faster onset)

Page 6: Decembery RQS – NBDE II

Combined remembered Qs 2020

4. Pt with neuromas on his skin,half tongue swollen which disease- Neurofibromatosis

5. Pt with micrognathia,glossoptosis,cleft palates-Pierre Robinson syndrome

Page 7: Decembery RQS – NBDE II

Combined remembered Qs 2020

6. Embrassure rest,I bar used on which side- mesial

 

7. GIC does show fluoride release and one more advantage of GIC is - ionic bond with dentin and enamel

Page 8: Decembery RQS – NBDE II

Combined remembered Qs 2020

8. East-west cryers elevator used for which tooth- mandibular molar roots

Cryers: #27 Left, #28 Right 9. Thick viscous material to fluid this is know as- Thixotropic ( periodontal ligament has it )

10. Width of attached Gingiva- options are tricky like mucogingival groove to vestibular Phoenix,free mucosa to alveolar mucosa

The width of the attached gingiva is defined as the distance between the mucogingival junction and the projection on the external surface of the bottom of the gingival sulcus or periodontal pocket

11. To decrease penumbra- decrease object film distance

Page 9: Decembery RQS – NBDE II

Combined remembered Qs 2020

The size of this penumbra can be reduced by using a smaller object film distance (OFD), a greater focal film distance (FFD) and a smaller focal spot size. The larger the focal spot, the larger the size of the penumbra and the lower the image resolution. With a smaller focal spot size, a smaller OFD and a larger FFD the image is magnified to a lesser extent and the resultant image is of greater accuracy.

12. Two implant distance-3mm 13. Implant to tooth distance-1.5mm 14. Bioharzard waste regulation by- EPA 15. Abused child: local authorities o social services

Child Protective Services: social servce stafif obtains facts from the person making the referral 16. Old person abuse-no DHHS ( doctor in human health services)

Adult Protective Services (APS) agency 17. Antibiotic prophylaxis needed for- Artificial Aortic valve pt

18. Antibiotic for penicillin allergy pt- Axzzithromycin 500mg

Page 10: Decembery RQS – NBDE II

Combined remembered Qs 2020

20. Post operative sensitivity after perio surgery decreased by doing-plaque control 21. Percentage of fluoridation water in USA- 74% 22. Mucous retention cyst of lower lip minor salivary gland due to- trauma 23. Hyperplastic tuberosity removed by which method- wedging technique ( distal wedge) 24. Tricyclic antidepressants mainly act on which receptor for depression- serotonin receptor 25. Crown fits on die perfectly but not on pt tooth cause- while trimming the die technician trimmed the finish 26. Taurodontism-morphodifferentiation

Failure in morphodifferentiation results in size and shape abnormalities, such as peg lateral incisors and macrodontia 27. After applying benzocaine on mucosa pt became bluish and some other feathers I forgot – methemoglobinemia 28. single one of aspirin action on platelet aggregation how long [time] 7-10 days 30. Cleft palate pt of 15years have underdeveloped maxillary even after surgeries what might be the cause:

Page 11: Decembery RQS – NBDE II

Combined remembered Qs 2020 Cleft palate repair. These procedures can affect the vertical, anteroposterior, and transverse development of the maxilla and alveolar processes. Periosteal stripping at the time of surgery and the resulting fibrosis are the most likely reasons for this response. 31. embrasure rest and I bar clasp work on which side: MESIAL

repeat question #6 above 32. pt with erupted 2 laterals palatally what might be the cause: ??? 33. -this is pale and told you that she is feeling unwell and faint what’ve is the cause: vasovagal syncope 34. Retrognatic mandible which ostectomy- sagittal (Bilateral Sagittal Split Osteotomy ) In patients for whom only mandibular deficiencies need to be corrected surgically, bilateral sagittal split osteotomy (BSSO) is the most common procedure. Intra-oral distraction osteogenesis during orthodontic treatment as a solution for a Class II malocclusion has been proposed as an alternative to BSSO. 35. Neutrophils <1,000-no surgery should be done on this patient 36. Pt with good maintenance visit now showing pocket depth>5mm- need skillful periodontist to remove the calculus with curettes one more option was ultrasonic tip cant be that effective in removing the calculus below 5 37. Guided imaginary: Guided imagery is an extremely, effective, non-painful, inexpensive and easy mind-body technique people can practice in advance of dental appointments for best results and can also listen to in the waiting room and during dental procedures. 38. Crown fits on die perfectly but not on pt tooth cause- while trimming the die technician trimmed the finish line repeat question #25 above 39. C factor- bound to unbound surfaces

Page 12: Decembery RQS – NBDE II

Combined remembered Qs 2020 40. PMMA as a temporary crown- exothermic reaction

Tensile strength decreases with increased water absorption.[27] Its coefficient of thermal expansion is relatively high at (5–10)×10−5 °C−1.

41. Marijuana effects except- increases appetite

The short-term effects of marijuana or cannabinoid use include:

● increased heart rate

● low blood pressure, orthostatic hypotension

● muscle relaxation

● slowed digestion

● dizziness

● distorted perception (sights, sounds, time, touch)

● difficulty in thinking, memory, and problem solving

● loss of coordination and motor skills

● agitation, anxiety, confusion, panic, paranoia

● increased appetite

● dry mouth, dry eyes

42. Main component of biological width-options junctional epthelium, 43. Which is not act on cell wall- azithromycin

Azithromycin is a bacteriostatic drug acts by inhibiting protein synthesis. It binds reversibly to 50S ribosomal subunits of sensitive microorganism. Azithromycin interferes with transpeptidation and translocation thus there is inhibition of protein synthesis and hence inhibition of cell growth. 44. Metastasis to mouth from all except- brain 45. Access opening of mandibular molar round bur directed towards-no option of pulp chamber dear options Towards furcation,towards mesiobuccal,towards mesiolingual,towards distal orifice

Page 13: Decembery RQS – NBDE II

Combined remembered Qs 2020

46. Sailolith most common which duct- Wharton’s duct

Sialoliths are most common within the body of the submandibular gland or Wharton's duct (80% to 90%). 47. Vestibuloplasty which flap- supraperiosteal flap 48. Single dose of aspirin effect on platelet aggression duration- I thought it mainly act on trombaxane A2 so it will be 10hours repeated question #28

the effects of aspirin last for the duration of the life of the platelet [10 days] 49. Before surgery warfirin should be stopped till- 3 days (1-3 days)

Warfarin therapy should be stopped five days before major surgery and restarted 12 to 24 hours postoperatively.

50. Change in taste sensation which drug- losartan(metallic taste) Losartan is angiotensin II receptor antagonists.

51. Strength of zirconia is due to- Phase transformation The high fracture toughness exhibited by many of zirconia ceramics is attributed to the constraint of the tetragonal-to-monoclinic phase transformation and its release during crack propagation. 52. Position of teeth while talking-teeth doesn’t touch each other 53. Pt who takes 30packets of cigarettes per year which stage he is in- pre-contemplation 54. Chroma- Saturation, Hue- Wavelength 55. Natural characteristics of enamel at its Incisal edge- options that I was in between was opalescence Translucent enamel displays the characteristic of opalescence. Opalescence causes tooth enamel to reflect blue light back to the observer. Enamel has a primary mineral makeup of hydroxyapatite, which is crystalline calcium phosphate. Hydroxyapatite crystals align in organized, tightly packed masses to form prismatic enamel rods.

Page 14: Decembery RQS – NBDE II

Combined remembered Qs 2020 56. Xylitol most effective- chewing gums repeated question

57. Pictures of Lymphoepithelial cyst

58. “ recurrent herpes

59. “ sinus retention cyst

Page 15: Decembery RQS – NBDE II

Combined remembered Qs 2020

60. “AOT X-ray On radiographs, the adenomatoid odontogenic tumor presents as a radiolucency (dark area) around an unerupted tooth extending past the cementoenamel junction.

61. Ear ring X-ray

62. Mandibular tori X-ray

63. Mediastinum space infection spreads from- Retropharyngeal space 64. Guided bone graft best in - 3 wall infrabony defect 65. Kids heart rate- 110 (70 to 110 beats per minute) 66. Dr said the pt to get change his amalgam filling with composite which ethics- Veracity 67. Antibiotic used for- ANUG (other options herpengina,lichen planus, aphtus ulcer

Page 16: Decembery RQS – NBDE II

Combined remembered Qs 2020 Treatment of ANUG is by debridement (although pain may prevent this) and antibiotics (usually metronidazole) in the acute phase, and improving oral hygiene to prevent recurrence. herpangina is a viral infection 68. Gardner syndrome : multiple osteomas, cotton wool, supernumerary, café au late 69. Micrognathia,cleft palate and glossoptosis- Pierre Robinson Syndrome repeated 70. Traumatic Neuroma

In the oral cavity, the most commonly affected sites are the lip, tongue, and mental nerve area. Pressure on the suspected area usually provokes pain. 71. Adrenocorticosteroids used for antiinflamatory and analgesic 72. IAN didn’t work which accessory nerve- Mylohyoid 73. 16 year old Pt says anaesthesia will go away very quickly which LA will you give – Benzocaine 74. 1cartridge no anaesthesia what to do before second dose- wait for 5 more min 75. Digoxin work by +cardiac inotropic affect increases the force of contraction on cardiac muscle.

Page 17: Decembery RQS – NBDE II

Combined remembered Qs 2020

76. Nitroglycerin work by- vascular smooth muscle relaxation 77. Morphin work like- enkephalins 78. Pear shape bur- 245, 330

79. Nitrous oxide toxicity symptoms- nausea,tingling of fingers

Signs and symptoms of nitrous oxide-induced neurotoxicity include: numbness and tingling of the distal extremities and impairment of vibration sensation, light touch and pinprick, proprioception, and gait.

80. Microabrassion question 81. Y line of Ennis X-ray- (nasal fossa&maxillary sinus)

nasal cavity (straight radiopaque line) and border of the maxillary sinus (curved radiopaque line) 82. Restoration covering the cusp- Resistance form 83. Picture of compound odontoma

● complex odontoma: irregular calcified lesions with no distinct tooth components ● compound odontoma: identifiable tooth components, collection of small teeth, average age of 14–18

years. They are slightly more common in females and more common in the maxilla, especially the anterior maxilla

● Odontoma may occasionally grow large, resulting in bone expansion

Page 18: Decembery RQS – NBDE II

Combined remembered Qs 2020

● Generally, odontomas have been associated with trauma during primary dentition as well as with inflammatory and infectious processes, hereditary anomalies (Gardner syndrome, Hermann's syndrome), odontoblastic hyperactivity and alterations in the genetic components responsible for controlling dental development

84. Stafne bone cyst clinical features Stafne’s bone cavity is an uncommon bony defect occurred especially only at the lingual cortex of mandible. Clinically, patients with SBD are usually asymptomatic. Ovoid or round radiolucency can be observed near the angle of mandible, usually located between the inferior alveolar nerve and the lower margin of mandible. 85. For small class 1 cavity which cement do you prefer- composite 86. Modified wickman flap- 4questions MODIFIED WIDMAN FLAP

● The aim of the modified Widman flap surgery is healing and reattachment of periodontal pockets with minimum loss of periodontal tissues during and after surgery reduction of probing pocket depth by shrinkage individually occurs.

● No surgical pocket elimination and apical displacement of the flap. ● The MWF is indicated for the treatment of all types of periodontitis and provides excellent result with

probing depths up to ca. 6 mm. ● Advantages --> 1. Root cleaning with direct vision 2. Protective of tissues, reparative 3. Healing by

primary intention 4. Lack of pain or complications postoperatively. ● The flap surgery should not be initiated until one or two months after completion of the hygienic phase

of the periodontal therapy. ● The 1st incision "internal reverse bevel incision" should be made with a knife that can be directed

parallel to the long axis of the tooth. ● 2nd incision is made around the neck of each tooth from the bottom of the pocket to the alveolar crest.

Vertical gingival releasing incision usually is not needed. A full thickness flap is elevated for only 1-2 mm from the alveolar crest as needed for access to the root surfaces and the interproximal one.

Page 19: Decembery RQS – NBDE II

Combined remembered Qs 2020

● 3rd incision is made with a narrow interproximal knife. The buccal and/or lingual flaps are deflected by a periosteal elevator on top of the alveolar crest to dissect free the collar of gingival tissues, which is been separated from the buccal and lingual gingival flaps and the teeth. The separated collar of gingival tissue is then removed with curettes.

87. Gingivectomy contraindicated in- if the pocket is below the crest of bone ● Access to osseous structure is critical or ● Gingival attachment is inadequate (minimal) or absent, ● Bottom of the pocket is apical to the mucogingival junction, ● Esthetic considerations, anterior region of maxilla

88. Peutz- Jegher Syndrome

mutation of tumor suppressor gene stk11 89. Apexification,Pulpectomy questions- 3 questions on them 90. Irreversible pulpitis questions 91. Dentigerous cyst- cyst with epithelial lining filled with clear fluid attached at CEJ 92. Gorlin Goth Syndrome which cyst- OKC 93. Attrition question 94. 3 years ago pt had trauma to the mandible radiographically large radiolucency. What is your next step in treating this patient - check for vitality and do RCT for non vital teeth 95. Chronic alcoholic abuse 4 molars extraction which analgesic- options Ibuprofen,acetaminophen,hydrocodone,acetaminophen + hydrocodone Ibuprofen- Use with caution in patients with hepatic impairment; patients with advanced hepatic disease are at an increased risk of GI bleeding with NSAIDs. Acetaminophen- Avoiding use in patients with advanced chronic liver disease or cirrhosis who are actively drinking alcohol, malnourished, not eating, or receiving a concomitant interacting medication. Hydrocodone - Use with caution in patients with hepatic impairment; dose adjustment may be needed. 96. Premolar class 5 cavity which filling- GIC 97. Adolescent Xerostomia due to- Mouth breathing 98. Catastrophizing- It is a common cognitive distortion that makes us anxious, depressed and unmotivated. It's basically expecting the worst of current situation or of future situation.

Page 20: Decembery RQS – NBDE II

Combined remembered Qs 2020 99. Fetal alcoholic syndrome c/f except- prominent philtrum&sharpe nose 100. Peripheral odontogenic fibroma-2times 101. Marginal ridge width in occlusal rest- 1.5mm 102. Malignant melanoma common on- palate,gingiva 103. Picture of blue Nevus 104. PSA artery on X-ray identification 105. H2 receptor blocker MOA- deceases gastric production 106. Pt has allergic reaction,hypotension,Bronchoconstriction first line of treatment for this patient- Epinephrine 107. Osteogenesis Imperfecta related to- Dentinogenesis imperfecta 108. Neck trauma pt for airway- lift head back 109. Primary stress baring areas of mandible- buccal shelf & alveolar ridge Note; (Primary in maxilla is alveolar ridge n secondary is palatal rugae mandible it is buccal shelf and secondary alveolar ridge) 110. Mnodulesonths baby presents with midpalatine whitish on examination they are not getting rubbed off with pressure what are they- Gingival cyst of palate 111. Deepest part of occlusal rest- fossa 112. Benzodiazepines antagonist- Flumazenil 113. Opiods antagonist- Naloxone 114. Ruler used in ceph for- magnification 115. High copper and low copper amalgam identification by seeing the pictures 116. Osteoporosis X-ray 117. ASA 4 question 118. LA calculation easy one 119. Collimation- lead 120. Infection from lower 2nd premolar goes into which space- sublingual space 121. evaluation of 2 groups A and B with 2 drugs for same period, what type of study- clinical trial Cross sectional study 122. A question on Paraphrasing- Tell the patient what you think you understand 123. Acid etch does all except- improve esthetics 124. Sterilization monitoring by- biological agents(spores of bacillus steothermophillus) 125. Pt has nocturnal pain- irreversible pulpitis 126. Medically compromised child- Chlorhexidine mouth wash 127. Hyoid bone - they asked like what you see beside C4 vertebrae

1. Fungiform papilla swelling. Sloughing of the tongue, red and swollen. Fever. → Scarlet fever.

Page 21: Decembery RQS – NBDE II

Combined remembered Qs 2020 2. Picture of red patch on tongue, and the question says that there are same red patches on palate, too → Syphilis. 3. X ray showed multiple radiolucencies in skull. P’t has bone pain. Multiple myeloma 4. 2 questions on eagle syndrome. P’t has pain when turning his head and swallowing. 5. In which condition does p’t get thinner hair? →Hypothyroidism. 6. Hypercementosis in which disease? →Paget’s 7. Antidote for opioids. →Naloxone. 8. Antidote for BZDs. →Flumanezil 9. Just cemented an onlay, 1 hr later p’t returns complaining that every time he bites, there is sharp shooting pain. →Hyperocclusione 10. Children with HIV are likely to get what kind of infection? →Bacterial 11. Why shade selection before RD application? →dessication makes tooth look whiter 12. What receptors do opioids act on. →Enkephalin (which is another na)me for opioid receptor). 13. Mechanism of BZDs. →GABA receptor agonist. 14. Posture of p’t just after he got an asthma attack and treated with B2 agonists? →Upright, and a little bit crouched. 15. Adult can make decision for old dude under what circumstances. →Power of attorney? 16. A dentists reports to insurance that he just did a 2 surface restoration. However, the insurance only paid for 1 surface restoration. →Downcoding. 17. 2 questions on mechanisms of antitumor drugs. Forgot the details, they all looked the same to me. 18. Higher concentration in gingival crevice? →Tetracyclines. 19. Cannot be used in p’ts with decreased plasma esterases? →Articaine (has an ester side chain).

Page 22: Decembery RQS – NBDE II

Combined remembered Qs 2020 20. What kind of caries is increasing? →Root caries (due to more elderly people). 21. Which of the following root caries is active? →The one with soft lesion and plaque covered. 22. P’t just got trauma and broke his central incisor, horizontal root fracture in middle third, what should you do? →Splint and follow, if necrosis RCT. 23. What will you not do in a through and through furcation defect? →GTR. Other options were tunnel, hemisection and extraction. 24. Highest chance of 2 canals in which mandibular teeth (Options: lateral, canine, 1pm, 2pm). →lateral 25. Perforation of maxillary central incisors during RCT opening? →Mesial 26. You give p’t a IAN block. His tongue is numb but not his lip. What do you do? →Another IAN. 27. IV sedation adverse effect? →Hypoxia 28. P’t taking NTGs 2-3 times daily →ASA IV 29. What is the advantage of brushing your tongue? →Get rid of malodor. 30. Highest concentration of NO? 70% NO/30% O2 31. A doctor uses a new technique for diagnosis. As a result, lots of people with the disease are classified as not having the disease. The test has low ___? →Sensitivity 32. Pano of a pterygomaxillary fissure. 33. X ray of a p’t with enlarged pulp occluso-apically. →Taurodontism. 34. Why can’t you use tetracyclines along with penicillins? →Inhibit bacterial growth. 35. The century-old debate about the dentist who gets pissed off when working but keeps his cool when working, but as soon as the patient leaves the assistant drops something and he yells at her. I still don’t know what the correct answer to that question is btw. There seem to be two different opinions in the group.

36. H2 receptor blocker MOA→ldeceases gastric production 37. A question about pierre-robin syndrome (micrognathia,cleft palate and glossoptosis)

Page 23: Decembery RQS – NBDE II

Combined remembered Qs 2020 38. Aspirin acts on? trombaxane A2 Rqs 1.Dissensitizing ingredient percent: potassium nitrate 3-5 % 2.Doses for antibiotic prophy for prosthetic heart valve, allergic to ampicilin : clindomycin 600mg 3.Cleft lip what week in utero: 6 to 9 week 4.Aspirin adult max dose: 4000 mg 5.Px with HIV what can you sedate them? Midazolam 6.Amalgam ingredient to decrease corrosion?Chromium 7.Restoration on cusp indicated for? A) resistance b) retention 8.What spore hard to sterilize: bacillus thermophillus 9.Cyclic neutropenia: CBC 10.LA doesnt work on what kind o perio- abscess 11.Tooth most susceptible to coronal caries-mandibular first molar 12.Invisalign edge importance? Hue, chroma translucent value 13.What holds sealants: mechanical, deep fisures 1sd14.Extrusion of posterior teeth causes? Deep bite, incisor intrusion, incisor extrusion 15.Probing numbers is an example of?? Nominal ordinal 16.Most esthetic pontic?modified ridge /ovate 17Least likely to produce gingival hyperplasia? Nephidine, propanolol, diazepam, phenitoin 18.Spectrum of penicillin? Narrow, broad, 19.Child with bad hygiene what ortho tx? Removible, fixed, partial removible

Page 24: Decembery RQS – NBDE II

Combined remembered Qs 2020 20.Kinetic frug question that doesnt matter about dilution? First order single compartment second compartment 2 order 21.Overnightpain: naproxen 22.Epithelium for implants attach: hemidesmosomes 23.Contraindication to implant placement: adolescent 24.Nerves from infraorbital- MSA /ASA 25.What nerves moves upper eyelid?

The oculomotor nerve (CNIII) innervates the main upper eyelid retractor, the levator palpebrae superiorus, via its superior branch. 26.What will you not see on 3year old teeth? Pellicle, calculus, plaque 27.Wich IL causes bone loss? ILL 1

Interleukin-1 (IL-1), a cytokine best known for its ability to stimulate lymphocyte proliferation, has recently been shown to stimulate bone resorption and modulate bone formation in vivo.

Yale researchers learned that interleukin-1 not only causes a loss of bone but has been shown to increase the synthesis of collagen, a connective tissue protein, which can cause scar tissue in the abdomen. Symptoms Linked to Interleukin-1 28.Corrosion resistance: chromium

Corrosion resistance of various steel types increases with increase in chromium content. 29.Sibilant position:

Words with the sibilant sound (hissing sounds) are pronounced correctly with the incisal edges of maxillary and mandibular almost touching. These sounds are usually produced between rest and the occluding position. 30.Dont need radio for? Hutchinsons teeth 31.Chronic vs suppurative(acute) : percusion 32.Endo perforation on maxillary central on what area? Mesial 33.Retention pin per cusp??

Page 25: Decembery RQS – NBDE II

Combined remembered Qs 2020 34.Composite photo initiator: camphoroquinone 35Varices in tongue: hypertension ` A varicosity is a condition indicating an enlarged and tortuous vein. Previously described lingual varices referred to sublingual varices on the ventral surface of the tongue or floor of mouth. This represents the first reported case of dorsal tongue base varices in a patient with portal hypertension. 36.Desquamative gingivitis causes: lichen planus 37.Epulis fissuratum resembles : fibroma 38.Absolute contraindication for endo: leukemia, HIV, rheumatoid artritis 39.Tb: painless ulcers 40.MRI: radio waves 41.Shortness of breath, pedal edema: CHF 42.Side effect for nitroglycerin? Adverse Reactions

>10%: Central nervous system: Headache (patch, ointment: 50% to 64%; sublingual powder, lingual spray: >2%)

1% to 10%:

Cardiovascular: Hypotension (≤4%), syncope (≤4%), peripheral edema (lingual spray: ≤2%)

Central nervous system: Dizziness (>2% to 6%), paresthesia (>2%)

Gastrointestinal: Abdominal pain (lingual spray: ≤2%)

Neuromuscular & skeletal: Weakness (all sublingual forms: ≤2%)

43.O in PICO ( population, intervention, comparison, outcome) 44.Ruler on lateral cephalogram: magnification

01/03/2020 Know endo pulpal diagnosis •Perio diagnosis

Page 26: Decembery RQS – NBDE II

Combined remembered Qs 2020 •Hardest 3rd impacted molar to extract- vertical impacted •Pulpotomy vs pulpectomy for peds •Apical scar •Apexogensis, apexification indcations •Sinus tract drainage- chronic apical abscess •Perio abscess vs endo abscess (wide lateral defect, percussion) •Endo file separation= best prognosis for vital tooth without par oYou separate an endo file 3mm from the apex and obturate above it... which case will show the best prognosis? a. vital pulp w/ no periapical lesion (yes) •Tooth avlusion splinting-7-10 DAYS •Most common permanent tooth missing - 3rd molar, then the mandibular 2nd premolar, then the maxillary lateral incisor. •Den in dens tooth? maxillary lateral incisor •Replacement resorption and ortho- ANKYLOSIS •Straight line access easier in short teeth •Red complex bacteria = Porphyromonas gingivalis, Treponema denticola, and Tannerella forsythia. •Some indirect restoration questions •Forceps for extraction permanent max PM-150 •Burs for smoothing out preps? - More flutes and shallow •Implants= 3mm between implants, 2mm from vital structures, high torque low speed •: You did a prep with high speed + diamond bur and tooth is sensitive, what is it about bur and handpiece that it caused sensitivity? A) Desiccation B) Traumatized dentin C) Heat 1.What would cause displacement of odontoblastic processes? Thermal Desiccation Mechanical Chemical •Whats the indication for # of pins to put when you are placing an amalgam (idk) •Indications for onlay- I put not enough dentinal structure under cusps •Cement veneers and see brown discoloration on margins? I said resin or microleakage depending on q •Inow indications for composite (not good for big posterior fillings) •Why GI is good for class 5 •**know HUE, VALUE, CHROMA (chroma=saturation) , choose hue first… know this well •Most rention of crown? Axial taper (came up 3x) •Which of the following do you not do in cementation of a porcelain crown: etch enamel with hydrofluoric acid •What could the reason be if you see opaque white porcelain in the incisal 1/3 facial of the PFM crown: Inadequate reduction of the inciso facial part of the tooth •Most common complication of internal bleaching: cervical root resorption (this wasn’t an answer choice on mine)

Page 27: Decembery RQS – NBDE II

Combined remembered Qs 2020 • Pontic for FPD should rest on the soft tissue without blancing •What pontic can you put on posterior FPD that will change esethcis and phonetics and cant be put on anteriors? (I put hygenic b/c idk) •Strength of soldered connected of FPD is due to increasing height •No occlusion q •A LOT of q facebow registration •Question about thyroid storms… •If pt with CD is talking and denture keeps “popping” off what is going on ? I put overextended borders •Pt feels like lip is sticking out with CD (I put adjust labial flange) •Light intiator for composites= camphoroquionine •QUESTION: What happens when you take an impression & lip immediately swells? Angioedema (allergy reaction) •Imbibition and syneresis affect which one the most a. Reversible hydrocolloid •Question about which is the best for indirect restorations and impression material (asked about charactiersics of the material but not the names of the material) •Best interocclusal record for MIP? (I put just cusp tips with some light showing ) •Fl- replace hydroxyl •Osteomas- gardners •Asked about cleiocraial and other ones that cause supernumary •Downs syndrome- lower affinity for caries •dI and dysplasia •how to treat internal resorption •pagets association with cancer (osteosarcoma) treated with bisphos •a lot of osteoradionecrosis (u can figure it out… not hard) >65 high risk of ORN •pyogenic granuloma and pregnancy •neurofibromatosis- lische nodules, etc •cysts not seen on radiograph •pemphigoid – basement membrane •histoplasmosis orally looks like tb •pleomorphic adenoma- most common salivary gland t •Kaposi sarcoma •Congential epulis •A good antifungal that can be taken orally and have systemic effect. Idko fluconazole •Actini cheliltis= loss of vdo •Symptoms of acute herpetic gingivitis •Warty and cauliflower? Hpv v •Oliodontia? Ectodberm

23rd rqs AOT Xerostomia pilocarpine Erythema migrans (description)

Page 28: Decembery RQS – NBDE II

Combined remembered Qs 2020 Gingival hyperplasia drugs Resceptors (Epi not epi) Multiple myeloma first symptom (bone pain) Dementia: short term memory loss Drugs mechanism of action Protective stebilization where to use

Jan 24th

-If you think the drug causes serious problem, should you repor to? FDA, CDA -Which muscle closes the tongue? Genioglossus, hyoglossal .. -Common fungal infection? Histoplasmosis, candidiasis, blastomatosis -Lesion at the basement membrane? Pemphigoid, pemphigoid, erosion lichen planus -Reduction of the crown should look like? Flat and round, follow occlusal morphology -As you get older, people lose which teeth first due to periodontitis? Max molar, Man molar -bulimia? Caucasian women, Caucasian men, black men, black women -Heparin? PTT, PT -Naloxone: completive antagonist? -Provisional for veneer? Bis-cryl, methyl methacrylc, composite, shell -got a patient who currently prescribed penicillin V for pain. He has a heart condition and need prophylaxis before procedure begin. which prophylaxis should you use? Amoxicillin 2g or clindamycin 600mg -A tooth was root canal treated and there was a small radiolucency under the apex. After 4 years it is still there. What do you think it is? Apical scar, apical cyst, forgot other choices -Which experimental design would you use to study gastric bypass and nutrition? Cohord, randomized clinical trial, observational -primary stability of implant osteo integration is? Patient health status, primary stability of the implant -you give an IA nerve block and the needle inserted medially from the pterygo raphe. Which two muscles involved? medial pteryroid, superior constricture, buccinator -children with congenital heart defect seem emotionally and physically dependent because? limit physical abilities..., parents overly protective, hospitalized frequently… -Coronal caries effect least on which tooth? Max molar, mand molar, max central, man central -Coronal cares affect most on which tooth? Max molar, mand molar, max premolar, man premolar -Patient has a deep carious, non-restorable #23 and the dentist recommended extraction. No medical history. The patient’s spouse told the dentist that the patient has frequent urination, thirst, hunger and lost a lot of weight recently. What is the recommendation for the treatment?

Page 29: Decembery RQS – NBDE II

Combined remembered Qs 2020 Extract #23 with local anesthesia,refer to physician refer to the periodontist, Refer to physician -How many % of adults have herpes simplex virus? 20-30%, 50-80%, 100% -calcific metamorphosis ? internal root resorption, external root resorption, something else -sterilize plastic instruments using? glutaraldehyde, phenol, alcohol.. -a pt has deep pocket depth, which procedure would preserve the gingiva? split palatal, apical positioned flap, lateral position flap, gingivectomy -contraindication of root canal therapy? HIV, leukemia, recent MI -Which facial space does infection of Lugwig NOT travel to? Submental, Sub mandibular, sublingual, retropharyngeal -maxillary artery is from which branch? internal carotid, external cortical artery -Development of cleft lip? 2-3weeks, 6-9weeks, 12-15 weeks, 18-21 weeks - You observed the a child was very upset going to the dentist due to a past experience at another medical office. What it is? Association, modeling.. You observed the a child was very upset going to the dentist due to a past experience at another medical office. What it is? Association, modeling.. -Amantadine is an? antivirus, antifungal… -optimal periodontal maintenance is in ? 1 month, 3 month, 6, 12 months -characteristics of body dysmorphic disorder -common sign of mandibular fracture? trismus, parasthesia -symptoms of thyroid storm versus anaphylaxic shocks - Thyroid storm-high temperature, fast heart rate, nausea, vomiting, confusion, perspiration, diarrhea.Anaphylaxis-low BP, difficulty breathing, fast heart rate

More RQS *What is component in algintae that react with calcium n give the desored working time. Trisodium phosphate. ***what is effect of post in post n core: Ferrule effect Retention of core ( i chose) Other choices dont remeber. * what is the factor that prvent the fracture of tooth with post n core: A. Reaking tooth structure ( i chose this) B. Post length C. Post width * clinic closed for 2 weeks what is best course of action. Leave without pay

Page 30: Decembery RQS – NBDE II

Combined remembered Qs 2020 Termination Non patient care dental practice Part time job * what is behavior shapping * child had multiple vist to clinic, after multiple visit what is cause of objective fear: Previous experiences ( i chose this) F4om parents ***what antibioic for mycoplasma Bacitracin Penicillin Clarithromycin Dont remebr other choice ***simple diagnosis for asymptomatic apical periodontitis, irreversible pulpitis. ** pic of pt had untreated caties n cavitation with small swelling on gum ,asked what was it? Parulis * what is parulis associated with ? ** intial dose of epinephrine in anaphylactic shock: 0.3 mg 0.003 mg 30 mg * i had an opg ,pt hd a lesion in maxillary sinus had no pain. What was dagnosis? Chronic maxillary sinusitis Antral Rention pseudocyst?? * yellow painless hard bony swelling in floor of mouth. Diangosis Calcified lymphiepithelaul cyst? Siaalolith ** * all muscles hav e attachment at mandible ramus excpet: Lateral pterygoid Medial pteryhoid Massater These were the only options, i choose leteral * other name for eosinophilic granuloma: langerhan cell histiocytosis * what is the sructure at rooth of maxillary tooth: Maxillr sinus Zygoma ( i chose this)

Page 31: Decembery RQS – NBDE II

Combined remembered Qs 2020 * what causes steep mandibular plane angle? A. Increase vertical grwoth patten with increase lower facial height( ans) B. Deep overbite with incread3 lower facial height * when we do molar uprighting what occurs: Lack of Anchorage Mesial movement of molar Distal movement. ***while access preparation if u move the bur mesial or dial perforation of what tooth occur? Maxillary first premolar( i chose this but dont know) Mand molar * flossing doesnt not clean the proximal surface of which tooth? MesialMaxillary first premolar Distal of max lateral incisor Dont remeber other options ** Dont remember other option * pt was had depression n hypertension which antidpresant with propanol will cause severe imcrese in blood pressure: Imipramine Fluoxetine * which of following describe extent of periodontitis: Chronic Localized * gingival enalrgement occur in all of following excpet: Gingivostomatitis Desquamtive gingivitis Leukema Pregmacy * 50 pound pt, haow many cartridge of lidocaine 2 4 6 8 **opiod odesnt cause Somnolence Periperal pain inhibition Contipation Dont remeber other choices * pt recently diagnosed with alzheimer what will u do: A. Start **what treatment for tmj?

Page 32: Decembery RQS – NBDE II

Combined remembered Qs 2020 Splintnonly * common site of salivary gland tumor? Palate minor salivary gland Parotid * hyperventilation symptoms Bradypnea Dizziness n lightheadedness * down syndrome all except: Delayed erution Maxroglossia Epicanthal fld Short strature * what is symptom of MI: A. Pain , ECG abnormalities, elevated serum enzymes B. Pain, frictional rub, elevated CPK ** CI for endodontic treatment: A. Diabetes B.uncontrolled hypertension. **what is wheezing A. on ausculation an exhaltion sound is heard that pops open n close B. Occur due to flow of ir in narrow bronchi.

RQ-17/18 1. Hypoglycemia signs except- 2. Shock sign except- 3. Pic of Healthy pt having -tongue lesion also on palate - candidiasis 4. For saliva control if pt given Anaesthesia- glycopyrrolate (in options) 5. Ace inhibitors - moa 6. Beta blockers- moa in angina kkkk 7. Propanalol + epi- which receptors work 8. Vasoconstriction by which receptors 9. Pics- fibroma vs neuroma,geographic tongue, median rhomboid glossitis 10. Digoxin+ diuretic 11. Hypertension stages

Page 33: Decembery RQS – NBDE II

Combined remembered Qs 2020

12. Pt is havi ng all crowns n implants - she is confused hw to take oral hygiene-ask

her to watch vedio on oral hygiene, give chlorhexidine mouth wash 13. Dual bond 14. Cheek biting - how to correct it - grind the lower buccal 15. Beading on major connector -to avoid food impaction and retention 16. Tongue tie - pt with denture what problem she will face - dislodgement of denture 17. A lot of patient management 18. popular licensure question 19. wine 2 glasses rrrquestion 20. doctor angry with pt-but put it on assistant 21. 2 ethics - when pt ask for extract all teeth to give CD but he is having good teeth - 22. Smoker pt what not to give for smoking cessation - smokeless tobacco 23. Many Endo vs perio diagnosis (should be so clear about this) 24. Diagnosis suppurative periodontitis 25. To diagnose class 1,2,3 not from pics of molars but from lat ceph 🥺 🤷🏻 ♀ but they do

particular ask about angle classification without molar pictures 26. Transillumination 27. How to change hue 28. Histamine cause except 29. If pt very anxious - what to give a day before when he is coming in for treatment 30. Pt is taking so many medication- TCA,smoking cessation-what will affect oral hygiene -

xerostomia 31. Pt is interested in smoking cessation- how will u help except -some motivational

interview n weird options 32. 16 yr old pt- lost per 1st molar - how to replace -Rpd, Fpd, implant 33. Fluoride supplements at what age start 34. Pregnancy pain killer safe acetaminophen 35. Turner hypoplasia 36. Randomized clinical trial - to reduce bias 37. Aspirin toxicity 38. EDTA - chelating agent 39. Some RQ from strawberry file,elmastro file, I don’t remember question but I have seen

them in some files.

Page 34: Decembery RQS – NBDE II

Combined remembered Qs 2020 40. Be clear with your basics you gonna need them in exam no memorizing the RQ,options r

different n complicated so try to find the correct ans n read that topic in detail 41. Day -2 - pt with so many different medications even you haven’t heard some of them but

they gonna ask u questions related to the main ones 42. Pharma- mostly moa of drugs n some basic interactions 43. On day 2 try to be as conservative in treatment as possible but in some question they

gonna ask you best treatment - diastema t/t- crown, veneer,composite 44. Some pt management even in day 2 45. Pt ask not to tell my family about cancer 46. Basic signs n symptoms-Asthma, hypertension, hypoglycemia, shock , penicillin allergy 47. Pt is hypoglycemia, 1st thing to check if he is unconscious 48. Asthma pt - what’s important -has to get asthma inhaler 49. What the pt can have if she is using beclomethasone Inhaler - candidiasis

DISHA’S – 22 AND 23 JANUARY

1. Combination syndrome features - tricky words.. I almost thought nothing matches

2. Mucocele - due to trauma

3. Mucous retention cyst - due to plugs

4. Exostosis and Enostosis

5. Subman infection spreads to mediastinum

6. Congenital epulis - there was some description and only

this matched partly .. again one of the question where I

thought nothing matches.

7. Diff between primary herpetic stomatitis and recurrent

herpes .. there were pictures to identify

8. LA calculation but units in cc ( know it.. cc is equal to ml) ..

but the calculation was very weird .. I had to choose LA

amount and epi amount.. nothing in LA amount matched but in epi it did so I just chose that

option

This table really helped for the epi calculation.

9. When does teeth start showing tetracycline stains ? Prenatal, at birth , 3 years , 6 years

Page 35: Decembery RQS – NBDE II

Combined remembered Qs 2020 1. Burs used for different finish lines. Chamber. - round end

tapered

2. Description of a dark blue lesion - cavernous hemangioma

or amalgam tattoo

3. Location of pier abutment

4. Minimum clearance for occlusal rest

5. Difference between crystalline and amorphous .. their

melting points atomic range

6. Description of a lesion (except question) - I chose verruca

vulgaris out of herpes, pemphigus, and pemphigoid

7. Which pulp horn is exposed first of primary man molar ? Mesial of 1, distal of 1, mesial of

2, distal of 2

8. Emergency drug for bronchospasm and something else.. I think we discussed epi on the

group

9. Stannous floured mouthwash for medically compromised patients

10.10% chor varnish best use

11.2 - 3 questions on endocarditis prophy - needed in aortic

valve replacement and the dosage and all 12.Pictures and description on granular cell tumor,

lymphangioma

13.Autoimmune dieases most common in which

demographics

14.Targetpoid lesions

15.Topical agent for Perio disease

16.Rheumatoid arthritis - oral manifestations

17.Pierre robin syndrome

18.Taurodontism - which developmental stage

19.Cafe au lait spots

Page 36: Decembery RQS – NBDE II

Combined remembered Qs 2020 20.Mccune albright syndrome

21.Weird radiograph.. none of the options matched again 22.Cetirizine side effects

23.ALARA principle application on a patient based radio

question

24.Keneddys classification identification on a cast 25.Peripheral cemento osseos dysplasia

26.Peripheral ossifying fibroma

27.Peripheral giant cell granuloma

28.Propanolol alters taste sensation 29.Radiograph with PSA seen in max sinus

30.Alzeimers medications MOA

31.Warfarin therapy stop before 2 days

32.Metabolic disorder except question

33.Perio prognosis is better for pulp necrosis leading to Perio

or Perio disease leading to pulp necrosis 34.Hematoma vs hemangioma - which one can be

blanched .. more on such red lesions

35.Effective only against anerobes - metronidazole 36.Contents of feldspathic ..

37.lots of questions on components of dental materials 38.Ingredients in chemicals used

39.Lots of questions on their chemical reactions .. I honestly

don’t remember because I didn’t understand the questions .. too long questions and options

40.Cement used in porcelain veneer resin

41.Radiograph showing radiopaque wavy lines - resin 42.Question on thyroid therapy - ans

was radioactive iodine

43.Question on ectodermal dysplasia but the option given was ectodermal hyperplasia .. so I

got confused

44.Treatment of OKC but it wasn’t asked directly,.. I had to find from the description that it

was OKC and then choose a treatment

45.Odontodysplasia

46.Endo questions on root perforation, pain on biting and

Page 37: Decembery RQS – NBDE II

Combined remembered Qs 2020 root fractures

47.Loss of attachment in a given case due to vertical root

fracture

48. Day 2 has typical questions on calculating missing

number of teeth in a mixed dentition pano

49.Lots of prostho cases..

50.Picture of inflamed minor salivary glands due to smoking 51.Precomtemplation question

on smoking but very twisted

words

52.Controlled substances ACT contains which drugs (except question)

53.Drug addict person - pain relief medication

54. Loss of attachment definition

55.Indirect pulp therapy definition

56.Pharmac questions on day one were very confusing. It

was about a lot of drug reactions.. pharmacokinetics and pharmacodynamics.. I did mental

dental vides, tufts pharma, el maestro,, all the other files and possible sources including DD ..

but they didn’t have any of these concepts.. lot of detailed question on microsomal and non

microsomal enzymes ,, their oxidation, conjugation and all

57.Day 2 pharmacy was easy and straight forward 58. Anatomy questions from part 1

59. Host modulation - doxycycline

60. PMMA reaction basics

61. lOts of questions on the basic actions for alpha and beta receptor . Know it well

Jan 26 1. Jan rq: very similar to this picture. Which sinus is not shown clearly? ethmoidWhich

facial profile? Straight, convex, concave

Page 38: Decembery RQS – NBDE II

Combined remembered Qs 2020

2. Jan rq: similar to the picture, the large lesion is in the middle of the central incisor, not

near the cervical. Abrasion, acidic chemical (erosion is not in the choices 3. got a patient who currently prescribed penicillin V for pain. He has a heart condition

and need prophylaxis before procedure begin. which prop hylaxis should you use? Amoxicillin 2g or clindamycin 600mg, erythromycin

4. How many cartridge of LA 2% lidocaine u can give to child 45 lbs1. 12. 33. 64.

9Ans 2 ( 3 cartridge ) 5. Patient has a deep carious, non-restorable #23 and the dentist recommended

extraction. No medical history. The patient’s spouse told the dentist that the patient has frequent urination, thirst, hunger and lost a lot of weight recently. What is the recommendation for the treatment?

Extract #23 with local anesthesia, refer to the periodontist, refer to the physician 6. You give an IA nerve block and the needle inserted medially from the pterygo raphe.

Which two muscles involved? medial pteryroid, superior constricture, buccinators 7. How many % of adults have herpes simplex virus? 20-30%, 50-80%, 100% 8. sterilize plastic instruments using? glutaraldehyde, phenol, alcohol... 9. Amantadine is an? antivirus 10. Coronal caries effect least on which tooth? Max molar, mand molar, max central, man

central 11. Coronal cares affect most on which tooth? Max molar, mand molar, max premolar,

man premolar 12. children with congenital heart defect seem emotionally and physically dependent

because? limit physical abilities..., parents overly protective, hospitalized frequently...

Page 39: Decembery RQS – NBDE II

Combined remembered Qs 2020 13. A tooth was root canal treated and there was a small radiolucency under the apex.

After 4 years it is still there. What do you think it is? Apical scar, apical cyst, forgot other choices

14. Which experimental design would you use to study gastric bypass and nutrition?

Cohord, randomized clinical trial, observational 15. calcific metamorphosis ? internal root resorption, external root resorption, something

else 16. Which facial space does infection of Lugwig NOT travel to? Submental, Sub

mandibular, sublingual, retropharyngeal 17. optimal periodontal maintenance is in ? 1 month, 3 month, 6, 12 months 18. a pt has deep pocket depth, which procedure would preserve the gingiva? split palatal,

apical positioned flap, lateral position flap, gingivectomy 19. contraindication of root canal therapy? HIV, leukemia, recent MI 20. You observed the a child was very upset going to the dentist due to a past experience

at another medical office. What it is? Association, modeling.. 21. primary stability of implant osteo integration is? Patient health status, primary stability

of the implant 22. maxillary artery is from which branch? internal carotid, external cortical artery 23. common sign of mandibular fracture? trismus, parasthesia 24. Development of cleft lip? 2-3weeks, 6-9weeks, 12-15 weeks, 18-21 weeks 25. characteristics of body dysmorphic disorder?

26. symptoms of thyroid storm versus anaphylaxic shocks

Page 40: Decembery RQS – NBDE II

Combined remembered Qs 2020 according to fb: Thyroid storm-high temperature, fast heart rate, nausea, vomiting, confusion, perspiration, diarrhea Anaphylaxis-low BP, difficulty breathing, fast heart rate 27. If you think the drug causes serious problem, should report to? FDA, CDA 28. Which muscle closes the tongue? Genioglossus, hypoglossal ... 29. Common fungal infection? Histoplasmosis, candidiasis, blastomatosis 30. Lesion at the basement membrane? Pemphigoid, pemphigoid, erosion lichen planus 31. Reduction of the crown should look like? Flat and round, follow occlusal morphology 32. As you get older, people lose which teeth first due to periodontitis? Max molar, Man

molar 33. Bulimia? Caucasian women, Caucasian men, black men, black women 34. Heparin? PTT, PT 35. Naloxone: completive antagonist? Morphine? 36. Provisional for veneer? Bis-cryl, methyl methacrylc, composite, Shell

FEBRUARY RQ 

1. epiphyseal plate? Synchondrosis

2. Implant to implant? 3 mm

3. Dentin binding and 2 canals?

Page 41: Decembery RQS – NBDE II

Combined remembered Qs 2020 Fusion Germination (only has 1 canal)

4. C.I. For RcT? Vertical roots fracture

5. CEJ apical to crest? 2 mm If they talk about emergence profile it should be 2-4 mm

6. Biological width 2mm CTand JE

7. Xray denture not fitting? Pageants ds

8. Tooth brush abrasions

9. tooth loading? Abfraction

10.Gingival hemorrhages and skin lesion? Leukemia

11.Pt Gagging? Inc VDO *( causes of gagging : increase of vdo - causes inadequate post extention

and food to be trapped, bulkiness of denture Hh palatal seal ( increase deph of posterior palatal seal cause unsealing of the denture) Buccal flange

12.Difficult to maintain space in early loss ? 2 molar* 1 molar

13.Sensitivity in test

Page 42: Decembery RQS – NBDE II

Combined remembered Qs 2020

14.Dose epi 1.7 Ml 1:5000? 0.036 mg or 36 mcg

15.Drug run report to FDA

16. Impacted tooth maxillary Canine

17. Alv. Osteitis most common? Mand. 3molar

18. Initiation of caries? Strep mutants

19. Progression of caries? Lactobacllus

20.Frequent urine in 3 trimester? Fetus pressing on bladder

21.Asthma? Albuterol or Surbetol (Ventolin) opening passage for asthma

22.Morphine antagonist? Naloxene

23.Benzodiazepine. Antagonist: Flumezenil

24.Tongue blue lesion? Blue mass on the tongue indicates the presence of : Lymphangioma Hemangioma Neurofibroma Pyogenic Granuloma Chronic Inflammatory gingival hyperplasia (epulis) Squamous cell carcinoma

25.Papilloma pic

26. Vercuous carcinoma pic

Page 43: Decembery RQS – NBDE II

Combined remembered Qs 2020

27. Pseudomemb colitis? Clindamycin due to over proliferation of Clostridium Difficile Bacteria

28. Tetracycline not with? Penicillin for example ( don’t mix bactericidal with bacteriostatic)

Make the list of cidal and static that we cant mix CAMP FV Celphalo Aminoglycosides Metro Penicillin Fluoroquin vancomycin

29.Lisinopril ? Lisinorol - angioten inhibitor used for hypertention (Ace – angiotensin converting enzyme

inhibitors)

ACE inhibitors, “inhibit” the conversion of inactive Angiotensin I→Angiotensin II (a vasoconstrictor). STIMULATE THE RELEASE OF ALDOSTERON

30.Chronic periodontitis? Black male

31.Oral cancer poor prognosis? Black male

32.Antibiotic and surgical therapy? ANUG, ( just do debriment) LAP *

33. Implant most retentive in? Anterior mandible* ( bone more dense ) rule : more in Mand than max , more in

anterior than post

Page 44: Decembery RQS – NBDE II

Combined remembered Qs 2020 Posterior mandible

34.Skin nodules and mouth pigmentation? Neurofibromatosis

35.Common in Gardner and pouts Jeghers? Polyps

36.Mucus retention on lips? Trauma* Mucus plug (choose this one if trauma is not there) Stone

37.Class 2 chances in permanent? Distal step in primary* (remember: the teps always stays the same or move a little mesially) Mesial step

38.maxillary hypoplasia ? Class 3

39.Nitroglycerin ? Angina

40.Not required in angina pt? Antibiotic prophylaxis

41.Attachment loss? CEJ to base of pocket

The periodontal examination includes probing pocket depth (distance from the gingival margin to the base of the pocket) and clinical attachment level (distance from the CEJ to the base of the pocket).

42.Sturgeon Weber? Port wine

Page 45: Decembery RQS – NBDE II

Combined remembered Qs 2020

43.Root mand 3 molar lost into? Sub mandibular

44.Herpes gingivostomatitis? 2 years

45.Non malifience? Referring and knowledge updating

46.Splint implant 4

47.Amalgam pin

48. .m , 7Horizontal root fracture ? Splint and observe ( 3 PA varying vertical position) If coronal (rigid splint - 3 months) If ½ root (flexible – 3 weeks) If apical (flexible 2 weeks) Chance of necrosis 25 %

49.3 year old central incisor intrusion? Radiograph and antibiotics* ( just x -ray, if not close the permanent, leave it there, no need

for antibiotic) Exo

50.Hypodontia a defect on which stage? Iniciation – anodontia or supranumerary Bud Stage – Hyper or Hypodontia* Cap Stage – Dens in dente, gemination, fusion and tubercules Bell stage – dentinogenesis and amelogenesis imperfecta, macro and microdonty Appositional _ enamel dysplasia, concrescence, enamel pearl

51.2 cm white patch on sub mandibular duct? Incisional biopsy (more than 1 cm , make incisional) the only white patch that u make

cytology or brush biopsy is pseudo membranousn

Page 46: Decembery RQS – NBDE II

Combined remembered Qs 2020

52.0.5 cm? Excisional biopsy

53.best test for pdl inflammation: Percussion

54.Buccal Roots max molar more mesial to palatal, position of tube Mesial * Distal is the answer Inferior Superior

55.Cells on tension side Osteoblasts Clasts * (seen on the compression site) Cyte PMN

56.Kid after active ortho treatment, retention by (history of poor oral hygiene) Fixed Removable * Other 2 options

57.Lesion not on tongue Peripheral granuloma * – periferial giant cell granuloma? – limited to alveolar ridge

anterior to first molar Ectopic thyreoid 2 more that I don’t remember

58.Osteoporosis Radiographically thin trabecular * less osteoid calcification other 2 options

59.Palatial root more mesial to buccal root, porition of tube

Page 47: Decembery RQS – NBDE II

Combined remembered Qs 2020 Mesial * slob rule Distal is the answer Superior Inferior

60.Not a method to check sterilization Chemical ( for instruments that cant be heat sterile) Mechanical Biological (most common used) Electrical *

61.DKA Hypoglycaemia Hyperglycaemia Hyperglycemia * - diabetes ketoacidosis 1 other options

62.Periapical cementoblastoma Is a relatively rare benign neoplasm of the cementum of the teeth. It is derived from

ectomesenchyme of odontogenic origin. Less than 0.69-8% of all tumors of the teeth Osseous dysplasia

63.Gingivectomy indications Suprabony indicated only in keratinized area

64.Carcinoma in situ. Erythroplakia Leukoplakia white sponge nevus lichen planus C in S: A group of abnormal cells that are found only in the place where they first formed in

the body (see left panel). These abnormail cells may become cancer and spread to nearby normail tissue (see right panel)Lateral Periodontal Cyst

Page 48: Decembery RQS – NBDE II

Combined remembered Qs 2020

65.Safer drug in HIV Erythromycin Amoxicilin Ibuprofen *

66.Class 3, 4, 5 composite bevel 15 30 45 * 60

67.Swelling of lip Mucus retentioncysts Minos SG something Sialolith

68.Swelling of lit after anaesthesia injection Angioedema * Anaphylaxia other 2 options

69.Nitrous Oxide And epinephrine antagonize by Physiologic * competitive 2 other options

70.Lots of questions on finishing lines Axio gingival – Class 1 - 2 Axio pulpal – class 5 Grooves – proximal

71.Hemorrhagic shock first sign Tachycardia * Hypotention other options

Page 49: Decembery RQS – NBDE II

Combined remembered Qs 2020

72.Epinephrine works on Alpha 1 * Alpha 2 Beta 1 Beta 2

73.3 questions about DKA diabetes ketoacidosis Lets search for everything about this

Tino Feb 7,8-2020 RQs (216 Qs)

1.QUESTION: Case: Patient with tooth that has sensitivity that lingers with thermal test, sinus tract, and positive to

percussion, what does the

patient have? Irreversible pulpitis with acute periapical abscess (other choices were Irreversible pulpitis with no acute

periapical abscess, and

2 other choices with reversible pulpitis in them).

2.QUESTION: Chronic periradicular abscess indicates: necrotic Pulp

Percussion- presence of inflammation in PDL or not

Palpation- spread of inflammation to periodontium from PDL or nothing

EPT- Pulp vitality, responsiveness (necrosis or not)

Thermal test (hot & cold)- pulp vitality. Hot (irrev), cold (rev)

3 QUESTION: Which is incorrect? Do EPT for traumatic tooth

4 QUESTION: When heat is applied to the tooth, lingering pain for several minutes indicates: irreversible pulpitis

5 QUESTION: What is diagnosis for lingering pain to cold and sensitivity to percussion? Irreversible pulpitis & acute

periapical abscess

- Usually periodontal abscess is sensitive to percussion, irreversible is usually positive to percussion

6 QUESTION: What is test to diagnose chronic periradicular periodontitis? Percussion

7 QUESTION: EPT does NOT indicate health of the Pulp, only vitality

8 QUESTION: How do you differentiate between an endo/perio lesion? EPT

Page 50: Decembery RQS – NBDE II

Combined remembered Qs 2020 9 QUESTION: Chronic endodontic lesion has what type of bacteria? Anaerobes

10 QUESTION: What is initial treatment of a combination perio and endo lesion? Perform endo with RCT first, then perio Sc/RP

11 QUESTION: What treatment is required with a tooth that has a draining sinus tract has been treated via RCT? No further treatment

12 QUESTION: A patient has a non-vital tooth & a fistula that is draining around the gingival sulcus. What kind of abcess is

it?

endo and perio at same time

perio and then endo

only endo

only perio v ,

13 QUESTION: There usually is no lesion apparent radiographically in acute apical periodontitis. However, histologically

bone destruction has been

noted.

a. Both statements are true

b. Both statements are false.

c. First statement is true, second is false.

d. First statement is false, second is true.

14 QUESTION: Least reliable test on primary teeth - Electric pulp test

- On primary teeth you don’t want to use EPT b/c thin enamel creates false results & after trauma, you don't want to use

electronic pulp

tester.

15 QUESTION: 7 yr old boy has vital pulp exposure of 1st perm max molar. What do you do for treatment? Pulpotomy

16 QUESTION: A 7-year-old patient fractured the right central incisor 3 hours ago. A clinical examination reveals a 2-mm

exposure of a "bleeding

pulp." The treatment-of-choice is

A. pulpectomy and apexification.

B. pulpotomy with calcium hydroxide. C. direct pulp cap with calcium hydroxide.

D. one-appointment root canal treatment.

17 QUESTION: Why would you do a pulpotomy in a mandibular first molar of a 7-year-old? To continue physiologic root

development

(apexogenesis)

Splint the tooth is for pt comfort

- Avulsion: 7-10 days non-rigid/flexibile splint, antibiotics

- Horizontal root fractures: Rigid splint, 3 months

- Extrusion: 2-3 weeks splint

18 QUESTION: Reason for failure of replantation of avulsed tooth: external resorption or internal resorption

19 QUESTION: Which is more damaging to the PDL? Extrusion, intrusion, lateral luxation, avulsión

Page 51: Decembery RQS – NBDE II

Combined remembered Qs 2020 20 QUESTION: Luxated tooth, negative EPT, why? disruption of nerves to tooth

21 QUESTION: Sodium hypochlorite is used for everything except? Chelation

- Bleach is not a chelating agent

22 QUESTION: Which is a chelator/chelating agent for endo? EDTA, sodium hypochlorite, etc.

- EDTA is chelator, removes SMEAR LAYER and inorganic material.

- NaOCl (sodium hypochlorite) only dissolves organic material, only disinfects & is most common irrigant.

23 QUESTION: Treatment for internal resorption: RCT

24 QUESTI ON: When a tooth is ankylosed, what type of resorption? replacement resorption

26 QUESTION: Incomplete removal of bacteria, pulp debris, and dentinal shavings is commonly caused by failure to

irrigate thoroughly. Another

reason is failure to:

A. use broaches.

B. use a chelating agent.

C. obtain a straight line access.

D. use Gates-Glidden burs.

27 QUESTION: You separate an endo file 3mm from the apex and obturate above it... which case will show the best

prognosis?

a. vital pulp w/ no periapical lesion (yes)

b. vital pulp wI periapical lesion

c. necrotic pulp wI no periapical lesion

d. necrotic pulp wI periapical lesión

28 QUESTION: Pt has crown cemented 2 weeks ago & is sensitive to pressure and cold, why? Occlusal trauma

29 QUESTION: Crack tooth syndrome is most likely found? Mandibular Molars

30 QUESTION: Cracked tooth with no pulpal involvement, what is the treatment? Endo, extracoronal restoration,

occlusion reduction, amalgam

with adhesive

31 QUESTION: pH that enamel starts to demineralize – pH = 5.5 (critical pH of developing cavities)

32 QUESTION: Most Cariogenic? Sucrose

- S.mutans adheres to the biofilm on the tooth by converting sucrose into an extremely adhesive substance called dextran

33 QUESTION: Where does caries start? Apical to proximal contact.

34 QUESTION: 40 y pt w/ all 32 teeth. No cavities. Has stain & catch in pit of molar. what do you do?

a. Watch & observe

b. sealant

c. composite

35 QUESTION: Main difference and advantage of using GMT instead of Enamel hatchet?

a. bi-angled cutting surface

b. angle of the blade

c. push/pull action instead of

Page 52: Decembery RQS – NBDE II

Combined remembered Qs 2020 36 QUESTION: What would cause displacement of odontoblastic processes?

Thermal

Dessication

Mechanical

Chemical

37 QUESTION: What is the most important thing for retention? Surface área

38 QUESTION: Most lab complain that the tooth is under reduced.

39 Not Sure answer: QUESTION: What causes the most retention of crown? Axial taper, surface area, surface roughness, retention grooves

40 QUESTION: Which of the following do you not do in cementation of a porcelain crown: etch enamel with hydrofluoric acid

41 QUESTION: What could the reason be if you see opaque white porcelain in the incisal 1/3 facial of the PFM crown:

Inadequate reduction of the

inciso facial part of the tooth

42 QUESTION: What is worse outcome of nonvital bleaching (internal bleach for endo)? internal root resorption /CERVICAL RESORPTION.

43 QUESTION: How long after vital tooth bleaching can you bond resin to it? 24 hours, 3 days, 1 week

44 QUESTION: Pontic of 3-unit FPD should rest gently on the soft tissue & should not blanch tissues.

45 QUESTION: Strength of abutment connection to pontic which is more important? occlusogingival width

46 QUESTION: Which represents position on the spectral wavelength? Hue

47 QUESTION: What kind of occlusion if in right lateral movement all posterior teeth are not in occlusion: canine guidance

48 QUESTION: Best imaging for TMD (soft tissue, disc & condyle of TMJ): MRI

49 QUESTION: What causes TMJ ankyloses? Trauma, Rheumatoid arthritis

50 QUESTION: Trismus includes what muscle? Medial pterygoid

51 QUESTION: What type of bond is composite on tooth structure?

a. chemical bond

b. mechanical bond (micromechanical)

c. organic coupling

d. adhesión

52 QUESTION: What do you place on a 75 y/o patient with ~ 8 class V carious lesions? GI

53 QUESTION: 65 y/o pt shows several new caries in molars and premolars class V, what material would you use:

a) amalgam

b) composites)

c) glass ionomer

54 QUESTION: Patient’s chief complaint is #8 and #9 don’t look right. Picture shows nothing is wrong with #9. #8 has extra

enamel at the incisaldistal

Page 53: Decembery RQS – NBDE II

Combined remembered Qs 2020 aspect. What do you do? Shave the inciso-distal aspect of #8.

55 QUESTION: Photo initiator of resin composite? Camphoroquinone

56 QUESTION: Etch removes the smear layer & exposed collagen fibers to form hybrid layer with resin

57 QUESTION: Which indicated for high caries risk or multiple class Vs? Glass Ionomer

58 QUESTION: What happens when you take an impression & lip immediately swells? Angioedema (allergy reaction)

59 QUESTION: Which is not recommended for final FPD cast impression?

• irreversible hydrocolloid

• reversible hydrocolloid

• PVS

• Polyether

60 QUESTION: Most rigid impression material: Polyether

61 QUESTION: What ion gets replaced in hydroxyapatite by fluoride? Hydroxyl

62 QUESTION: What’s the concentration of acidulated phosphate fluoride is used in the dental office? 1.23%

63 QUESTION: Which space is not involved/associated with Ludwig's angina?

Sublingual

Submandibular

Retropharyngeal

Submental

64 QUESTION: Turner’s tooth is caused by: trauma or local infection

65 QUESTION: HIV patient with oropharyngeal candidiasis, what would you prescribe? Fluconazole

66 QUESTION: Healthy 36-year-old, red patch on palate, redness in middle of tongue:

- Kaposi sarcoma

- Syphilis

- Median rhomboid glossitis à Candidiasis

- Gonorrhea

67 QUESTION: 85% of people have herpes

- 65-90% worldwide; 80-85% USA

68 QUESTION: Patient gets recurrent herpetic lesions very often with gingivostomatitis. What should be done? (herpetic

gingivostomatitis)

Acyclovir ( recurrent is by Acyclovir , you give acyclovir 3 days before the symptoms and palliative is during the

symptoms)

Palliative tx

Systemic antibiotics

Steroids

69 QUESTION: Lesion in lip with cauliflower shape: Papilloma

70 QUESTION: Which one resembles Epilus Fissuratum – Fibroma (both share trauma as etiology)

71 QUESTION: If you have leukoplakia for biopsy, do you incise or excise for biopsy? Incision

Page 54: Decembery RQS – NBDE II

Combined remembered Qs 2020 - incise multiple areas w incisional biopsy

72 QUESTION: Chewing Betel nut can lead to à SCC, xerostomia, gingival recession

73 QUESTION: Warthin tumor is most common in what gland? Parotid (don’t get mixed up with Wharton’s duct)

74 QUESTION: Which one can lead to ameloblastoma? Dentigerous Cyst

75 QUESTION: Radiographic picture: upside down molar with lucency around crown, what is it?

Dentigerous cyst

76 QUESTION: Amelogenesis imperfecta is autosomal dominant.

Dentiogenesis Imperfecta’’’’: Crowns are short & bulbous, narrow roots, obliterated Pulp

77 QUESTION: X-ray: Dentiogenesis Imperfecta – obliterated pulp chamber

78 QUESTION: All of the following are differential diagnosis for Dentinogensis imperfecta except?

ectodermal dysplasia

amelogenesis imperfectat

enamel dysplasia

dentinal dysplasia

enamel hypoplasia (AI)

79 QUESTION: Radiographs of a patient's teeth reveal that the crowns are bulbous; the pulps, obliterated; and the roots,

shortened. These

findings are associated with which of the following?

Porphyria

Pierre Robin syndrome

Amelogenesis imperfecta

Osteogenesis imperfecta

Erythroblastosis fetalis

80 QUESTION: 12 y/o boy’s X-ray shows roots are short & open apex. Sister also has same condition. What condition is

this?

DI - autosomal dominant

AI - autosomal recessive

Dentin dysplasia – autosomal dominant

81 QUESTION: Ectodermal dysplasia: which of the following is correct? It is X-linked, not autosomal dominant

82 QUESTION: Characteristic of Ectodermal Dysplasia is? Oligodontia (some missing teeth, > 6 teeth, not all teeth) and

hypohidrotic (reduced

sweating) or anhidrosis (lack of sweating)

83 QUESTION: Not a bone cyst? Nasolabial cyst b/c it occurs outside of bone & is a soft-tissue cyst

84 QUESTION: Most common location for mucocele? Lower lip

85 QUESTION: Sialoliths are most common in what gland? Submandibular gland & duct

Parulis = localized collection of pus in gingival soft tissue. Pus is produced as a result of necrosis of non-vital pulp

tissue or occlusion of a deep periodontal pocket.

Page 55: Decembery RQS – NBDE II

Combined remembered Qs 2020 86 QUESTION: Neurofibromatosis clinical presentations: Cafe au lait, lisch nodules of the iris

87 QUESTION: Dens in dente are most commonly seen in maxillary lateral incisor.

88 QUESTION: Sjogren’s syndrome: destruction of salivary and tear ductsà dry mouth

89 QUESTION: The mucosa of the hard palate is the usual intraoral site for which of the following conditions?

- Mucocele

- Sialolithiasis

- Minor aphthous ulcer

- Major aphthous ulcer

- Necrotizing sialometaplasia

90 QUESTION: What is primary source of radiation to the operator when taking x-rays?

radiation left in the air

scatter from the patient

scatter from the walls

leakage from the x-ray head

91 QUESTION: MRI uses what electromagnetic wave? RADIOWAVES

92 QUESTION: What does collimation do? reduces x-ray beam size/diameter & volume of irradiated tissue, reduces area

of exposure

- usually with circle diameter of 2.75 in

93 QUESTION: Collimation control of size & shape of x-ray beam

94 QUESTION: X-ray tube target metal is made out of: tungsten (target = tungsten/filter = aluminum)

95 QUESTION: Overlap on bitewings due to horizontal angulation

96 QUESTIO N: What does it look like on a pano when your patient moves during the pano? A vertical blur line vs horizontal defect.

97 QUESTION: If you have lesion of maxillary sinus, what kind of radiograph do you take? Waters

98 QUESTION: If change from 8 mm cone to 16 mm, how much exposure time do you need to increase by? 2, 4, 6, 8

- Remember that going from an 8 mm to 16 mm cone means the cone/target is LONGER. This is the PID (target to film

distance). If the PID

is increased there is LESS magnification. If the PID is shorter there is MORE magnification. Also density (darker x-ray)

increases when kA,

mA and exposure are increased.

99 QUESTION: What is most radio-resistant cell: Muscle

100 QUESTION: What is the mechanism of action of bisphosphonates? Inhibit osteoclasts

101 QUESTION: Pt taking bisphosphonates for 1 yr. IV, highest risk during dental tx? Osteonecrosis

Page 56: Decembery RQS – NBDE II

Combined remembered Qs 2020 102 QUESTION: Pt has sickle cell anemia & has a thrombolytic crisis, what could precipitate this?

a. Nitrous oxide / oxygen use

b. Cold

c. Trauma

d. Infection

- Sickle cell anemia is seen exclusively in black patients. Periods of unusual stress or of O2 deficiency (hypoxia) can

precipitate a sickle cell

crisis.

103 QUESTION: Pt is taking warfarin (Coumadin), what test do you run prior to extraction or surgery: INR (= 2.0-3.0)

104 QUESTION: Pt taking ginseng. Which med should be avoided?

• Penicillin

• Aspirin

• Digitoxin

104 QUESTION The drug contraindicated in pt taking gingko biloba: HEPARIN

105 QUESTION: Proposed modes of action for the oral antidiabetic agents include each of the following EXCEPT one.

Which one is the EXCEPTION?

A. Blockade of glucagon release from pancreas

B. Block?ade of catecholamine release from adrenal medulla

C. Stimulation of insulin release from pancreatic beta cells

D. Action as direct receptor agonists for the insulin receptor

E. Increase affinity of tissues for utilization of available plasma glucosa

106 QUESTION: Patient with orthopnea (shortness of breath-dyspnea-while lying flat), dyspnea, pedal edema

a. Emphysema

b. Pulmonary edema

c. COPD

d. Congestive heart failure

107 QUESTION: Pt has history of cardiovascular disease and now, pt is taking aspirin. Pt needs ext. What should dentist

do?

• Med consult with physician

• Normal extraction

• Stop aspirin 3 days before and 2 days after surgery

108 QUESTION: Side effect of nitroglycerin: orthostatic hypotension and headache

109 QUESTION: Epi and Nitroglycerine: antagonist

110 QUESTION: Child makes a wheezing sound before injection? Asthma (induced by stress)

111 QUESTION: Which of the following drugs is can trigger asthma?

a) narcotic analgesic

b) NSAID

c) corticosteroid

d) sympatolytic amine.

112 QUESTION: Patient begins to wheeze, what do you not do?

o Beta-2 blocker inhaler

Page 57: Decembery RQS – NBDE II

Combined remembered Qs 2020 o sit pt up & make them more comfortable

o corticosteroid inhaler

o Give oxygen

113 QUESTION: Pregnant in supine position, what gets too much pressure?

Fetus

Placenta

Inferior Vena Cava

Superior Vena Cava

114 QUESTION: Most common dental complication/emergency in office? Syncope

115 QUESTION: After receiving one cartridge of a local anesthetic, a healthy adult patient became unconscious in the

dental chair. The occurrence

of a brief convulsion is

A. pathognomonic of grand mal epilepsy.

B. consistent with a diagnosis of syncope.

C. usually caused by the epinephrine in the local anesthetic.

D. pathognomonic of intravascular injection of a local anesthetic.

Benzodiazepines: - MOA – modulate activity of inhibitory NT (GABA) receptor

Grand mal seizure: Phenytoin (Dilantin)

Status epilepticus: Valium (diazepam)

116 QUESTION: Which of these is indicated for grand mal seizure? DILANTIN (phenytoin)

117 QUESTION: Most common seizure in children – Febrile seizure

118 QUESTION: Which of the following drugs, when administered intravenously, is LEAST likely to produce respiratory

depression?

A. Fentanyl

B. Diazepam

C. Thiopental

D. Meperidine

E. Pentobarbital

119 QUESTION: What’s the action of the Benzodiazepines? Facilitates GABA receptor binding by Increasing the

frequency of chloride channel

opening.

120 QUESTION: Which drug best reverses the effect of benzodiazepines? Flumazenil

- Flumazenil: Benzodiazepine antagonist b/c competitive GABA receptor.

Anti-Histamines: competitive histamine receptor blockers

121 QUESTION: How does antihistamines work? Competitive inhibition of histamine receptors

122 QUESTION: Which of these opioid analgesics is associated with a serious life threatening drug interaction when

administered with an MAO

inhibitor?

Meperidine (Pethidine, Demerol)

Page 58: Decembery RQS – NBDE II

Combined remembered Qs 2020 morphine

fentanyl propoxyphene

codeine

- Can cause life-threatening hyperpyrexia reactions (fever)

123 QUESTION: Opioid usage shows all except: xerostomia, chronic cough, diarrhea, miosis, constipation

124 QUESTION: Which of the following symptoms is the most distinct characteristic of morphine poisoning?

A. Comatose sleep

B. Pin-point pupils (miosis)

C. Depressed respiration

D. Deep, rapid respiration

E. Widely dilated, non-responsive pupils

125 QUESTION: Naloxone: use for Opioid overdose. - Naloxone blocks or reverses the effects of opioid medication, including extreme drowsiness, slowed breathing, or loss of consciousness

126 QUESTION: Tylenol - can cause hepatotoxicity

127 QUESTION: Pt has hepatic dysfunction; which pain medication can prescribe?

a. Oxycodone

b. naproxen

c. acetaminophen

128 QUESTION: Which of the following does not have anti-inflammatory action? Acetaminophen

129 QUESTION: NSAIDs – mech of action of suppressing platelets – inactivate cyclooxygenase à decreased prostaglandin synthesis

130 QUESTION: After one effective dose of aspirin, how long must you wait before there is not effect on bleeding time? 1 week

131 QUESTION: Aspirin works on which pathway for pain? Cyclo-ox pathway

132 QUESTION: Biopsy - indicated when treatment doesn’t work after 14-20 days

- about 2 weeks—any red or white lesion that doesn’t resolve itself in two weeks

133 QUESTION: Oral candidiasis biopsy of choice is:

a. incisional biopsy

b. excisional biopsy

c. brush biopsy (collects the cells for cytological smear)

d. cytologic smear

134 QUESTION: White lesion is 2x3x2 cm, what type of biopsy?

excisional biopsy

incisional biopsy

smear

135 QUESTION: What kind of bacteria is under implants? At the apex of root canal? Gram (-) rods and filaments anaerobic

136 QUESTION: What is the least important factor when evaluating for implant?

Page 59: Decembery RQS – NBDE II

Combined remembered Qs 2020 concavity of mandible

bone density

distance to mandibular cancel

bone width

137 QUESTION: Minimum distance between adjacent implants? 3 mm

138 QUESTION: How much space between implant and tooth? Answers were 1.5 mm, 2, 3.5 3,

139 QUESTION: In anterior maxilla, for a 4mm diameter implant, how far apical to the CEJ of adjacent tooth for optimal

emergence profile?

1 mm above cej of adj tooth

1 mm below cej of adj tooth

2-4 mm below cej of adj tooth

140 QUESTION: Implants osteointergrate best in? posterior mandible

141 QUESTION: What causes the greatest incidence of implant failure?

Smoking

Osteoporosis with HTN

Hypotension

Allergy to antibiotics

142 QUESTION: Epithelial attachment for implant?

• Hemidesmosome* (epithelial attachment to tooth structure and implant are the same)

• fibronectin

143 QUESTION: What speed and torque for implant is used? High Torque, slow speed

144 QUESTION: How to clean implant- prophy cup, plastic scalers, not stainless steel!

145 QUESTION: You are considering the placement of an upper and lower implant-retained complete denture. How many

implants will you place in

the anterior region?

a. maxillary one and mandibular one

b. maxillary two and mandibular two

c. maxillary four and mandibular two

d. maxillary four and mandibular six

- If implant supported complete denture, add 2 more screws to each.

146 QUESTION: Which tooth is least likely to be missing?

Canine

Most commonly missing teeth are the 3rd molars, 2nd premolars and upper lateral incisors

147 QUESTION: Extraction of #30, which way do you section? Buccal- lingual

148 QUESTION: Where are you most likely to damage a nerve in vertical release of flap? lingual, Wharton’s duct and the

sublingual gland

- void vertical incisions in lingual and palatal

149 QUESTION: When doing flap surgery on mandible, what structure do you watch for? mental nerve, mentalis attachment

Page 60: Decembery RQS – NBDE II

Combined remembered Qs 2020 150 QUESTION: What number forceps to use when extracting mand premolars: 151 A

151 QUESTION: Treatment of alveolar osteitis: placement of a palliative medicament/dressing. DON’T USE antibiotic.

152 QUESTION: Lefort I fracture are associated with?

nasoethmoidal air cell

frontal sinus

maxillary sinus

mastoid air cell

153 QUESTION: A patient experiences numbness of the left upper lip, cheek, and the left side of the nose following a

fracture of his midface. This

symptom follows a fracture through the

A. nasal bone.

B. zygomatic arch.

C. maxillary sinus.

D. infraorbital rim.

154 QUESTION: Most common complication of sagittal osteotomy: IAN, loss of sensitivity

Red Complex – group of bacteria grouped together based on their association w/ periodontal disease

- red complex = P. gingivalis, Tannerella forsythia, treponema denticola

- BOP & deep pockets

Orange Complex - fusobacterium, prevotella, campylobacter

- Precedes red complex, plaque formation & maturation

155 QUESTION: What is the 1st step in bacterial plaque formation on a tooth? Pellicle formation (glycoproteins, enzymes,

proteins,

phosphoproteins).

- 2nd step is adhesion and attachment of bacteria

- 3rd step is colonialization and plaque maturation

156 QUESTION: Which one is not a periodontal risk factor? Smoking, oral hygiene, malnutrition, diabetic mellitus

157 QUESTION: What cytokine responsible for osteoclasts? IL-1, IL-8, IL-5, IL-3

158 QUESTION: If you have 1 mm recession and can probe 3 mm, how much attachment loss is there? 4mm. CAL= PPD +

recession

159 QUESTION: Best brushing technique to clean periodontal pockets:

A. Charters

B. Sulcular (another name for modified Bass)

C. Whitman’s

160 QUESTION: The role of chlorohexidine is cause: Substantivity (anti-plaque)

161 QUESTION: Best time for supportive periodontal therapy? 1, 3, 6, 9, months post s/rp

162 QUESTION: Initial tx for Localized aggressive periodontitis

Sc/RP

Antibiotics

Sc/RP and Antibiotics

Page 61: Decembery RQS – NBDE II

Combined remembered Qs 2020 Antibiotics for 1 week and then Sc/RP

163 QUESTION: First step in initiation treatment of HIV necrotizing ulcerative gingivitis? debridement and antibacterial

rinse, antibiotics

Gingivectomy

164 QUESTION: What’s the #1 cause of medication

induced gingival hyperplasia? Anti-convulsant

meds Dilantin (30% of all drug induced)

165 QUESTION: Indications for gingivectomy – hyperplastic gingiva & suprabony pockets

166 QUESTION: Gingivectomy is contraindicated with? Minimum attached gingiva

167 QUESTION: How does a gingivectomy heal? Secondary intention

168 QUESTION: Decalcified freeze dried bone allograft: has bone morphogenetic proteins (BMP). BMP is a protein complex responsible for initiating osteoinduction (ability of molecules contained in the graft to convert neighboring cells into osteoblasts)

169 QUESTION: What is indicated for the tx of unilateral posterior cross bite? Elastics from lingual of max mol to Buccal

of mand molar

- A single too th cross bite can be adjusted by placing cross elastics from maxillary lingual to mandibular buccal.

170 QUESTION: Calcification of premolar tooth at birth? NO ( premolar = starts at 2years), 1st molars = birth

171 QUESTION: In relation to their parent drug, conjugated metabolites are what? more ionized in plasma (more water soluble)

172 QUESTION: What best describes biotransformation? Increase in polarity, more ionized and more water soluble

- Whatever helps its excretion – polar and more water soluble

173 QUESTION: Epinephrine = physiological antagonist of histamine & nitroglycerin

- Doesn’t act on same mechanism (epi = α vasoconstriction vs nitro = smooth muscle dilatator) but opposing action

- Same mechanism = competive antagonist; physiological antagonist = competing physicological effects

174 QUESTION: What is bioavailability of a drug? amount of drug that is available in blood/plasma

175 QUESTION: Two different drugs with same dosages, bind to the same receptor, and cause same intrinsic affect.

However, they have different

affinities for the receptor. In which aspect these 2 drugs are similar?

a. ED50

b. LD50

c. Potency

d. Efficacy

- Efficacy bc they can both produce the same maximal response if enough is given

176 QUESTION: Main sign of dementia:

a. confusion

b. short term memory loss

c. long term memory loss

- short term memory loss = first main sign. Long term loss occurs later.

Page 62: Decembery RQS – NBDE II

Combined remembered Qs 2020 177 QUESTION: What catecholamine do tricyclic antidepressants affect? Dopamine, serotonin, acetylcholine

178 QUESTION: Know drugs used for conscious sedation à SSRIs/BDZ Diazepam and Prozac (fluoexitine

179 QUESTION: Why is nitrous oxide used on children? Alleviate anxiety

180 QUESTION: When is nitrous contraindicated? Asthma or COPD

181 QUESTION: What is not on cocaine overdose? pinpoint pupil

- Cocaine OD—mydriasis

- Opiate OD—pinpoint pupil

182 QUESTION: Indication for antibiotic prophylaxis: Prosthetic valve

183 QUESTION: (Patient’s medical tab say he is allergic to Amoxicillin), He needs to be premediated, what do you

prescribe? Clindamycin, 600mg 1hr

184 QUESTION: Pt w/ total knee replacement but was taking Amoxicillin for a while; how do you premeditate? NO (or

MED CONSULT)

185 QUESTION: Which med kills only anaerobic and parasites: metronidazole

186 QUESTION: Systemic antifungal: Fluconazole (diflucan)

187 QUESTION: Purpose of Major Connector

Stability and Rigidity

Stability and Retention

Retention and Rigidity

Rigidity and Esthetics

188 QUESTION: Main purpose of buccal flange of Mx denture? Stability

189 QUESTION: Which of the following best explains why the dentist should provide a postpalatal seal in a complete

maxillary denture? The seal will

compensate for:

A. errors in fabrication.

B. tissue displacement.

C. polymerization and cooling shrinkage.

D. deformation of the impression material.

190 QUESTION: Asked about what sound will determine VDO? S sound. This will bring teeth slightly together with 1-1.5

mm separation. This is the

“closest speaking space”

191 QUESTION: When you find VDO & the max tuberosity touches retromolar pad, what should you do?

• Make metal extension on mand RPD

• Surgery on max tuberosity

• Surgery on retromolar pad

• Open VDO

Page 63: Decembery RQS – NBDE II

Combined remembered Qs 2020

192 QUESTION: Patient feels fullness of upper lip after delivery of complete denture: Overextended labial flange

193 QUESTION: Pt wearing a complete dentures & is cheek biting: posterior teeth set up with no horizontal overlap.

194 QUESTION: Open mouth while maxillary border molding - Coronoid process will block buccal extensión

195 QUESTION: Posterior buccal extention of a mandibular complete denture is limited by:

Masseter muscle

196 QUESTION: Mand CD interfere with what muscle in lingual side? Mylohyoid.

197 QUESTION: You would relieve a mandibular denture in the area of the buccal frenum to allow which muscle to

function properly? Orbicularis

Oris

198 QUESTION: What is the best way to treat a tooth supported lower denture? Use metal copings to cover teeth

199 QUESTION: When does cleft lip and palate develop? 6-9 weeks in útero

200 QUESTION: What is true of patients with Down Syndrome/trisomy 21? Lower incidence of dental caries

201 QUESTION: What resembles epiphyseal plate? Synchondrosis

202 QUESTION: Sphenooccipital closure, what kind of tissue fills it in? Cartilage

203 QUESTION: Which of these undergo suture closure latest?

• sphenoethmoidal

• Sphenoccipital

• Intrasphenoid

• Intraoccipital

204 QUESTION: Cleidocrainal dysplasis – supernumerary teeth & problems with eruption

205 QUESTION: Which will give you very narrow facial structures and delayed eruption of permanent teeth?

• cleidocranial syndrome

• downs syndrome

206 QUESTION: LA with epinephrine contraindicated in? Uncontrolled Diabetes, hypothyroidism, hyperthyroidism

207 QUESTION: Tell patient that he needs to take of amalgam fillings bc they are not good for his health (hazardous): not

practicing veracity

(truthfulness)

208 QUESTION: Pt presents with amalgam restorations in good shape and the dentist suggest to change them for

composites due to systemic

toxicity of the amalgam. What ethic principle is the dentist is violating? Veracity

209 QUESTION: When should patient sign informed consent forms for surgery? AFTER there has been a discussion w/

the dentist about the

Surgery

210 QUESTION: Definition of rapport? mutual openness / harmonious relationship

Page 64: Decembery RQS – NBDE II

Combined remembered Qs 2020 - Rapport = mutual sense of trust and openness between individuals that, if neglected, compromises communication

Paraphrasing: repeating, in one’s own words, what someone has said. This serves to confirm one’s understanding,

validate a patient’s

feelings, convey interest in the patient’s experience (thereby building rapport), and highlight important points.

211 QUESTION: Pt. says, “I do not have time to quit smoking.” What stage is s/he in?

A: Precontemplation, contemplation, action, denial

212 QUESTION: How to reduce stress & dental anxiety? Tell-show-do

213 QUESTION: Autistic kids have what characteristic? Repetitive behavior

214 QUESTION: if you find problems with a medical conditions occurring with a certain drug, who do you contact? OSHA,

FDA, EPA

215 QUESTION: If there is an adverse reaction to a medication in the office, who do you notify? a) FDA b) CDC c) HIPPA d) OSHA e) EPA.

216 QUESTION: What is it called when a dentist charges several procedures instead of one?

a. upcoding

b. downcoding

c. unbundling

d. bundling

Feb 10- 11 -child with bruise on his belly Tell assistant to call child abuse Or call after appointment -Ear lobe on panoramic -Pterygomaxillary fissure on pano -Kid felt from bicycle has Headache & nausea what u Do ? -what's the most important when choose the tooth shade ? I made value Info: Hue is the color Value darkness & lightness Chroma saturation of color -Also i had the question about who is responsible for hygienist mistake!?

Page 65: Decembery RQS – NBDE II

Combined remembered Qs 2020

All 3 dentists and hyginest -Like wt kind of force applied to a tooth with vertical bone loss: A/Heavy w great moment B/Heavy w light moment C/Light w great moment D/Light w light moment? I choose this but not sure -translation movement, the force must be applied through :

A- center of resistance B- Center of rotation C-Root D-Crow -What is component in algintae that react with calcium n give the desored working time.

Trisodium phosphate. -Most of hypertension pt show little complaince of the medication why Because It cause : A-Baldness B-Or sexual dysfunction -How to handle asthmatic patient Position & medicine & precautions -ADHD pt 9 years dental management With previous bad experience: A-Discus with pt B-Discus with parent C-Don't discus while extraction of molar Fractured pieces of bone Wht to do? A/Remove all fractured pieces B/remove all except pieces attched to periosteum -Lingual inclines of baccal cusp of mand molars: working Non working -3 Questions on the sequence of treatment

Page 66: Decembery RQS – NBDE II

Combined remembered Qs 2020

Memorize PROSP -Most recurrent infection with HIV: bacterial

fungal _what's the eye dysfunction called if someone has eye problem in which If he is looking at moving object And both eyes moves but at the midline one eyes stop moving and the other keep going?? A/ I can't remember the options؟ All option end with gmus and only recall nystagmus? When the pt all time nagging and dr got angry and shout on assistant when she dropped the instrument what is that? .patietn is scheduled to extract multiple teeth, best time to do alvioloplasty? -Same day of the extraction If vertical dimension of rest is 66, what is VDO? - 63(should be 2-3 mm less) what is not important in the first visit for patient needs cleaning? -update medical history -X-rays -periodontal charting -Diagnostic casts? Kenddy class III main retention? -direct clasp -indirect clasp -rests requirement for major connector?

Page 67: Decembery RQS – NBDE II

Combined remembered Qs 2020 -Support -retention -stabilization Rigidity and stability injury to which nerve/artery could lead to blindness? Temporal main sign of hypoxia ? -Tachycardia -blue finger tips after you give a block of LA to a kid he feels agitated, main reason? -injection in the artery main reason when doing CPR, stomach inflation ? compressing with too much force? main reason for failed CPR? obstruction air way The space difference between primary canine, first & second molar and the succedaneous teeth ? freeway space primate spacing leeway space -pathology, pt with pain in neck and when swallow and when turn neck to lateral side A- eagle syndrome -condyle inclination is used for? Protrusive record

Page 68: Decembery RQS – NBDE II

Combined remembered Qs 2020 -which X-ray technique shows the floor of the maxillary sinus imposing on the roots of maxillary? -bisected angle technique periapica - DETERMINE TONGE in pan - DETERMINE Airway in pan -What not cause nephrotoxcicity - 2 Q like if dentist Extract all teeth and do complete denture under which code of ethic -Central incisors looks wide to shape it what to do : A- put 2 Lines in middle of crown B -know the exact definition of granuloma and absces and cyst and which one of them contain macrophage and lymphocyte ? -Unrestorable tooth in pt take Biophosphante what to do: Extract Rct -day 2 I had 2 children ortho cases 2 cases on complete denture 3 caseson RPD Few patho Many Pt management on both days Over all I like day 2 than day 1 and exam is durable just focus in high yield fact they like pt management then prostho and operative then pharma and lastly period patholog Kamalijot RQ Carcinoma in situ- erythroplakia White sponge nevus Lichen planus Safer drug in hiv- Erythromycin Amoxicillin Ibuprofen Class 3,4,5 composite bevel 45 Not a method to check sterilization-

Page 69: Decembery RQS – NBDE II

Combined remembered Qs 2020 Electrical Buccal roots max molar more mesial to palatal, position of tube- mesial distal inferior superior Cells on tension side- osteoblasts pMn Kid after active ortho TX, Retention by- (history of poor oral hygiene) Fixed removable Other 2 options Perio disease more in- Black male Max sinus seems more close to max teeth in- Periapica Best way to control caries- Flouride varnish In IV for post op pain relief we Ketorolac DKA- Hypoglycaemia Hyperglycaemia 2 other options So many OP ques Periapical cementoblastoma Ossesous dysplasia Cemento* Gingivectomy indicaitons Lesion not on tongue- Peripheral granuloma Ectopic thyroid 2 more i dont remembr Lots of ques on finish lines, Axio gingival axio pulpal, grooves-blah blah I hope you got what i mean Hemorrhagic shock first sign- tachycardia hypotension other options Epinephrin works on- Aloha 1 Alpha 2 Beta 1 Beta 2 Answer: All DKA- 2,3 ques on this topic Ethics basic questions All staff once in yr should go for checking of- TB Osteoporosis radiographically- Thin trabecular Less osteoid calcification Other 2 options

Page 70: Decembery RQS – NBDE II

Combined remembered Qs 2020 Palatial root more mesial to buccal root, position of tube- Mesial Distal Superior Inferior Swelling of lip Mucus retentions cysts- Minor SG something Sialolith Swelling of lip after anesthesia injection- Angiodema Nitrous oxide and epinephrin antagonise by- Physiologic Competitive Other 2 options

NBDE PREPARE RQ nbde part 2 rqs 1.Pic(o) o in pico?? Outcome.

2.Multiple myeloma initially what sign do v see?? Bone pain !!

3.cold lingering,cold non lingering.

4.blue nails wer ?? Like in asthma or sarcoidosis or wer??

5.many questions on cross sectional study.

6.super numerary teeth n multiple osteomas .wat do u investigate?? In options intestinal

polyposis is der.its a twisted question I think for gardener’s syndrome.

7. Divergent pupal to gingival is the answer dnt remember question dnt knw if this is correct

ans also.

8.many questions from operative n perio flaps especially modified widman flap incision like

apical to mucogingival? It goes like dat. wer do u give n y do u give??regarding pocket

elimination or y ?? Very little prostho n very little pharma.

9.many mango almost all n Rita.

10.radiograph radiolucent area wid radioopaque foci.

11.3 yr kid 16 kg hw much lidocaine??

12.tooth n tissue born appliance.

13.10 MA exposure 1 gy .wat is the exposure for 0.5 gy if the density is same for both.i don’t

remember.

14.t or f question vertical incision on mid root area....and vertical incision to cover roots or to

cover recession it goes like dat..

15.dentigerous cyst radiograph.

Page 71: Decembery RQS – NBDE II

Combined remembered Qs 2020 16.cellulitis Wat do u see??neutropenia,lymphocytisis are options.

17.folic acid something in which cancer drug??sulfonamide block folic acid synthesis by

competing with PABA

. 18.bur with many flutes.

19.which microorganism in periodontitis without necrosis dint rem question .options were like

filamentous rods,spirichetes

20.plaque hypothesis all are related to plaque hypo except

1. Options I forgot Day2. New drug names are given like Rouvastatin,bupiropreone I think it

causes (xerostomia) check it ,lisinopril,ameloperene I don’t

remember.,cevimelin,chantix,nifedipine these were mentioned in patient history question is

like which causes xerostomia?options were rouvastatin+bupiropion ,lisinopril+rouva n so on.

2.18 yr old case with narrow arch n supraerupted canines a question is while performing

extractions he loses consciousness heart rate 60 n bp is also very less like 70/80 or

something wats the cause hyperventilation or vaso vagal syncope n other options.

3.extraction to a patient which options are correct

1.deep anesthesia is more affective by intravenous injection

2.Nitrous oxide inhalation causes amnesia

3.oral sedation has better titration .i don’t remember.

4.periapical cemento osseous dysplasia radiograph.

5.hyoid bone radiograph.

6.radiolucency between mandibular central incisors no signs or symptoms.answer is

menta fossa

MARCH ● Kv=onset ● Supernumerary=initiation ● Angina/MI how to differentiate duratin ● Increase caries= root

Page 72: Decembery RQS – NBDE II

Combined remembered Qs 2020

● Differentiate root caries= soft ● Recent studies prove periodontal disease related with=oral cancer (??)cvs ● Lingual inclination of lower buccal with buccal inclination of upper lingual

when?=non-working ● When we split procedures make it multiple is this unbundling? ● when the insurance decide to pay for crown not for endo is this bundling ● Positive Nikoliski sign but there was no pemphigus or pemphigoid...all weird options i

picked hyper sensitivity reaction TRUE. Steven Johnson has + nikolsky ● Pt on anticoagulants with no more details...u want to do simple implant an ok INR

would be 2.5...3.5 (3.5 is minimum for surgery) ● Pt is alcoholic and u want do surgery. What to check? Cbc..pt time. Something likes

fat. Or glucose level ● Diabetic in day of procedure. Liquids with half dose of insulin? ● Best prognosis? 3 walls ● Best furcation II? GTR ● Healing after perio flap surgery? I picked long junctional epi..othe option was by

formation of new bone, ligaments and ct ● Root amputation best prognosis: coronally and the roots are diverge...coronally and

the roots are converge...after my brain got freeze and start not differentiating between diverge and converge i ended up with convergep

● Pt came 3 days after ext with 6mm antrum communication....i chosed cose with gold foil stent ...there was flaps

Decks: Sinus communication abx amoxicillin Opening 2-6mm figure 8 suture Opening more than 7 Closed with flap procedure

● first 24 hours free gingival graft get from host nutrients from underlying connective tissue.

● attached gingiva thickest in lateral maxilla thinnest lower first premolar T ● most congenital missing was 2premolar lower ● lateral lower most to have 2 canals? i choosed T ● Study between two groups’ boys and girls? Chi square ● Null hypothesis question? Group A is equal to B (There is no difference is null hypo) ● New invention drug experiments? Random clinical try ● most associated with vital tooth i picked middle palatal cyst ● Man molars get space by? i picked apposition of ant ramous . Resorption of ant border

Page 73: Decembery RQS – NBDE II

Combined remembered Qs 2020

● Surgical guidance for implant for what? I picked to lead for location of implant. Others where to know the length of the implant

● U put palatal splint why? I picked to avoid contamination with fluid...other was to protect bone and ct from trauma, to prevent hematoma and support the flap.

● Tooth gets its color from? Enamel, dentin, cementum? i picked dentin ● u make bulk PFM why? bec the opaque dentin layer was thick so u fix the color? i

picked this other was the dentin porcelain layer was thin ● minimal accepted crown root ratio 1:1 true this is minimum its in decks ● best crown for posterior zirconium ● phenothiazine =extrapyramidal ● Why u prefer zirconium in posterior? long longevity ● Anterior cross-bite in child with central only what to do? Refer to ortho or fix it

immediately? i picked immediately ● pt with gold inlay come after 3 hours with shooting pain why? galvanic shock or

hyperocclusion....i don’t remember which one i ended up with but i think galvanic (DEPENDS IF THEY SAY IF DIFFERENT METAL IS OCCLUDING)

● 45 yrs with bilateral post cross bite how to fix? i picked surgery ● ortho tooth with vertical bone resorption what to expect...i chosed less force...less time ● ? i picked to restrict the x-ray beam ● What is illegal for dentist? i picked to prescribe schedule 2 for back pain not wine, not

sexual relation not advertise with price ● not ADA code ...i picked Credential needs to be a dentist (licensure) ● what is not in concent...i picked price ● distance when to discuss price with pat i pick 3-6 feet...there was 8-11 feets ● pt have pain in one side of face this pain worse when moving his face what is most

probably? i picked unilateral cavernous sinus thrombosis...other was sinus but the said when move the head not lay down or bent

● Collimination restricts the size and shape of xray beam reducing patient exposure ● Can mump be bilateral? YES ● pt come with swelling upper lip happen bc yesterday had treatment why? i pick

haemangioma ( probably angioedema) ● Tooth with lingual anatomy cause calculus to stick? i pick upper central or upper

lateral can be if in options if central it’s spcingulum if lateral it’s gongivoocclusal groove ● Why not to take color while put rubber dam? i pick rubber dam make tooth look lighter ● How to diagnose vertical roots fracture? 1-ray 2- visual 3- use the probe and look for

pocket… X-ray is the only way to diagnosis for any root fracture radiolucent halo seen. ● DMF...Filled tooth is with most? White ● u broke the ridge what to do? i pick u put lingual bar and make sure its stable

Page 74: Decembery RQS – NBDE II

Combined remembered Qs 2020

● What protection for root caries? Home stannous fluorideor home neutral sodium fluoride?

Decks say stannous is best for root caries ● What fluoride does? I put decrease enamel solubility ● How saliva help in reminerlization/demineralization? Options like saliva has calcium,

other saliva has ions, I picked the option says saliva is there 24/7 and it help in demineralization/re-mineralization

● What adhesive do? All options about removing smear layer i put it don’t remove ● All improve crown retention except? I put decrease convergence of preparation. Other

option was shoulder All around ● Elastic for posterior molar? Put on lingual upper and buccal lower ● Upper central intruded 6mm what to do? I put leave it ● Sealant close fissures by? I put micromechanical ● Sealant....stop caries if there is small in deep fissure or fossae i put true ● If animal were feeding through stomach feed what happen? Caries reduce ● Type to allergic II reaction? I closed RH blood transfer from mother to baby ● In asthma attack? Put pt on upright position ● What first thing to do after u gives shock to pt with cardiac arrest? I put check pulse ● Pterygomandibular raphe...i pick junction of buccinator and superior constrictor ● Ian block only tongue numb no lips? Re do the block incisive block ● U give 1 carpule of lido to kid and he start being super active and agitated why? I pick

intravenous administration. Others were allergic to lido..allergic to epi ● What to make sure to do in pedo anesthesia? I pick aspirate and administer slowly ● What make intracranial anesthesia work? Pressure anesthesia ● Dentist was nice and professional with pt then after appt he become mad on his

assistant for dropping instruments? I pick dissociation ( Displacement ) You had a very difficult pediatric patient who was crying a lot. You remain calm and

professional the whole time. When the patient leaves, your assistant accidentally drops an instrument on the floor and you get extremely angry at her and yell at her. This type of defense mechanism is known as what? Dissociation Displacement Desensitization

Page 75: Decembery RQS – NBDE II

Combined remembered Qs 2020

● Dentist dont do treatment on pt..didn’t contact him...i put neglecting ● Xerostomia dont do..i pick teeth attrition

All of the following cause xerostomia except? a. caries, b. candidiasis c. dental attrition

● Refuge pt don’t speak English has severe pain and she believes bad spirits behind

this. What is the most concern of the dentist will be? Language? Consent form from her daughter? Her believe? Or removing the pain? I picked removing the pain

● Refuge pt dont speak English but her daughter does. What best to do? Ask daughter to translate? Ask office manager to translate? Call healthcare interpreting phone service? I picked the phone service since they should be professional and know the terminology

● What is not obligated by HIPPA? I pick dentist send bi-annual report to DHSS ● What annual screening mandated for all healthcare workers? I pit TB test ● What bacteria in pocket? I picked spirochetes??? ● What primary disinfectant should do? I picked kill tb (inject according to Trump) ● X-ray of head when u see it its typical picture of moon surface with two or three empty

circles. Border is fine? I picked osteoporosis ● Osteoradio necrosis pt with pain what to do? Endo and crenectomy ● Pt with red bleeding gingiva and with something in the extremities ...no picture just

description i picked acute leukemia. I don’t remember if the age was young but mostly it was young

● Less hair? Hypothyroidism (it suppose to be thin hair if less ectodermal) ● They asked me about the eyes are not in the same direction what is this? No diplopia

in options STRABISMUS ● How to see vertical resporption and diagnose heavy caries for endo ? Options where

cbct, occlusal, pano and mesial shift x ray? I picked mesial shift ● What used to diagnose perio bone level? I pick pano…options where bitewing,

periodical’s, cbct ● Eagle syndrome..pano with calcified stylo ● Mixed dentition help dentist to? I pick know the dental age ● One big central Mesiodistally with two pulp chambers is an example of? Fusion ● Diastema closure is an example of? External something and i put intraoral something

(the something is like splint. Or approached don’t remember the word)- BONDED BRACKET WITH INTER TOOTH TRACTION

● Posterior landmark of upper denture landmark is? I put palatal vault

Page 76: Decembery RQS – NBDE II

Combined remembered Qs 2020

● Most reason to replace anterior composite? I put color change ● Major connector? Rigidity and stability ● Premolar is moral and under it is a white radiopaque big circle? I pick idiopathic

osteosclerosis ● Cyst around canine? AOT ● Implant platform is? 2-4 apical to adjacent gingival margin tooth ...all other options

where apical ● Implant 3-4 from adjacent cej ● Cavernous sinus thrombosis? Swelling in canine fossa region ● 3rd molar disappeared? Take x ray to locate options where open from canine fossa

and do the procedure ● MTA stimulate? INDUCES HARD TISSUE FORMATION ● Neuroparaxia...dint say temporary or motor. Just pick the opn say cut in axon but not

in neurostium ● What big nerve effect on nerve order? I said no change other were increase, decrease

inhibit ● Drug A is more effective than B what means? I pick drug A can cause stronger effect

higher potency ● Diagram of three drugs asking which one is more effective I just pick the drug with the

high line in the diagram ● Color stability=TEGDMA( causes low color stability ) only UDEMA make color stable ● Pt class three skeletal what decrease with age? ANB ● Impression not for fixed prosthesis=irreversible hydrocolloid ● Effect polymerization=eugenol ● Cleft palate does not cause? Change in teeth shape ● You saw a pt a year ago he was taking nitroglycerine for angina once per week when

he do exercise. Today you saw him with a permanent patch of nitroglycerine that he take daily 3-4 times…what is his ASA I pick 4

● Dry mouth=anticholinergic ● Overhanged filling wedge was the problem ● 3 months baby=no Fluoride supplement ● At birth teeth calcification= the only option with permanent first molar said: permanent

first molar and all primary teeth ● Ectodermal displace= oligodontia ● Agenesis= canine least ● Sialosis= submandibular (Wharton’s duct) ● How to make tooth looks narrower? I pick to bring the lines together and the

embrasure look shallower

Page 77: Decembery RQS – NBDE II

Combined remembered Qs 2020

● Least one to get NO2 in the clinic? Patient ● Effect of radiation on the body which molecule? H2O ● Distal rest? To resist vertical force ● Not present in class V=incisal wall

(axial/pulpal wall is present) facial wall isn’t) ● Tooth was discolored Q what to expect this tooth had before? Trauma ● Serial extraction=CD4 ● Tooth painful on hot relieved by cold=irreversible pulpitis ● Longer treatment plan less compliance of patient? Both true ● Periodontal disease due to intrinsic causes. Intrinsic or systemic factors play role in

progress of disease. First false second true ● Picture of probe 7 mm pocket depth +2 recession total=9 ● Secondary occlusal trauma ● Cherubsim ● Chronic apical abscess = sinus tract ● Airspace in pano right side ● Zygomatic bone in pano left side ● One pano with crest in the middle of centrals I put nose curve ● Two upper and lower no horizontal overlap / reduce buccal to avoid cheack bite ● Increase VOD and decrease intercuspal = patient denture has no retention, chewing

while talking

Day 2

1-cryer elivator...ext lower molars roots

2-Tooth#3 has filling it’s not big normal one, tooth is asymptomatic, pt dont complain, u

checked and its non-vital what to do? No treatment

3-Pt live in Fluoridated area. His teeth are all clean like manikin teeth. The question what

preventive treatment on his lower molar? Options sealant, varnish, composite, no

treatment...i pick no tx

4-Pt has 9 teeth upper arch all of them are good. Pt ask to extract them all...what is the least

u should do? Complete denture? I choose this

5-If u did what he asked for what code of ethics u break? Non maleficience and autonomy

Page 78: Decembery RQS – NBDE II

Combined remembered Qs 2020 6-The patient asking for full extraction is related (don’t remember the exact word but means

is under which right as the pt think) to which code of ethics? Autonomy and beneficence

7-Pt with poor oral hygiene 100% plaque...has diabetes controlled...asked me what the

contraindication for putting implants is? I picked plaque

8-Pt with missing 14-15...there is opposing...and missing 30 and there is opposing...the

question which improve the occlusion? One option is to put partial 2 units 14-15 and implant

30

9-ADHD pt

Q he start becoming overacting and aggressive during the procedure...talk to his parents

privately about what is wrong...talk to the assistant about this issue...cancel appt..talk to the

child directly to know what bothers him? I picked D

10-What make his gum looks like this? What i saw was red, dry, little bit like hyperplasia...I

picked his medication

11-Does his medication do? Xerostomia

12-Class II Kennedy

13-Fulcrum class II

Pt 16 years with pano he has retained uper right 1st primary molar and 1st-2nd left molars

...no permanent premolars clinically or in xray..his upper teeth from canine to canine are all

scattered in distal directions and spacing ..diastema...his teeth, gum are healthy

14-when to do frenectomy? I pick before closing the diastema before frenectomy

● You wait until canines erupt, then ortho, then frenectomy

15-what to ask the pt? Caries history, systemic conditions, family missing teeth history...C

16-what angles classification? The picture shows the canines both sides and both were class

III

17- What is the pt profile! I pick strait because. The centrals where in class one relationship

with lowers normal over jet -over bite in cephalometric x ray

It was the upper canines inclined and spaced distally due to missing teeth

18- Why the primary teeth still there? I put ankylosed

Page 79: Decembery RQS – NBDE II

Combined remembered Qs 2020

19-Don’t remember the question but they want to pick which medication is not good for the pt

Ibuprofen

Tynolol 3

Acetaminophen +hydrocodone

Oxycodone

I picked ibuprofen since i believe all other 3 are related

20-Asthmatic pt become anxious what to do? I put nitros oxide

21-Bipolar...lithium

22-Pt become getting stress and anxiety what preoperative med? 2 MG valium

Other was Prozac (we are not physicians)

23-Pt with 9-8 want the incisal edge to be esthetics... the #9 is perfect the #8 has a very

small chip in the incisal edge like the size of the probe tip..all options where over treatment

and include #9 i pick correct #8 with a bur

24- Is lingual artery entering the mandibular canal? No

25-Class II furcation? GTR

26-Tooth 2 no opposing tooth was in x ray looks super erupted..q. Does it need re

arrangements for occlusion if we put lower implant? I pick yes

27-Pt need upper denture there is a big elevation not painful pt even didn’t know it’s there

what is this? I pick exostosis

28-What best to diagnose? I pick incisional biopsy

29-U extracts a tooth and under u hyper plastic vascularized ct what is this? Granulated

tissue

30-U extract tissue found radicular lesion encircled by epithelial tissue? Cyst

31-Small pink spot under the lower incisor? Parylis, fistula, Fordyce granule fibroma? I pick

fibroma

Pt with #8 very slightly prepped facially and no cover. It’s the only tooth has edge to edge

with lower central...2 questions

32-what was the restoration? Veneer

Page 80: Decembery RQS – NBDE II

Combined remembered Qs 2020 33-why it fails? Occlusion

34- pt move to sit upright and he became dizzy what to do? Give oxygen, give glucose, take

vital, sit in semi-supine position? D

35-why the pt had these symptoms? orthosthatic hypotension

36- Diabetic pt unconscious? Administer glucose

37-line within cervical area of molar? External oblique ridge

38- Treatment sequence for bad perio pt with problems perio – ortho - prostho

39- Treatment sequence for pt with primary retained teeth with spacing and

need...Extraction-ortho-prostho

40- Deep caries? Remove infected, leave affected if two close

41-Class 5 lower premolar what filling?

GI

42-What is the difficulty? Isolation

43-Pt drink energy drinks 3-5 times a day. While he was walking to clinic he was drinking

also? What is his mouth PH once he sits on your chair? 5.6 - 6.5 - 7.5 - 8?

44-Who build Dentin Bridge? Calcium hydroxide

45- Ferule effect? Prevent root fracture

46-Tooth 7 with big post

47-Tooth 8 with short post

48-Tooth 8 with surgical retreatment

49-What surgical guide don’t decide?

Number of implants

Location of implants

Size

Angulation

50-Bridge bulbous

51-Extraction done before the picture show irregular tissue and l...i pick loss of buccal cortical

bone during the previous extract

Page 81: Decembery RQS – NBDE II

Combined remembered Qs 2020 52-Which tooth need endo retreat? #7

53-Which bridge to cover? All options were for missing teeth except 1

54-Pt on smoke session what not to give? Smokeless tobacco

55-Internal resorption primary EXT

56-What is not count if u decide to extract this primary? Furcation involvement (because

tooth has disto caries)

56- Anxiety pt preoperative med? 2 mg valium

57-lithium...For bipolar

58- Lateral tooth picture with mamelons..Question what are these? Normal development

Shamili Sam’s Questions 1. Best area to place an implant ant mand 2. Best area for osseointegration- post mandible 3. What determines the color coefficient of composites hue 4. Pt wants to Extract all his teeth and want a CD, even though all his teeth are good. What ethical principles clashes with this autonomy and non maleficence A. Autonomy & Justice B. Autonomy & beneficence C. Non malfeasance & Beneficence 5. Characteristic of cherubism

● Most cases in mandible ● Bilateral expansion of jaws ● Tumors stop growing after puberty ● Histologically resembles central giant cell granuloma

6. Which is seen least in elders A. Drug abuse is mainly alcohol B. Drug abuse with illegal drugs mostly C. More prone to over use of drugs than misuse D. Least likely to miss use the drugs 7. Radiographs on A. Multiple myeloma B. Pterygomaxillary fissure

Page 82: Decembery RQS – NBDE II

Combined remembered Qs 2020 C. Ear lobe D. Air space E. Dentegerous cyst F. Taurodontism G. Ext oblique ridge H. Short crown and roots but pulp is not obliterated and no Periapica radiolucencies also - I chose AI I. Eagles syndrome ( pt is unable to open mouth and pain) 9. Pictures on A. Primary HGS B. Papilloma C. Squamous papilloma D. Recurrent herpes E. Amalgam tattoo (Rct with retrograde filling in 7. What is the pigmentation in the vestibule of tooth 7) F. Pyogenic granuloma or exostosis (confused b/t these two) 10. Dentist writes a prescription to the pharmacy for a 60 year old and mentions not to give child proof lid, this is because A. Social barrier (Options were weird, don’t remember) 11. Pt is irritating the dentist and meanwhile assistant drops an instrument. Dentist shows his anger on her. This is called (the answer given in the file is not in the options) 12 pt has cl 3, what keeps decreasing as he grows A. ANB B. SNA C. SNB D. Point B 13. Point A in ceph is from most concave point of anterior maxilla The “most posterior” point from ANS to supradentale with the head in Natural Head Posture. 14. Pt has anxiety, what do you give him for this in next appointment to keep him calm - Diazepam 15. All of the following frugs MOA is by effecting the ion transport, except A. Proprenolol B. Lido C. Ca blockers

Page 83: Decembery RQS – NBDE II

Combined remembered Qs 2020 16. Lingual inclines of buccal cusp of mand molar interferes in lateral movement, which is side interference is this A. Working B. Non working C. Protrusive 17. Fracture of condyle, jaw deviates to A. Anterior, medial.✔ B. Superior, medial C. Posterior, lateral D. Anterior, lateral 18. Purpose of face bow transfer- retain 19. Keratinic lesion in which location most likely to turn into dysplasia A. Buccal mucosa B. Floor of the mouth ( think of carcinoma in situ) C. Tongue 20. Pic of bilateral edentulous jaw (day 2) A. What kennedy classification? Class 1 B. Where to place rest: mesial of last abutment tooth always C. Fulcrum passes between which tooth 21. Pt has wheezing with aspirin, best alternative is Nsaids are contraindicated in asthma. Give tylenol 22. Special characteristic of codon-unambiguity, redundant and universal 23. Increased dose of Acetaminophen causesHepatotoxicity 24. What bacteria seen at root apex of necrotic tooth: ANAEROBIC A. Facultative anerobes B. Aerobes C. Motile 25. Improperly drained puss from perio pocket leads to A. Abscess B. Cyst C. Granuloma

Page 84: Decembery RQS – NBDE II

Combined remembered Qs 2020 26. Most recurrent oral cyst OKC 27. Which of the following is assosciated with Basal cell nevus OKC, gorlin syndrome 28. Soft tissue swelling in the mucobuccal fold lateral to root of lateral incisor - nasolabial cyst 29. Why tetra and penicillin not given together static and cidal. They cancel eacother out 30. Advantages of GIC fluoride, bond 31. Implant distance from tooth 1.5 32. Implant should be placed where in relation to the CEJ of adjacent tooth 2-4 mm from cej 33. 1ppm of fluoride in 1L water. How much in mg 1 mg 34. Handicapped condition associated with brain damage and causes neuromuscular dysfunction A. Trigaminal neuralgia B. Bells palsy C. Cerebral palsy 35. Medication of Trigaminal neuralgia Carbamazepine 36. Function of major connector in FPD stability and rigidity 37. How should a Pontiac be designed not blanch 38. Vertical incision should be placed in mid facial area of tooth ( true or false kinda question) Vertical incision is used to raise the flap 39. Space for mand molars is created by 40. Post extension of mand denture limited by Retromolar pad is the distal termination of mand denture (which is post to start of rise of ramus) 41. Pendulated maxillary tuberosity, touching the opposing counter part. What do you do A. Osteotomy B. Ostectomy C. Bone file 42. East west cryer used for mand molar roots 43. Flabby anterior maxillary area, dentist uses which imp material zinc oxide eugenol paste 44. Questions of diagnosis and treatment of Reversible and irreversible pulpitis 45. Questions on apical periodontitis 46. Questions on apexification and apexogenesis

Page 85: Decembery RQS – NBDE II

Combined remembered Qs 2020 47. Black pigmented area over 7, best diagnosed by (pic related) A. Incisional biopsy B. Excisional biopsy C. Cytology 48. Change in hair texture is associated with which gland disfunction - Thyroid 49. After giving nitroglycerin to stroke pr, how will you you differentiate if it is MI or Angina A. Intensity of pain B. Location of pain C. Characteristic of pain SHOULD BE DURATION OF PAIN 50. Pt comes with pain, tells he is unable to sleep last night as pain was waking him up. Treatment is A. RCT B. Antibiotics C. I and D of root canals 51. Question on calculating attachment loss 52. Question on wavelength in shade selection –HUE 53. Bonding of pit and fissure sealants with tooth by micromechanical retention 54. Dentist conducted a study following a new research. But it gave more false negative. even diseased people are shown as non diseased. Which factor is affected A. Sensitivity B. Reliability Specificity non diseased Sensitivity diseased 55. Most common reason to redo anterior composites is A. Stain/ color change B. Chip 56. Which position to place the pt during syncope tredelenburg. Supine with feet up. 57. Asthma pt, which position you make him sit during the middle of the treatment A. Straight with slightly leaning forward B. Supine C. Left supine D. Straight with nasal catheter with o2 58. First thing dentist should look for if a pt goes unconscious in the middle of treatment A. Carotid pulse B. BP

Page 86: Decembery RQS – NBDE II

Combined remembered Qs 2020 C. Glucose levels D. HR 59. Hyperglycemic pt faints 1 hr after the treatment started, what do you do TXT hypoglycemia: page 143

● Conscious oral carbohydrate OJ, table sugar, cola, candy ● Unconcious EMS should be contacted then 1mg of glucagon injected IM. Should

restore pt within 15 min.

60. What age cleft palate occurs Cleft lip 4-5 weeks Cleft palate 8-9 weeks 61. Pt has depression and is on antidepressants, which of the following drugs causes xerostomia. TCA is the strongest anticolinergic 62. Antagonist to midazolam flumazenil 63. Antagonist to meperidine naloxone 64. Sequence of treatment planning after perio A. Endo,ortho,prostho B. Prostho, endo, ortho C. Ortho, prostho, endo EPOSOP 65. Signs of narcotic over dose miosos 66. All of the follow cause bilateral parotid gland swelling, except (I got 2-3 questions on bilateral and unilateral swelling of PG🤦🏻 ♀) A. Mumps B. Warthins C. Pleomorphic adenoma D. Sialolithiasis Anorexia causes bilateral swelling African,5 years boy complaining of bilateral facial swelling, x.ray showed multilocular radiolucenency,cause roots resorption with (Starry Sky appearance): A- Burkitte`s lymphoma

Page 87: Decembery RQS – NBDE II

Combined remembered Qs 2020 B- cherubism C- fibrous dysplasia The most common cause of bilateral swelling of the parotid glands in children is A. Mikulicz' disease. B. mumps or acute infectious parotitis. C. mixed salivary tumor (pleomorphic adenoma). D. sialolithiasis. Bilateral symmetrical swelling of the mandible of a child is likely to be caused by A. Acromegaly B. Paget’s disease C. Giant cell lesion D. Primordial cysts E. Dental cysts 67. Most commonly associated with Sialolith wharton 68. Pt has fever, unilateral facial swelling, yellow discharge from parotid on the swollen side, diagnosis A. Sialolithiasis B. Warthins C. Pleomorphic adenoma D. Nectrotizing sialometaplasia 69. What causes turner’s hypoplasia - trauma or infection of primary tooth 70. Grade 3 furcation, best treatment hemisection, root amputation Grade1:scaling Grade2: gtr Grade 3: hemisection for mandi Root amputation for max Grade 4: extraction 71. Class 2 bone loss, best treatment 72. You can only place one implant in a pt taking warfarin. The INR should be A. 2.5 B. 3.5 C. 7 D. 12

Page 88: Decembery RQS – NBDE II

Combined remembered Qs 2020 73. Test to check kidney function - creatinine 74. Chronic alcoholic pt comes for extraction of 22 teeth, which test you suggest A. CBC B. INR C. BT 75. Which of the following is not an NSAID A. Ibuprofen B. Aspirin C. Celecocsib D. Acetaminophen 76. Questions on unbundling 77. Questions on downcoding 78. To study effect of specific drug on a disease, what is it called A. Cohot B. Case control C. Randomized trails D. Cross sectional 79. Preauricular surgery or incision which nerve is mostly likely be damaged facial 80. In IANB, you inject into a artery that is a A. Lingual artery branch of ICA B. Lingual artery branch of ECA C. Inf alveolar artery branch of ICA D. Inf alveolar artery branch of ECA 81. Pterigomandibular raphe consists of what sup constrictor and buccinator 82. What is the only muscle that gets you pierce through while giving IANB buccinator 83. Dentist places pit and fissure sealant on a active decay, what happens to the decay A. Increase B. Decrease C. Stay same D. Arrest

Page 89: Decembery RQS – NBDE II

Combined remembered Qs 2020

84. How does arrested caries look on a root surface- leathery brown 85. Two different drugs with same dosage binds to the same receptor. One drug has more effect than other. It has more A. Efficacy B. Potency C. Pka Drug a has higher efficacy than drug b: options: drug a needs lower dose bc its stronger, drug a has higher affinity for receptors Drug A gives maximum effect than drug B drug A has higher efficacy than B more potent smaller dose When comparing drugs with respect to intensity of response the drugs that produce greatest maximum effects is the one with the highest Affinity Potency Efficacy Therapeutic index

Page 90: Decembery RQS – NBDE II

Combined remembered Qs 2020 86. Some question about pka also was there ( don’t remember exactly) 87. Where is the thickest and thinnest attached gingiva (question in the exam was framed differently, but the meaning is same as what I wrote above) Thinnest mand pm and canine Thickest maxillary centrals 88. Mandatory Annual screening test for dentist is TB 89. Dentist did a composite class 5 filling, pt came back next day with pain because A. No liner or base B. Too much acid etching C. Improper cavity prep D. Axial wall is deep 90. Characteristic of ectodermal dysplasia 91. What condition it is if it has cleft palate, glososptosis and micrognathea - Perry robin 92. Not required in a cavity prep in a class 2 on a primary tooth- need not rounden the internal line angle No gingival bevel 93. Cledocranial dysplasia characteristics 94. Which local infiltration you give for maxillary premolar A. Endosseous Local Infiltration B. Paralingamental C. Transligamental D. Supraligamental (Weird option, I know) 95. Which is common between chronic periodontitis and localized aggressve periodontitis A. Number of teeth involved B. Treatment C. Other options I don remains Find for pocket, tooth mobility, bone loss is similarity and difference is aggressive doesn’t have plaque, calculus, inflammation 96. Initial treatment for localized periodontitis srps and abx metronidazole, tetra or amoxi 97. Which of the following condition requires gingivoplasty - ANUG 98. 7 year old, fractured Right central incisor, 3 hrs ago still bleeding. Treatment is A. Pulpectomy and apexification B. Pulpotomy and Cal hydroxide C. Direct pump capping

Page 91: Decembery RQS – NBDE II

Combined remembered Qs 2020 D. Pulpotomy with apexogenesis 99. Dentist gave IANBon right side of mandibule, pt feels tongue numb but not lip. Where do you give anesthesia to numb lip A. Inscisive B. Mental C. Local infiltration D. Left IANB 100. Reason for saliva seeping through the rubber dam A. Holes too far B. Holes too close 101. Pt is allergic to latex, what is the next step you do A. Do not use rubber dam B. Use rubber dam with napkin C. Use nitrile dam 102. Most common problem a dentist faces while restoring a class 5 with composite A. Cavity prep B. Moisture control C. Bond to tooth 103. Medication for status epilepticus diazepam 104. Commonly used porcelain for PFM (weakest) Feldspathic porcelain <Leucite-reinforced ceramic < Castable glass < Glass-infiltrated alumina (strongest) 105. Eruption sequence of primary teeth central lateral 1st molar canine 2 molar 106. Each of the following cause perio ptoblem, except A. Trauma from occlusion B. Nicked soft tissue while tooth preparation C. Rough over hanging restoration 107. In DMF, F is more in - whites 108. Which has greatest effect on polishability of composites filler size 109. Dentist can start prescribing fluoride supplements since what age A. 6 months B. 3 years C. 12 years

Page 92: Decembery RQS – NBDE II

Combined remembered Qs 2020 D. 16 years 1. Pt os 16 year old, which of the following you’ll not do - pit and fissure sealants 110. Termination of local anesthetic articaine - plasmacholinestirase 111. MOA of penicillin V oral..cell wall synthesis 112. Reason my preferring Pen V over Pen G because, pen G is

● Pen v oral, is less sensitive to acid degradation that’s why its given oral and not Pen g

114. Tetracycline acts by interfering with protein synthesis 30 115. Mental retardation caused by placental transfer of rubella is caused by A. Aquired B. Chromosomal C. Genetic March 3-4 Rq's DAY 1 1. Drug interaction Epi+LA 2. Drug interaction Epi+Propanolol/Amox

● Propranolol non selective beta blocker, used to treat hypertension ● Drug of choice for adrenergically induced arrythmias ● Beta blocker and epi hypertensive crisis ● Duration of lidocaine can be increased in presence of propranolol (blocks b1 and b2) ● Propranolol + epi will lead to increase in BP without tachy ● Block both b1 and b2 but mostly b1 (mosby) ● Propranolol by itself causes bronchoconstriction, heart block

3. Diagnose Lichen planus(pic from mastery day 2 file)

Page 93: Decembery RQS – NBDE II

Combined remembered Qs 2020

4. Benzodiazepine preferred over barbiturates reason?

● Barbitures are more potent ● less addiction ● Benzo doesn’t produce hangovers ● Barbiturate serious drug dependance ● are much safer that benzo ● Less resp depression ● Barbiturates have greater sedative effect and hypotonic

Thiopental most common used barbiturate 5. Benzodiazepine MOA potentiate the inhibitory effects of GABA. Enhance the effect of GABA Sulfonamide inhibit gaba 6. Naltrexone used to treat addiction. Also methadone 7. Flumazenil benzo antagonist 8. Prazosin a1 blocker. Ending in “osin” is a1 blocker

α1 blockers are used to treat hypertension, heart failure, and benign prostate hypertrophy (α1 blockers cause vasodilation, reduce afterload and preload of the heart, reduce contraction of Qsmooth muscle in the sphincter and trigone muscles of the bladder, and reduce contraction of the prostate).

Page 94: Decembery RQS – NBDE II

Combined remembered Qs 2020 Adverse effects of α1 blockers—hypotension (especially first-dose effect), fluid retention, dry mouth, nasal stuffiness.

9. Tetracycline bacteriostatic, inhibit protein synthesis 30s 10. Broad spectrum side effect? Opportunistic infections, superinfection commonly candidiasis and resistant bacteria. 11. Drugs causing xerostomia atropine, amphetamines, benzo, anticholinergic, ganglionic blockers, antidepressant

● TCA Doxepin, Imipramine,Amitriptyline Inhibit reuptake of serotonin and ne ● Phenothiazine ● Dephinhydramine ● Atropine ● Diazepam ● Midazolam

Pilocarpine (head and neck xerostomia) and cevimeline (pt with sjorgen) used to treat xerostomia. 12. Antacid cimetidine h2 blocker H2 ranitidine or cimetidine to decrease acid 13. Why Opioid not safe? Resp depression 14. Metoprolol-selective/non selective selective b1 blocker 15. Which is both agonist & non agonist drug? Pentazocine 16. Atonolol side effect b1 antagonist Which of the following is not related to a drug toxicity of Atenolol? A. CHF B. Tachycardia C. AV block D. Sedative appearance

Page 95: Decembery RQS – NBDE II

Combined remembered Qs 2020 17. Pilocarpine causes cholernergic. its for xerostomia and treats open angle glaucoma too Causes: sweating, nausea, heartburn, diarrhea , miosis 18. Antimicrobial agent chlorhexidine 19. Diphenhydramine it’s benadryl - allergic to both lido and ester 20. Physostigmine reversible cholinesterase inhibitor. Antidote for atropine

Physostigmine inhibits acetylcholinesterase, which metabolizes acetylcholine.

TCA overdose phygo

TCA Overdose. drug of choice is: Atropine Phenytoin Physostigmine phenobarbital

21. Amitriptyline TCA. #1 side effect is drowsiness and xero TCA causes ortho hypotension, hyposalivation, drowsiness Epi and TCA cant be used together cause severe hypertension

a drug Amitriptyline had adverse effect, Except?

a. Hyposalivation

b. Ortho Hypotension

c. Drowsiness d. weight gain 22. Asthma management-give oxygen, upright position,etc...in options

Page 96: Decembery RQS – NBDE II

Combined remembered Qs 2020 Acute asthmatic attack

● Supplementation of oxygen with inhaled beta 2 adrenergic agonist (albuterol) ● If pt resists to beta agonist theophylline

IF SEVERE asthmatic attack that is unresponsive to above txt 0.3mg of 1: 1000 epi subcutaneously

23. Dentigerous cyst pic identification: always associated with a crown of eneruoted tooth especially 3rd molar. Lined by non kera stratified squamous epithelium, no rete pegs

24. Kennedy class 3 modification 1 25. Why opaque is seen above porcelain teeth Opaque showing through means:

● Inadequate tooth reduction dentist fault ● Opaque layer too thick lab fault ● Inadequate thickness of body porcelain lab fault

According DD,function of Opaque porcelain are 1.mask dark oxide color 2. gives the restoration its basic shade 3. provide porcelain metal bond 26. Porcelain failure reason: Poor metal framework main cause of fracture

Page 97: Decembery RQS – NBDE II

Combined remembered Qs 2020 Surface micro cracks are caused by cooling stresses related to poor thermal conductivity of porcelain. Built in stressed in the porcelain contributes MOST to PFM failure. 27. Gold malleability term-brittle Ductillity metal ability to easily be worked into desired shapes Malleability-a metal’s ability to be hammered (compression) into a thin sheet without rupture. Malleability depends on plasticity, but is not as dependent on tensile strength as ductility. Malleability increases as temperature increases 28. Composite restoration failure in a day reason-debris on filling while curing, condense 2mm in chunks? 29. Pulp floor of composite class 2 perpendicular to buccal & lingual walls-always, only in amalgam, round line angles? 30. White spot in center of healthy max central incisor-hypocalcification, fluorosis,Dentinogenesis imperfect 31. Facial bow in an anterior retainer reason-tip lingually? the facial bow in maxillary retainer used to . A.tipping maxi teeth lingually . B.produce no movement .just retain It`s used for retain...if some relapse happens you can activate the facial bow to some extent for little lingual tipping of the anterior, but it`s not its primary function (and nobody wants that relapse happens lol) 32. Lateral periodontal cyst R/G 33. Furcation depth for max molar-4mm 34. Furcation depth for mand molar-3mm?

Page 98: Decembery RQS – NBDE II

Combined remembered Qs 2020

35. Reason for post placement retain core The post should equal the crown length or 2/3 the root length (whichever is greater). 36. Patient is 21 year old, mother financially responsible. Should you answer her questions or not or with permission of pt. Or only answer ques financially related? Answer with permission of pt, pt is overage 37. Syneresis irreversible hydro. Skrinkage when placed in humidity 38. Unbundling 39. Heart+kidney=Steven Johnson. Type 4 hypersensitivity Anticonvulsants/antipsychotics. Ethosuximide and valproic acid 40. Supernumerary cleidocranial 41. Oligodontia+hypodontia=ectodermal dysplasia 42. Condensing osteitis: from inflammation. Entire root is seen rather than in cementoblastoma

Page 99: Decembery RQS – NBDE II

Combined remembered Qs 2020

43. Gingival recession due to plaque other factors=age? Can be bc of age yes Betel nut cause everything except: Gingival recession Xerostomia SCC Staining 44. Drug causing gingival hyperplasia? Phenytoin, calcium channel blockers, valproic acid, cyclosporine

● Nifidipine ● Amlopidine ● Felodipine ● Verapamil ● Dilitazem

45. Imipramine-antidepressant TCA DIA, Doxepin, amitriptyline, imipramine 46. Cheek sweating-Frey syndrome 47. Irreversible pulpitis-"lingering" 48. Pulp necrosis-"no response to tests heat or cold"

Page 100: Decembery RQS – NBDE II

Combined remembered Qs 2020 49. Contraindications in cleaning implant-wet pumice ( I think this is false) 50. NSAID without COX mechanism cox 2 only is Nsaid does not affect platelets Cox 2 selective

● Celecoxib ● Rofecoxib

51. Pulse oximeter used to measure? measures the saturation of o2 carried in your red blood cells. Percentage of oxygen in red blood cells. 52. Which antibiotic prescribed for children-erythromycin,streptomycin,etc in options? AMOXICILLIN THE BEST 53. Intestinal polyps Gardener 54. Pt. On warfarin/coumadin, what blood test-CBC,INR,Hb? 55. Anesthetizing canine and premolar only which nerve? No lingual in option only Inferior alveolar and lingual 56. Facial nerve passes through facial canal, pterygomand. fossa? 57. Implant bone width can be found by which process-CT,MRI,Study models,PA? 58. Bells palsy occurs when needle passes too far forward,backward,lateral,medial? 59. Electrical shock pain after biting trigeminal neuralgia 60. "Radiolucency"-always chronic periodontitis 61. "Sinus/Swelling"-always acute periodontitis 62. Parulis-infection/pus/abscess 63. Masseter buccal flange in denture 64. Patient management- I can't even explain the questions as my brain was in 'WTF' mode. Day 2 cases:

Page 101: Decembery RQS – NBDE II

Combined remembered Qs 2020

● Condensing osteitis ● Angles class 2 div 1 ● class 3 ● exostosis ● melanotic nevus

Melanotic macule is flat Melanotic nevus is raised

● which condition causes TMJ crepitus can be osteoarthritis aka degenerative joint disease

● talon cusp

● epulis fissuratum: surgical removal before new dentures. Caused by overextension. Clinically like fibroma but histologically like granular cell tumor.

● lichen planus ● herpes labialis/primary herpetic gingivostomatitis ● increased VDO ● patient's upper lip interfering in the Pantograph ● patient on drug abuse what should you cautious to prescribe ● patient financially down as single mother what would be treatment option for

upper(overdenture,rpd,implant,OTC denture liner) ● asymptomatic chronic apical periodontitis ● irreversible pulpitis ● periapical abscess ● amalgam seen on top of porcelain veneer reason(not triturated properly,not

condensed properly,creep,corrosion) ● drug causing osteoporosis corticosteroids..prednisone

Page 102: Decembery RQS – NBDE II

Combined remembered Qs 2020

● patient has history of prostate carcinoma, HIV +ve, HT,hypercholesterolemia, taking Ca channel blocker, warfarin-(kaposis sarcoma, reason for xerostomia glands are dry upon manipulating:medication/sjogren/etc),

● same question in another patient glands secrete very little upon manipulation reason for xerostomia,

● identify teeth with low copper amalgam, ● what do you consider dangerous in this patient before proceeding with dental

treatment( bleeding,infection,etc), ● pt with hep A(dentist deny treatment comes under which poilcy-pt.mgmt justice ● pt with knee replacement 6 months ago undergone osteoradionecrosis 60 Gy for

some carcinoma 10 years ago do you prescribe antibiotics or send to his specialty physician for prescribing

● patient with ampicillin allergy what other antibiotic do you prescribe ● pt. with generalized periodontitis, very slow improvement with very little bleeding on

probing what surgical procedure do you do ● A case on aggressive periodontitis in a single tooth, a case involving furcation and its

treatment. For day 2, "mastery app day 2" and "mastery day 2 dental cases file" are good sources and well enough. Make sure to manage time well. As it will for sure take time to think on cases before answering. 1 hour 50 minutes for each section. 7 cases with 49 questions and 2nd session with 7 cases with 51 questions. Kindly look at photographs & radiographs before answering any radiology questions. And same applies for O.path. because you never know where you can find the clue. That's it guys. All the best! Keep me in prayers. May we all pass! Amen. March 6/7 It is my remembered question 6-7march The exam over all accepted , hope i will pass Ortho Tri-Cyclen Lo (Norgestimate +Ethinyl Estradiol) which component cause dry socket: estrogen part Bared alveolar bone after a week of treatment and the patient complain of pain ( alveolar osteitis, acute osteomyelitis) ??!! here it should be alveolar osteitis as there is no pus and occurred in 1 week after extraction Prefered place for ginigval graft (palate, retromolar pad)??!!

Page 103: Decembery RQS – NBDE II

Combined remembered Qs 2020 Sulcular brush technique ( modified bass) Stress and fear ,stress relation to pain : Fear inc pain tolerance pain, stress more pain Endodontic treatment should be some time ,liippstressful , and the patient say he have his own ways to control stressful situation so his pain will (increase , decrease )?? If he face stressful situation Tranqulizers and drugs u should avoid with alcohol Extrusion movement 1mm per week , 1 mm per months ......??!! Histoplasmosis lesion resemble ( candidiasis , squamous cell carcinoma ) TB and its ulcer in oral cavity Intraoral lesion of TB seen as a)tonsilitis and ulceration b)ulceration on buccal mucosa c) involving tongue Granular cell tumor (psuedoepithetusliomatous hyperplasia) Epulis is histologically similar to what: Traumatic fibroma Traumatic neuroma granular cell tumor Epulis is clinically similar to what: Traumatic fibroma Traumatic neuroma granular cell tumor Agonist ( high intrinsic activity and high affinity) and antagonist ( no intrinsic activity high affinity ) Rapport Cleft palate and it predisposing factors , and time of surgical treatment 6-9 months( According to ADA it should be between 3-6 months ) Most common seizure type in children febrile

Page 104: Decembery RQS – NBDE II

Combined remembered Qs 2020 Ttt of grand mal seizure phenytoin Ttt of status epileptics valium diazepam What is the meaning of conjugation of drugs adding molecule to drug Trigeminal neuralgia treated with carbamezepine Naproxen Epithelial attachment to implant hemidesmosones Percentage of calcific metamorphosis 5-18% Percentage of herpes simplex 85% Dentigerous cyst 1cm treatment ( enucleation , marsupalization ) In decks it says uncover the crown and keep exposed Panorama (hyoid bone , ear ) Panoramic xray defect if patient move his face for 1 sec during exposure what is the pattern of the defect can be vertical changes border changers what would happen if patient moves during panoramic 1-, one vertical line blurry plus a discontinuation of mandibular border 2-wavy mandibular border Xray for mesiodent ( occlusal , angulated with shift technique ) ?? Varices of the tongue ( old age ?? , hypertension?? ) hypertension then age Idiopathic osteosclerosis Bolimia prevelence most in ( male, female,black,white,hispanics,) white female 16 yrs Culture hypersensitivity meaning ?! Culture sensitivity test? Bacterial resistance Determine the antibiotic tt

Page 105: Decembery RQS – NBDE II

Combined remembered Qs 2020 Thyroid storm no epi, high BP, tachy Mederated calcified upper central at high risk for what ( file separation ? , access perforation ??! ) Success rate of direct pulp capping depend on ( age , good coronal seal )???!!! March 13

1. Which is the best test for long term controlled diabetic patient Hba1c

2. When using an occlusal rim what we are transferring to the articulator centric relation..vdo?? - Ans:vdo

3. Why we give atropine before anesthesia - to reduce salivary secretion and bronchial

secretion atropine is given before anesthesia because

Decrease incidence of cardiac arrest b. Decrease in resp secretion c. Muscle relaxation d. Rapid recovery e. Smooth induction

General anesthesia with halothane is commonly preceded by administration of atropine to 1. inhibit vagal overactivity commonly caused by halothane. 2. induce muscular relaxation by blocking cholinergic receptors. 3. reduce salivation and bronchial secretions caused by halothane. 4. All of the above

4. Mechanism of action of carbidopa Carbidopa decreases the peripheral metabolism of levodopa

5. Dementia (3 /4 questions about the symptoms)

6. Steps for delivery veneer

Page 106: Decembery RQS – NBDE II

Combined remembered Qs 2020

7. Antibiotic against anaerobes and parasite ( metronidazole)

8. Pregnant lady.. which side we make her turn and which vessel we relieve ( left side / inf vena cava )

9. INR index value 1 too good to 4 max to perform any dental surgery

10.Calculate the mean

11.Lot of questions on case study, clinic trial and cross sectional

12.Main Difference between Medicaid: for destitute and Medicare : for elderly

13.Where should the tongue be in relation with occlusal plane on complete dent( above it

)

14.Uses of lidocaine as intravenous- arrhythmia for ventricular arrhythmia

15.What we give to convulsant patient (diazepam)

16.Hypoglycemic crisis what we give to the patient (something with a sugared drink) OJ juice if conscious

17.Pain control medication for recovered alcoholic options where oxycodone ibuprofen

acetaminophen

18.Exceecises comparing drugs A and B .. they were examples of potency and efficacy

19.Where we use cryer elevator mand molar roots

20.What we loose last when given local anesthesia ( pain touch pressure temp) lost in following order: pain→cold→warm→touch→deep pressure→motor

21.ASA classification of patient wearing a nitroglycerin patch plus taking it orally 2/3 day asa 4

22.Picture with mucocele

23.Where is most common to find sialoliths Wharton subman

24.What type of study emite X radiation

Page 107: Decembery RQS – NBDE II

Combined remembered Qs 2020

25.Patient limited opening pain and jaw deviate to the right .. where is the injured side right (deviates to same side)

26.Característic of trygeminal neuralgia

27.Best rinse for child with high Caries NaF

28.Uses of 10 per cent chx varnish: Articles online say root caries

Where do we use 10%chx varnish? 1 P&f caries prevention 2 white smooth surface 3 caries prevention 4 secondary caries preventionand root caries

29.Stages of gingivitis and microorganisms

30.A picture of a dentinogenesis type 2

31.Garner syndrome

32.Cleidocranial dysplasia panoramic xrays

33.A lot of diagnosis of endo

34.Combined perio-endo lesions EPT

35.Causes of xerostomia medication

36.What to do when detect abuse of elders: report to HHS

37.Pregnant patient 14yo.. should we tell the parents or not don’t need too, shes

pregnant

A conscious, mentally competent patient younger than 18 may give consent to his or her own medical treatment, counseling, or testing if he or she is emancipated, married, a parent, pregnant, or in an emer- gency situation. An emancipated minor can also consent to treatment of his or her child.

38.Muscles on denture molding for mandibular dentures masseter

Page 108: Decembery RQS – NBDE II

Combined remembered Qs 2020

39.Característics of zirconia

40.Where should we stand when taking xrays : 6 feet away

41.What we correct when we change the angulation in the articulator.. something about the occlusal cusps (NO CLUE)

The incline (angulation) of the condylar element on the articulator is anatomically related to the slope of the condylar articular eminences (condylar inclination).

42.How we make sure we obtain informed consent

43.What to do when patient feeling nauseous during nitro at 50/50 rate 100% oxygen

44.Allergic reaction what we give .. Epineprine 0.3

45.Ranula picture then what’s the best treatment of recurrent

Decks:The treatment is surgical, either through complete excision or by removing the roof of the cyst. If it persists, excision of the gland may be needed.

46.Prolongated Corticosteroids teraphy consecuentes - osteoporosis and hyperglycemia

47.A picture of side occlusion patient determine the oclusal class

48.Causes of loose crown on implant

49.Causes of failure of gingival free graft Decks: disruption of the vascular supply

before engraftment. The second most common reason is infection.

50.Digitalis mech of action increasing cardiac contractility

51.Best xray to evaluate bone shape bitewings

Page 109: Decembery RQS – NBDE II

Combined remembered Qs 2020

52.What we change if we want to increase energy on xray inc kvp

53.Parts of the biological space junctional epithelium and ct

54.How far an implant has to be of tooth next to it 1.5mm

55.Bell’s palsy facial

56.Neurofibromas on tongue

57.Common location of intraorl melanoma max gingiva and palate

58.Face bow uses

59.What’s the support on removable

60. Identify Kennedy class picture

61.Some questions on Gracey curettes number

Universal is 2 cutting, 90 Gracey is 1 cutting 60 70 (cutting age offset and curves not like universal) Gracey working angulation more than 45 less than 90

62.Etiologíc factor of warts (sexual) papilloma virus HPV

63.When we premedicate

64.Hypertensive patient.. suddenly blurred speech and progressive loose of conciuos

what is it.. acv?

65.Angles on preparation for amalgam (round -sharp?) Decks: Internal line angles in amalgam class 2 are round Internal line angles in inlay are sharp

Page 110: Decembery RQS – NBDE II

Combined remembered Qs 2020

66.Gingivectomy healing secondary

67.Lesión less than 1cm that doesn’t go away what we do ... excisional biopsy if we

suspect malignancy then we to incisional

68.They asked me which antibiotic is against anaerobes and some parasites The option where : penicillin , cyprofloxacin, metro, azytro

69.Then they asked me mechanism of action of carbidopa and the options where so long

..

70.Why is it that they combine hydroxycodone with acetaminophen: syngertic effect they potentiate eachother (from group) One says that they have diferent effects so they both together are better

March 16/17 1. Which cells are affected in most common leukemia in children? ALL, acute lymphoblastic 2. Dens in dente most common? Max lateral 3. What will happen if the sinus tract is not drained? Decks:cellulitis/bacteremia o osteomyelitis 4. Hypodontia in which syndrome? Ectodermal dysplasia 5. Wavelength associated with what? Hue 6. Where to give retention grooves in class 2? Incisoaxial and gingival axial 7. What is not included in informed consent? Price according to decks…can be written or oral 8. Success of indirect pulp capping depends on? Success of indirect pulp capping depends on 1 age of the patient 2 exposure location 3 coronal seal 4 thickness of CAOH 9. NSAID MOA? Blocks cyclooxygenase function, inhibiting platelet aggregation. 10. Why it is difficult to perform flap surgery near mandibular 1st and 2nd molar? Less attached gingiva 11. Kennedy class 3, modification 1, which RPD design? Single palatal strap 12. Supernumerary teeth present in? Inititation 13. TB oral ulcers most common where? Tongue 14. What can not be advertised by a general dentist? Specialty 15. GTR can not be performed in which class of mobility? Grade 3

Page 111: Decembery RQS – NBDE II

Combined remembered Qs 2020 16. Fractured tooth fixation time? 3 months 17. Minimum distance between 2 implants? 3mm 18. What should be the width of the bone if we want to place 4mm diameter implant? 6mm 19. Best root support FPD? Best root support FPD 1 long less bone lose 2 Short less bone lose 3 Conical more bone loss 4 Conical less bone loss 20. Displacement of odontoblastic processes caused by? Dissecation 21. Most common osseous defect? 2 wall aka crater. (remember hemiseptum is 1 wall, crater is 2 wall, trough is 3 wall) 22. Varicosis associated with? Hypertension 23. Down syndrome will have all except? Rampant caries 24. Which muscle is most likely to get pierced in IANB? Buccinator 25. Xray: Hyoid bone, ghost image of earing, tongue, calculate the age of the patient by the xray 26. Which cyst is most likely to cause ameloblastoma? Dentigerous cyst 27. Cocaine side effect CAUSES MYDRIASIS. All other opiods cause miosis 28. Bypass and nutrition study cohort 29. Home internal bleach causes what sensitivity 30. Treatment of internal resorption? RCT 31. Paraphrasing questions 2 times 32. Most allergic material nickel 33. Migrane treatment triptan 34. Relation of teeth when saying Siliant sounds Close together not touching 35. Role of zoe pack after perio surgery decks: doesn’t help in healing process 36. Learn number of rests and location of rests in all Kennedy classifications. Most prostho questions from RPD 37. All drugs cause gingival enlargement except 38. Mouth wash for down syndrome patient CHX 0.12% 39. Pictures: papilloma, varicosis, leukoplakia, lichen planus ‘[p2240. Where to put pin in amalgam restoration? 2mm into dentin 2mm within amalgam Should be 0.5 to 1.00mm of dentin in between pin and DEJ The largest pin that can safely be placed should be placed Places at the linge angles or corners of the tooth (where tooth/root mass is >) 41. How to prepare posterior teeth? 42. Enamel demineralize at which ph? 5.5 43. How much exposure to the patient who has occupational exposure? Occupational 0.05

Page 112: Decembery RQS – NBDE II

Combined remembered Qs 2020 Non occupation is 0.001 44. MRI uses which radiation? radiowaves 45. Benzo MOA potentiates GABA 46. Whom to contact in medical emergency? 47. Biopsy for 2mm*4mm lesion excisional excisional biopsy should be done if lesion is less than 1cm (10mm) 48. Biopsy for white lesion on ventral surface of tongue incisional 49. Sinus tract is associated with? Chronic apical abscess 50. Marginal discoloration of crown? microleakage 51. Orange discoloration of teeth due to? Poor OH 52. Luxated tooth gives negative EPT result, why? Nerve endings have been interrupted 53. Cracked tooth with no pulpal involvement extracoronal 54. Turner’s tooth cause infection 55. Cleft palate cause multifactorial failed fusion of palatine shelves 56. Differential diagnosis of epilus fissuratum fibroma

57. What to do when patient has burning sensation under the denture, patient wears dentures for 5 years

Decks: Pressure areas on the impression surface of dentures is checked with PIP (Pressure Indicating Paste). Use digital pressure only, one denture at a time. Special attention should be given to the hard palate and the mylohyoid ridge areas.

58. Dentinogenesis imperfecta associated with? Osteogenesis imperfecta 59. Dry mouth- Sjogrns syndrome cevimeline 60. Most common carcinoma oral SCC aka epidermoid 61. Toughest dental impression material polyether 62. Xray- coronoid process 63. Orthostatic hypotension associated with which drug nitroglycerin ? 64. Pt has sickle cell anemia and has thrombolytic crisis, what can precipitate this? COLD 65. Patient has hypotension, reason for unconsciousness in middle of the treatment? syncope 66. Bisphosphanates affect which cells osteoclast 67. Which drug used for mild sedation most commonly used benzodiazepine as a premedication for sedation? Midazolam (confirmed in decks) lorazepam diazepam especially for IV because no thrombophlebitis because lack propylene glycole

Page 113: Decembery RQS – NBDE II

Combined remembered Qs 2020 68. Question about meperidine synthetic opioid, causes mydriasis (this is an exception just like cocaine) cant be mixed with MAO, avoided in renal disease 69. Best location to place implant ant mand 70. What to check first if the patient becomes unconscious responsiveness 71. What to check first while placing crown internal/esthetics Decks: When trying crown

1. Check internal 2. Check proximal contacts and Pontic ridge relationship 3. Check marginal integrity 4. Check stability 5. Check axial contours 6. Check occlusion

72. Why the crown wont fit on tooth but will fit on cast? You notice void on occlusal of cast. Crown will a. Fit on die and not on tooth b. Fit on tooth and not on die c. Fit on both d. Not fit on either 73. How to place patient position with asthma attack upright 74. Steps in formation of plaque pellicle, biofilm, material alba, plaque, calculus 75. Why tetracycline used in perio inc concentrations of GCF 76. Where does the extra fluoride get deposited in the body skeletal 77. Which vaccine for staff every year? Influenza 78. Pharmacology: cardiovascular drugs, sedatives, antibiotics, nsaids, analgesics, antidepressants (so many questions) 79. Xray- mandibular canal 80. Treatment of alveolar osteitis Decks: Flush out debris with slightly warmed saline solution- gently!!! •Place asedativedressingin socket(eugenol).Thedressing shouldberemovedwithin 48 hours and replaced until the patient becomes asymptomatic. Note: (1) The gauze provides an attachment for the obtundentpaste so it stays in the socket (2) Eugenol is the active component in most sedative dressings (3) If gel foam or another resorbable materialis used then, the dressing does not needto be removed (4) The medical term for dry socket is alveolar osteitis • Nonsteroidal antiinflammatory analgesics should be prescribed if necessary. 81. Treatment of 7 years old, fractured central incisor 3 hrs ago, pinpoint pulp exposure: pulpotomy 82. GMT and Hachet angle is different 83. Class III patient, What increases as age of the pt.increase- given all angles

Page 114: Decembery RQS – NBDE II

Combined remembered Qs 2020 Snb increases with age Anb decrease with age Dye 84. Odontomas location mand molar ramus 85. Fluoride table 86. Cross allergy with penicillin which drug cephalosporins 87. Prazosin acts on? Alpha 1 blocker 88. From which side do you see distal caries more clear in posterior teeth 89. Favorable position of soft palate for complete dentures line between hamular notches is 5-13, which is class 1 (House’s classification) 90. Strength of connector in FPD is determined by? Occluso gingival 91. Generalized widening of PDL in? scleroderma or osteosarcoma 92. Why there is space between primary teeth? 93. Drugs used to stop the saliva anticholinergics..atropine 94. Calculation about perio pocket with gingival enlargement 95. Which block to give if you want to anesthesize mandibular anteriors and premolars IAN 96. Which insurance where an employee can not choose the dentist of his choice closed panel 97. Luting cement for porcelain crown resin 98. Which structure of permanent teeth is damaged when a primary teeth gives pressure on it during formation enamel 99. Beaten metal seen in skull radiograph Crouzon syndrome 100. Patient receiving chemotherapy needs extraction, which lab test full cbc to check white blood count and neutrophil count 101. Amalgam failure reason moisture ( fructure mostly cavity preparation?) 102. Patient got punched on right side of the mandible, which side would be fractured Left side 103. Most common site of fracture? condyle 104. Most imp factor that determines the success of implanting avulsed tooth? time 105. In a group of 100 people, 50 has restoration, 20 had caries and 30 had both restoration and caries. What is the probability that if you pick a patient, he would have caries 50 106. There is caries still left in the middle after you prepared the cavity. How would you excavate the caries with bur large round bur 107. For class III jaw relation, what is done along with BSSO surgery lefort 1 108. Most common malignancy of salivary gland mucoepidermoid 109. Which ion is blocked when we give LA NA sodium 110. All are the characteristics of Kelly syndrome, except? Inc vdo 111. Most common missin teeth, no molars and premolars in option max lateral 112. Blue sclera seen in? osteogenesis imperfecta 113. Ethics questions mostly with cases on 2nd day 114. 2nd day most of the cases were about old patients with CVS diseases 115. Patient is taking ACE inhibitor, patient urines more or less less 116. What does dycal do in pulp capping Decks: Pulp capping is the placing of a sedative and antiseptic dressing on an exposed healthy pulp to allow it to recover and maintain normal function and vitality. The dressing most commonly used is CaOH 2 (Dycal). 117. Scarlet fever strep pyogenes causes strawberry tongue (inflamed fungiform) and skin rash

Page 115: Decembery RQS – NBDE II

Combined remembered Qs 2020 118. Schedule drugs list schedule 2 is anything finishing in done, cant call in, need prescription 119. Condyles seen best in which view Reverse towne 120. Drug for sialolith 121. What is aversive conditioning positive punishment for example a ortho appliance

Singh RQ March

1. Trephination-hole in cortex of bone 2. HBa1c -uncontrolled is above 8%

a. 7-7.5 b. 8-8.5

3. HbA1c a. serum b. blood

4. NIOSH-ventilating nitrous oxide in office

5. Noble metal a. palladium b. titanium

Noble metals are gold palladium platinum

6. Arch length is between 1st molar

Arch length is measured at the midline from a point midway between the central incisors to a tangent touching the distal surfaces of the second primary molars or the mesial surfaces of the first permanent molars.

7. Most common oral cancer in

Page 116: Decembery RQS – NBDE II

Combined remembered Qs 2020

a. white b. black c. hispanic d. asian

8. Most common periodontal problem -black males 9. Silver diamine fluoride-side effect is staining(read about this a little 3-4 question on

this )38% SAF is used 10.Lisinopril-cough as side effect 11.Lisinopril-ace inhibitors 12.Which of following is not competitive blocker - one option was erythromycin

ok so erythro is a macrolide and its competitive so that wouldn't be an answer...in general all bacteriostatic are competitive

13.Propanolol with erythromycin -hypertension and brady 14.Selective beta blocker -atenolol

· Metaprolol is also a selective beta 1 receptor blocker. Atenolol and metaprolol both

· Propranolol is beta 1 and beta 2

15.Diazepam-flumazenil 16.Opioid-naloxone 17.A question stating what u dont ask from guardian of asthma patient and one option

was does does she need to take both sambutamol and albuterol inhalers 18.Symptomatic Periapical periodontitis-which medication for chief complaint(pain)

a. ibuprofen b. ibuprofen and antibiotics

No need for abx

Remember: NSAIDS are contraindicated in asthma, causes bronchospasm

19.Two question on LA dose calculation 20.Patient got mutiple carpules of 2% la 1:100000 epinephrine.got agitated due to

vasoconstrictor 21.Steroids avoided in gastric ulcers. ( other option was it causes mental disturbances) 22.For acetaminophen u need to check patient liver status

Ibuprofen-->kidney

23.Bleaching question -coronal leaking do not cause rct failure,yes because most rct causes hermetic seal-both statements are incorrect

24.Sickle cell anaemia patient came for extraction what is least desirable a. systemic antibiotic

Page 117: Decembery RQS – NBDE II

Combined remembered Qs 2020

b. nitrous sedation

NITROUS OXIDE is NOT contraindicated in sickle cell

25.Tetracycline bacterioCIDAL for bacteria (false statement). Tetracycline in reality is a

bacteriostatic drug

Cidal mnemonic: CAMP FV

Cephalo

Aminogl

Metro

Penicillin

Fluoro

Vancomycin

26.Biotransformation is mostly done in plasma and kidney(true or false statement) 27.Enamel dissolution under 6.5 ph(incorrect) the correct answer is under 5.5 pH

Enamel demineralization ph 5.5 or lower

28.Remineralisation of enamel from minerals of saliva not floride 29.Fluoride effect except increase phosphate in tooth 30.Anticonvulsant all except

a. phenytoin b. barbiturates c. ✅ lasix (diuretic) d. carbamazepine

31.Question on sensitivity.they gave two tables with ppl with diabetes and without diabetes with their true positive and negative results.

Page 118: Decembery RQS – NBDE II

Combined remembered Qs 2020

32.Frankfort horizontal plane 33.Anb 5-class 2 34.Lack of good margin on amalgam restoration but no recurrent caries

a. redo full restoration b. prepare and fill only the defect

OBSERVED! 35.Mand molar lack buccolingual retention what's given for resistance

a. buccal groove b. lingual groove c. proximal groove

Page 119: Decembery RQS – NBDE II

Combined remembered Qs 2020

d. 36.What defines definite patient behavior or something like that all options saying i WILL

do it but one says i MUST do it(i picked must) 37.Pvs-latex are gloves 38.Spore testing positive what is done next

a. check with physical indicators b. check which method of sterilization is used c. put autoclave out of order

39.How Sensor of digital rxworks- a. detects radiation b. senses radiation(check this one)

40.Scale on cephalogram tells us the magnification 41.No IR in class 1 RPD results in

a. tissueward movement of distal extension b. away from tissue

Indirect retainers: are placed as far away from the distal extension base as possible to PREVENT VERTICAL DISLODGEMENT of the base from the tissue

42.Class 3 mod 1 what's for stability

a. major connector b. rest

43.Major connector connect-rigidity and support 44.Full wax up of crown is done before wax cutout on metal framework of fod for uniform

flow of a solder b porcelain c metal alloy d retainer

45.Not in combination syndrome-inc vdo 46. Irritational growth in ant maxilla while making new denture what is considered

a surgerb relining c static denture

47.Denture induced candidiasis picture 48.AOT-ant maxilla

Page 120: Decembery RQS – NBDE II

Combined remembered Qs 2020

Rx showing erupting canine(normal follicle of canine)

49.Lack of attached gingiva on distal to last tooth .what is CI a distal wedge b gingivectomy

BOTH ARE CONTRAINDICATED BUT THAT SITE DESCRIPTION IS FOR DISTAL WEDGE.

50.Microorganism with root canal a obligate anaerobes b obligate aerobes c faculative anaerobe d facultative aerobe

51. Initiation of caries-streptococcus mutans 52.Progression of caries -lactobacilli 53.Pit and fissure caries-

a catch with explorer b subsurface opacity

54.Caries progress max at DEJ 55.Xerostomia can result in

a leukoplakia b salivary gland duct infection

Xerostomia causes all of the following EXCEPT

A.Smooth surface caries

B. Candidiasis

C.leukoplakia

D. Salivary gland infection

56.Root caries incidence has increased 57.Least to consider while composite post restoration

a temp conductivity b esthetic

58.With time microleakage around amalgam restoration decrease 59.Gum injury due to brush-recession 60.Least like to occur after ant crossbite correct

a recession of gums at ant mand incisors b stop growth of immature maxillary incisor

61.Seal perforation-caoh 62.Least likely treatment of internal resorption -let it heal on own

Page 121: Decembery RQS – NBDE II

Combined remembered Qs 2020

63.Fibro osseous dysplasia pic - stating vital tooth 64.Epinephrine acts on

a alpha 1 b aloha 2 c beta 1 d beta 2 e dopamine 2

65.Angina don’t need antibiotic prophylaxis 66.Diabetes 2 ,hypertensive patient having bp 172/96 most likely to suffer what during

treatment a hyperglycemia b hypertensive crisis c syncope

67.Patient with chest tightness and ache going to left arm first step A. oxygen B. aspirin C. raise chair to supine semi supine position

D. 68.Ranitidine-antacid

H2 69.Alginate exhibit syneresis even in 100% humidity 70.Arcin is better over non arcone

a transfer of facebow b transfer of cr (don't remember other)

Mosby: Arcon-condylar elements is in lower member Condylar path is in upper member -resembling

as tmj-in this condylar guidance and occlusal plane is fixed. Non arcon-condylar elements in upper

member. Condylar path in lower-most widely used for fabrication of complete denture.

Page 122: Decembery RQS – NBDE II

Combined remembered Qs 2020 Semiadjustable articulators—there are two types of articulators.

a. Arcon, in which the condyles are attached to the lower member of the articulator, and the fossae are attached to the upper

member. The mechanical fossae are fixed relative to the occlusal plane of the maxillary cast. This makes them more

accurate for fabricating fixed restorations, especially when an interocclusal record is used to mount the mandibular cast.

b. Nonarcon, which has the upper and lower members rigidly attached. The occlusal plane is relatively fixed to the occlusal

plane of the mandibular cast. These articulators provide easier control in setting teeth for complete and partial dentures.

(1) Semiadjustable articulators generally use an arbitrary facebow record; this orients the cast in the anterior-posterior and

mediolateral position in the articulator to anatomic average values (e.g., the use of the external auditory meatus to stabilize

the bow).

71.Orbital on facebow is for a. distance bw tmj n mand b. distance bw orbit and plane

Axis Orbital Plane is a horizontal plane, that is formed by the transverse horizontal axis of the mandible and a point on the

inferior border of the right or left bony orbit

72.Recording in protrusive jaw relation what changes are must in articulator related to guide pin a. increased distance bw guide table and pin b. add material to maje it angle n touch

73.Vdo assessment sounds a S b F c TH

74.Unilateral balanced a. mutual protected b. canine guided c. group

75.Freeway space calculation 76.Biological load rests in

a abfraction b attrition

77.Distance between implant and teeth - 1,5 mm

Page 123: Decembery RQS – NBDE II

Combined remembered Qs 2020

78.4mm implant what the min required space - 6mm, one 1mm on each side (buccolingually)

79.Anti Rotational elemment function in implant are to prevent rotation of: a- implant b- abutment

80. Internal connection in inplant a for abutment and implant b luting abutment c interimplant connection

81.Sebaceous glands in buccal mucosa -pic

82.Not a consideration while placing implant-caries rate 83.Auto Polymerization of acrylic 84.Voids and surface irregularities

a-increase stress b released stress

Internal void and surface defects = 1) leads to stress relaxation 2) decreases elastic modulus 3) increases translucency

85.Bull- working lubl -nonworking 86.Kennedy class 3 identification 87.Not a consideration for fpd a pt not willing for

a. perio assessment b. b caries risk c. c cost of treatment

Page 124: Decembery RQS – NBDE II

Combined remembered Qs 2020

88.Reciprocal arm of clasp- a cross arch stability b do not let the abutment tooth move while placing in rpd

The function of the reciprocal clasp arm is to

1. act as an indirect retainer.

2. stabilize the abutment teeth.

3. act as a direct retainer for the distal base.

4. counteract any force transmitted by the retentive arm.

A. (1) (2) (3)

B. (1) and (3)

C. (2) and (4)

D. (4) only

E. All of the above.

Reciprocal arm Should touch tooth before the retentive arm

Page 125: Decembery RQS – NBDE II

Combined remembered Qs 2020

89.Not a adv of 7/8th crown over full -retention 90.Not advised if crown is not fitting on tooth preparation-altering luting powder liquid ratio 91.Biological width -2.04mm 92.Epi in retraction cord-

a rapid pulse b necrosis

93.Supragingival finish line adv- a resistant b anterior c harm gingiva less during impression making

94.Color stability i put hema no udma there 95.Bsso nerve to be taken care of

a ian b lingual

96.Alv osteitis no antibiotics 97.Mand fracture-sports injury 98.Hyoid in x ray 99.Cleft lip palate most common congenital 100. DI associated with osteogenesis imperfecta 101. Mri -disc 102. Lingual frenal attachemnt -blue color

a venous channels b sublingual acini

103. Bite block a save operator fingers b prevent tmj injury b prevent trismus d keep tongue cheek out of place

104. Incisional biopsy more than 1 cm 105. Excisional less than 1 cm 106. Cytological smear→ candidiasis 107. Transillumination→ sialolith 108. Strands of neutrophil-

a. cyst b. abcess c. periodontitis

109. Middle face triangle -cavernous sinus 110. Infection from canine to orbit canine space 111. From premolar to orbit canine soace 112. Malocclusion Class identifications 113. Commonly impacted ant teeth- canine 114. Congenital Missing teeth max li (no pm,molars in option) -max lateral 115. Protraction of maxilla-protraction headgear facemask 116. Occlusal splint not for traumatic occlusion 117. Cross bite correct ASAP 118. Unilateral cross bite in very young a palatal expander b cross elastics finger spring

Page 126: Decembery RQS – NBDE II

Combined remembered Qs 2020

119. 26 months with 12 upper and lower carious tooth- under GA 120. -early loss of primary mandibular 2nd molar space maintenance a distal shoe b

nance c band and loop 121. Polyp -gardner and peutz jegher 122. Problem during apposition of teeth-color changes 123. Anug-blunted papillae 124. Anug -treatment local debridement Not a single question on apexo apexi😭 all the

very best guys please pray for us!thanks for your support -Singh & kahlon Another March RQ

1)Pt took aspirin nd got bronchiospam

What is contra indication? all anti inflammatory

Ibuprofen

Pentazocaine ?

2)Acytylate cyclooxygenase

Aspirin or acetaminophen ?

3) dfm calculation

4)Kid diffrent in treatment than adult that kid more probe to

1 dehydrated

2 bleed easily

5)Enamel of primary teeth

Most thick in

Cusp tip

Cusp tip and groove

In prox

?

6)Pateint didnt see dentist 10 yrs what x ray indicated

Page 127: Decembery RQS – NBDE II

Combined remembered Qs 2020 4 bitewing

Full sries

Panoramic

?

7)Best antibiotic for gingiva

Azithro or augmentin ?!

No doxycycline no tetracycline in options

8) We do hemisection nd root amputation in all except

Options

Root perforate from mesial

Root of mesial perforate and resorp

Most of crown lost due to caries !

9)Strongest

Paladium silver

Palladium gold metal

Type iii gold ???

10)Radiolucent Looks like stafne

Then options were traumatic

Abc

Or salivary gland something

11)what component of cement contributes to adhesion?

Aluminum silicate

So2

And two more

12)Frenectomy u do it for all except

Speech

Esthetic

Ortho

Page 128: Decembery RQS – NBDE II

Combined remembered Qs 2020 High pull margin ?

13)cellulitis dr want to do drain and antibiotic and pain medication

Pencillin or no treatment because of drainage

14 ) match the correct

Furocimide lasix , diuretics

Verapmil calan calicium canal blocker

15 ) medication no biological effect but use for preclude other medication

Placebo

16) alot of competitive and antagonist and physiological please read

17) infection and LA not working why

Unstable at low ph or ionized ? LA doesnt work because there is ionized form. La works in

free or non ionized form.

18) pic of lichen planus , scc, something in base of sinus is root tip or mucous retention cyst ,

classes of ortho , post and getta perca length,

19) least cause of early teeth loss

Lack of water flouride or incomplete tooth development

20) benzodiaspinase. Antagonist

21) after iv diazepum in 15 days got swell and tender in area of puncture why?

22) null hypothesis

23) after tooth exo there is netrophil

Cyst or abcess

24) acute infection has single strand anoerobic or multiple strand anorobic

25) suspecious malignant what type of test

I

Biopsy

pass through buccinator

47 IAN nd lingual for exo of lower 1st pm nd canine

48 u guess the future caries from lower ant fi Alan ala al

Page 129: Decembery RQS – NBDE II

Combined remembered Qs 2020

Remembered questions may 5th-7th

1. Supernumerary teeth: cleidocranial dysplasia

2. Granular tumor most common location : tongue

1. Kid (6ys) has unilateral macroglossia, small pink and gray bumps with fluid in them: I

put granular tumor (other options) lymphangioma

1. 0.7ppm fluoride community

2. 74% u.s has fluoridated water

3. What if the following causes dry mouth and altered taste?

a. Options were HTN drugs (lisinopril, sartan

b. Qsymia(topiramate phentermine) I chose this one) the ot was taking it foe

obesity and I looked it up afterwards and guessed correctly

4. What fracture is most seen caused by a gunshot in face or high velocity impact

accident:

Page 130: Decembery RQS – NBDE II

Combined remembered Qs 2020

a. Simple

b. Greenstick

c. Comminuted (I chose this one)

d. Complex

5. Xray of AOT

6. Inverted Y and what it is composed of

7. X ray of fibrous osseous dysplasia

8. Who most common has an autoimmune disease; middle aged females

Google: Almost all autoimmune diseases affect women more often than men.

9. Pt taking a medication for Alzheimer's. Question asked what does the medication do?

I chose “targets amyloid plaque “

The U.S. Food and Drug Administration (FDA) has approved two types of medications 

— cholinesterase inhibitors (Aricept®, Exelon®, Razadyne®) and memantine 

(Namenda®) — to treat the cognitive symptoms (memory loss, confusion, and 

problems with thinking and reasoning) of Alzheimer's disease.

Page 131: Decembery RQS – NBDE II

Combined remembered Qs 2020

10.Melanoma where is it commonly found? I was between: Gingiva And buccal mucosa

or gingiva and hard palate ( I chose that)

Decks: It exhibits a definite predilection for the palate and the maxillary

gingiva/alveolar ridge.

11.Glass ionomer asked what it does: I said for ionic bond with dentin and enamel- bonds

with tooth structure

12.Asked during a checkup of elderly pts perio health and recession what else do you

check for…I said cervical caries.

13. It asked what is the cut off for a pedo pt ANC? I didn’t know and guessed >1200.

Other optiosn were >1300 >1500 or <1000.

a.

Page 132: Decembery RQS – NBDE II

Combined remembered Qs 2020

14.Hazard something…I chose OSHA.

15.How often do you go over manual something. I said annually.

16.Which has a high fracture strength? I chose zirconi

17. Which has a high c factor? Class 1

18.Picture of class 4 RPD

19.Kid has excessive Over Jet what class division? Class 2 div 1.

20.Kid pic what ia profile ? Convex

21. If a pt wanted to do ortho to align max incisors? I chose lingual bar. Other options was

nance, hawley, RPE

22.Described a ot xray had a beated metal appearance? I said crouzon syndrome

Page 133: Decembery RQS – NBDE II

Combined remembered Qs 2020

23.Pt had a oral white lesion that was present for 7 days and then disappeared. I put

althous ulcers

24.What medication would best treat perio? ….i put tetracycline other options were

amoxicillin, penicillin, cephalexin

25.Pt first visit, 14yrs for checkup and prophy. Before pt informs she is pregnant. What do

you say? These were the options.

a. Inform the pt that you won’t do txt until she tell her parents. b. Inform the pt she will do txt, go over pregnant and advise her to tell her

parents c. Don’t do txt and tell her parents d. Do txt and then tell her parents .

26. Pt had a mo amalgam placed…a few months later restoration fractures. Why?

a. Isthmus was too narrow

b. Restoration at isthmus wasn’t deep enough

c. Didn’t condense well.

d. Didn’t bevel or smooth pulpal/axial line angle.

27. INR to do surgery-I put 2.5

Page 134: Decembery RQS – NBDE II

Combined remembered Qs 2020 28. Which of the following would cause blanching? I pur hemangioma

Important: How to distinguish between a hemangioma and a hematoma — hemangioma will blanch on diascopy, hematomas do not blanch.

29. Xray of genial tubercle

30. X-ray of lateral periodontal cyst

31. X ray of stafne defec t

32. Xray of calcified parotid( other options were ear, hyoid bone, cervical vertebrae) chose

the parotid one because they described it as radiopaque and adjacent to the ramus…it

couldn’t be ear.

Page 135: Decembery RQS – NBDE II

Combined remembered Qs 2020 · 3mm by 3mm pink little thinf above canine I chose papilloma

· Described vertical root fracture and aside to J shape on x ray what else would u

discover; I said isolated deep pocket depth

· X ray of a steep smile. I said pt chin too far down

· Sialothiasis most common in: wharton duct ( dem submandibular gland)

· Pt month ago with bell's palsy: what will happen?

§ Will gradually get better ans restore some function.(I chose this)

§ Will get worse

§ Remain the same.

After its sudden onset, the paralysis begins to subside within 2 or 3 weeks, and gradual,

complete recovery occurs in over 85% of patients.

· What would cause suppuration? I said actinomycosis.

Page 136: Decembery RQS – NBDE II

Combined remembered Qs 2020 · What causes gingival hyperplasia? Verapamil (other options were htn drugs)

· What can you take ti treat xerostomia? Prilocaine

· Firsr sign of multiple myeloma: bone pain

· Non- Vital tooth I said condensing osteitis.

Condensing Osteitis is a diffuse radiopaque lesion representing a localized bony reaction to a low-grade inflammatory stimulus, usually seen at the apex of the tooth. This process is asymptomatic and benign and does not require root canal therapy.

· Best way to take xylitol. I said chewing gum

· Pierre robin syndrome description

· Penicilin: low toxicity

· Description of addisons and most importantly they mentioned pigmentation.

· Nitrous ans pt felt uneasy what to do? 100% oxygen

· If treatment of a pregnant patient, why lay them on the left side to avoid what? IVC

Page 137: Decembery RQS – NBDE II

Combined remembered Qs 2020 · Prevalence definition

· What doea nitroglycerin med do? I said directly cause vascular smooth muscle

relaxation

exert anti-hypertensive effects by causing a direct vasodilator action on the SMOOTH

MUSCLE of arterioles, to produce a decrease in peripheral resistance & BP. Compensatory

responses may be marked such as salt retention and tachycardia.

Adverse Effects: GI upset, headache, dizziness, and tachycardia.

· How would you decrease value? Add complimentary color

• Adding a complimentary color can reduce value. Violet is used on yellow restorations, which has the added effect of mimicking translucency.

• Stains are metallic oxides that fuse to the porcelain during a predetermined firing cycle

• Staining a porcelain restoration will reduce the value (as will using a complementary color). It is

almost impossible to increase the value

· What age range can a kid start to brush their teeth on their own? I put range 6-9.otber

options were 2-5, 11-14.

Page 138: Decembery RQS – NBDE II

Combined remembered Qs 2020

· Xray of pt rct done 8 yrs afo still has a radiolucent small speck. Asymtomatic. I put

apical scar

· Best post. I pur serrated tapered

Parallel-sided posts are preferred over threaded screw posts

· Description of cerebral palsy pt.

Page 139: Decembery RQS – NBDE II

Combined remembered Qs 2020

· Descriptionu of down syndrome pt.

· What is the first thing happens after cleaning teeth? Pellicle formation

· Intra and skin nodules . Pigmentation. Neurofibromatosis.

· Pt 5yr old . avulsed tooth..do not replant

· ASA question on pt was taking nitroglycerin 2 to 3x weekly a yr ago and now only when

he does extraneous exercise. What is he now. I said asa 3 I think.

· Pt. Taking htn drug. After lidocaine was given blood pressure increased. What

adrenwegic recepyoe responsible for such action? Alpha 1 alpha 2. Beta 1 beta 2

Page 140: Decembery RQS – NBDE II

Combined remembered Qs 2020 Alpha 1 with ( lidocaine with epinephrine as vasoconstrictor)

· What is normal range for ped pt bp? I chose 110

Ok that is all I can remember at this time.

Good luck!!

May 15

1. All of the following are considered for a demineralized lesion except:

· Texture

· color

· age

· location

2. Associated with rheumatoid heart

· heart mumurs

· Joint effusion

3. Most important detail that would most affect the outcome of a fracture tooth

Page 141: Decembery RQS – NBDE II

Combined remembered Qs 2020

· 48 hr delay of txt

· Fracture being far away from apical

· Larger than normal pocket

· Infection

4. Non odontogenic max sinus what antibiotic?

· Amox with clavu

5. #1 chemical or ingredient in weed: 2 options ending in anabol

6. Mumps etiology

7. All are bilateral except: sialolithiasis

8. Max molar with divergent roots close to the max sinus what is a method of percaution

you should take for extraction?

· odontosection

· Not use elevator

· Use a small elevator that is not curved

· Open the sinus more

9. Max pm infection goes where? Buccal space

10. Atenololà selective beta blocker

11. anthistamine moa:

· blocks histamine at receptor

Page 142: Decembery RQS – NBDE II

Combined remembered Qs 2020

· physiologic antagonism at histamine receptor

· Prevents histamine to develop (literally exactly same words)

12. What is false for EPT:

· Best test to determine if tooth is necrotic or not

· Helps determine pulp health

13. What is not a competitive antagonist?

· Epi

· Propranolol

· Scopalmine

· Forgot the last option

14. 4 quest of propranolol

15. Nonselective b blocker and selective b blocker..why do we select selective rather than

non selective?

16. Epi+ propranolol= Inc BP and bradycardia

17. According to ADA What is percent of base metal 40% 60% 80%

18. Sickle anemia mutation: missense

Page 143: Decembery RQS – NBDE II

Combined remembered Qs 2020

19. Down syndrome: non disjunction

20. Hepatitis A route of infection: food and drink

21. What does have a dentist have to have in order to not get sued is: competence

22. Nitrous oxide ventilation: niosh

23. Amalgam what do we not do

· Recycle amalgam

· Put it in hazards

· Something about trap

24. In charge of office materials: 1.something agriculture 2.fda (I put this) 3. Osha

25. Why does the body produce ketone bodies?

Page 144: Decembery RQS – NBDE II

Combined remembered Qs 2020

1. Something alcohol was in options

2. Related to gluconeogenesis

3. Related to glucose I forgot

4. Don’t remember last option

26. primary support for max and mand

· Max palatal rugae and mand retromolar pad

· Max Palatal rugae and mand residual ridge

· Max Alveolar ridge and mand retromolar pad

· Alveolar ridges

27. What is the distal border of the denture? (didn’t specify sup or mand)

· Mylohyoid

· Fovea palatini

· Hamular notches (I put this not 100% sure)

· Something else which was wrong about mand

28. Picture of a tooth that was displaced lingually. Question: Tooth is lingually displaced and

present with alveolar fracture. The treatment is to reposition and splint. How long do you

splint it for? 2 weeks, 3 weeks, 4 weeks and 5 weeks (5 weeks bc you treat alveolar fracture

for longer since its of most importance)

Page 145: Decembery RQS – NBDE II

Combined remembered Qs 2020 29. Tension side for orthoà osteoblast

30. 4 year old first appt great oral hygiene what xrays?

· Bitewings based on findings (I put this)

· two Bitewings

· Pano and bitewings

· Pano only

31. Fremitus is checked in which occlusion

· Laterally displaced

· Mesially displaced

· Centric occlusion

· Centric relation

32. Cardiopulmonary resuscitation most common reason why the patient is not getting

oxygen to lungsà

· mask isn’t sealed tightly (this)

· Obstruction

33. Gastric distention whyàtoo much air causing inflation to stomach

34. Infection of max 1st pm where too à buccal space

35. Chronic perio black male

Page 146: Decembery RQS – NBDE II

Combined remembered Qs 2020 36. Trismus what spaceàsublingual, submental, masseteric, temporal (no submand or

pterygomand in options J )

37. Patient felt tingle in his hands (was having MI) what do you do first?

· Establish responsiveness and call emergency

· Call emergency and chest compressions

38. Advantage of indirect over amalgam:

1. better facial contour

2. Better occlusal wear

39. Calculate the amount of epinephrine in 2% lidocaine 1:50,000

A. 34 mg B. 17mg D. 0.17 d.34mcg

40. Calculate negative predictive value (with no calculator so much fun J)

41. Calculate sensitivity

42. What is the most important thing to establish so the patient stops smoking?

· Why the patient wants to stop

· Talk about health benefits of stoping

· Tell patient he is going to die if he doesn’t stop lol

· Talk about what happened last time he tried to stop what triggered

43. What is correct about abuse in eldery

Page 147: Decembery RQS – NBDE II

Combined remembered Qs 2020

· Patients do not report abuser because they have a sense of loyalty to

them

· Most abuse is at healthcare centers

· Oral hygiene in elderly is not good overall

44. Pt had syncope whats the first thing you do:

· Wait and observe

· 100% oxygen

· Inject something saline

· Call 911 (I picked 100% oxygen, there was no tredenburg or position the patient in

options)

45. Doesn’t cause loss of furcation:

· enamel pearls

· Developmental groove (I put this)

· Endo infection

· Something periodontal related

46. Sealants shouldn’t be placed where: high risk patients, low risk patients

47. Topical Antibiotic supragingivally placed to reduce bleeding and plaque (true or false)

part 2 of same question: topical supragingival is use as an adjunctive therapy at home with

treatment for periodontitis cases. (true or false)

Page 148: Decembery RQS – NBDE II

Combined remembered Qs 2020

48. All are signs of corticosteroids use except: Mental psychosis, inc resistance to infection

49. You need to know the exact correlation of nutrition and perio. There was 2 true of false

question on this.

Pictures from day 1:

1. Similar

Page 149: Decembery RQS – NBDE II

Combined remembered Qs 2020

2.

Same exact pic. Hemangioma

3.

1. Characteristic amoxicillin which one: low toxicity or broad spectrum

Page 150: Decembery RQS – NBDE II

Combined remembered Qs 2020 2. Know your cidal and static antibiotic. Easy questions about except that if you know them youll

get them right. Super easy.

3. Best test for sympt apical perioà percussion

4. Taking impression inflammationà angioedema

5. Hypochlorite accident all except

1. Pain

2. paresthesia

6. Patient management question of what is a cultural advisor

· Help people feel at home from different cultures

· Advise patients about different cultures

· Establish different txts according to culture

· Don’t remember

7. EPTà a delta

8. What can help you see significance of patient bleeding?

· Pt

· Inr

· Bleeding time

· History

9. Least inflammation associated with?

Page 151: Decembery RQS – NBDE II

Combined remembered Qs 2020

· Chronic inflammation

· Atrophic gingiva

· Aggressive periodontitis

· Desquamative periodontitis

10. A question on independent variable and dependent variable

11. Reason why implanted teeth fail?

· Replacement resorption

· Inflammatory resorption

12. Most common reason for early exfoliation of primary canine

· Trauma

· Caries

· Root resorption (In asda its arch length discrepancy so I think its this)

· Anterior cross bite

13. 2 questions on calcific metamorphisis

14. Febrile most common in kids

15. 22 years old with fever , malaise, lymphadenopathy, multiple ulcers on

tongue, palate with swollen gingiva

1 anug

Page 152: Decembery RQS – NBDE II

Combined remembered Qs 2020 2 acute herpetic gingivostomatitis

3 marginal gingivitis

4 herpangina

16. Not in boneà nasolabial

17. Most associated with retention of crown

· Axial Taper (NOOOO)

· Surface area yes J

18. Retention of class 3 comp

19. Retention of class 5 composite

20. What do inlay and amalgam prep have in common?

21. Sjorgen all except: bilateral parotid enlargement

22. 3 years old comes to have what do you see:

· Plaque and pellicle

· Nothing

· Bacteria

· Nasthmiyh (idk how to spell it)

23. Serotonin syndrome= ssri+ opiod

Page 153: Decembery RQS – NBDE II

Combined remembered Qs 2020

24. same question like ASDA

1. Weird question and options were lichen planus, goid, gus. Didn’t mention acantholysis, basement

membrane of anything like that in decks. No clue (read about it)

2. Best to compare internal and root resorptionà pink color, rdx in different angles

3. All of these reasons are why you keep mand 2molar in mouth that’s ankylosed as long as possible

except:

· Mesialization of per 1m

· Dista of 1pm

· Supraeruption of opposing tooth

· Maintain bone width

4. Pt tmj pain when opens and moves to right the jaw what do u do:

· Occlusal equilibriam

· Ortho

· Occlusal splint

Page 154: Decembery RQS – NBDE II

Combined remembered Qs 2020

· Resposition splint

· Inject botu

5. What resembles epiphyseal plate? Synchondrosis

6. Patient has came with red erosive like gingiva what do you NOT do as initial txt:

· Biopsy

· Corticosteroids

· OH

· Prophy

7. Secondary hyperparathyroidism: renal failure

8. 80 yr old you inject 6 carpules of anesthesia (didn’t say if with epi or not). She starts talking a

lot, figity and tingiling of fingers. What is it? (ALL THE INFO THEY GAVE)

· Hyper (don’t remember if it was tension or ventilation)

· Lido toxicity

· Lido allergy

· Anxious

9. Enamel pearls what stage

10. Cracked tooth most common sign

· Cold

Page 155: Decembery RQS – NBDE II

Combined remembered Qs 2020

· Pain on bitting and release

11. You did exfoliative biopsy and came positive with dysplasic cells what do you do next:

· Confirm with another exfoliative biopsy

· Incisional biopsy

· Excisional biopsy

· Wait

12. most important in denture:

· support and rigidity

· retention and rigidity

· rest were wrong

13. Least one that makes you drowsy out of all of these: the awnser was a 2nd generation

antihistamine: Loratadine

14. Questions of antibiotics all are true EXCEPT. I did not know which one is true but I knew my

static and cidal and that was enough to know the exception

15. Xylitol best inà gum

16. internal void and surface defects = 1) leads to stress relaxation 2) decreases elastic

modulus (I put this) 3) increases translucency

17. Teeth respond to thermal, rest test are norma. You see mix RL and RO lesions in anteriorà

cemento dysplasia

18. Which disease is recently seen most with periodontitis in the past years

Page 156: Decembery RQS – NBDE II

Combined remembered Qs 2020

· Hyperthyroidism

· Cardiovascular disease (I put this)

19. Recent tests have shown that out of the following one is the most effective:

· Apf 1.23% for 25 secs (25 secs is wrong)

· Fluoride mouthwash

· Sodium fluoride gel tray for 1 min (I put this)

· Fluoride varnish

20. Patient has asymptomatic white lesion on cervical, not cavitated what do you do?

· Periodic evaluation

· Fluoride placement

· Remove and gic

· Remove and amalgam

21. Nsaid decrease the effect of thiazide, know this fact for a question

22. Most radioresistant

· Striated muscle(I put this)

· Nerve

23. The following when checked orally has one tooth less present in mouth:

· Germination

Page 157: Decembery RQS – NBDE II

Combined remembered Qs 2020

· Fusion

24. Same exact question

25. Patient is anxious coming to appointment what is the least invasive treatment that you can do?

· Sit the patient next to a window with a nice painting lol

· Prescribe meds

· Nitrous oxide

· Let the patient talk (Picked this)

26. What type of xray is most effectiveà short wavelength and energy

Page 158: Decembery RQS – NBDE II

Combined remembered Qs 2020 27. Mandibular denture patient complains of pain by ridge:

· Traumatic neuroma

28. All of the following are advantages of ¾ crown over full crown except:

· Retention

29. Modified ¾ crown can be used forà

· Lingually tipped mandibular molars

30. To make sure casting seats, do the following EXCEPT:

• Increase thermal expansion of investment

• Mix cement thin

• Remove internal nodule with occlude

31. Best area for successful placement of implant: ant mand

32. Implant implant distance: 3mm

33. Ace inhibitors cause HYPERkalemia, thiazide cause HYPOkalemia

34. Abx+ surgery= LAP

35. Drug conjugation what happens: More water soluble, more polar, less lipid soluble. It was choice

of 3’s

36. Pregnat needs to go to the bathroom a lotàFetus pressue

37. Pt has hba1c 6.3% and taking Lasix why does the patient go to the bathroom a lot

· Diabetic

Page 159: Decembery RQS – NBDE II

Combined remembered Qs 2020

· Medication (this, less than 6.5 to 6.0 is prediabetic, Lasix makes you pee)

38. Advantages of screw vs cemented

Screw retained Cement retained

· Retrievability allows for crown

removal, facilitating maintenance

(e.g., soft tissue evaluation, calculus

removal).  

· Future modification capability.

 

· Access hole is through the

occlusal table of posterior teeth or

lingual of anterior.  

· Main disadvantage is that the

screw may loosen during function

because of excessive lateral forces,

excessive cantilever force, or

improperly screwed crowns.  

· Screw retained can be shorter

and take up less space

· Takes up more space

· More economical  

· Allows minor angle

corrections to compensate for

discrepancies between the implant

inclination and the facial crown

contour.  

· Easier to use in small teeth

than screw-retained implant

crown.  

· Requires more chair time and

has the same propensity to loosen.

 

39. Bacteria 3 days: gram +

40. Psychotics stay how long in bodyà 5-6 days

Page 160: Decembery RQS – NBDE II

Combined remembered Qs 2020 41. Sign of successful endoà no dentin regeneration

42. First thing you do when you reline a dentureà try on framework

43. Parkinsonà have mask facies

44. Gracey has one cutting edge, it is NOT triangular

45. Insertion of gracey angle à 0

46. Amalgam pin all except

· The bigger the pin the better the retention

· The bigger the pin the more strength of amalgam

· 0.5 axial wall

· 1 pin per missing line angle

47. They asked me like 3 questions of hybrid, microhybrid, and microfill

· Basically know that hybrid not good with esthetics and that microfill not good

with occlusal wear and strength.

48. Another question of particles and what does it mean in terms of strength, the more = what, or the

lest =what

49. What is the most common etiology of periodontitis

· Plaque

· Calculus

· Systemic diseases

Page 161: Decembery RQS – NBDE II

Combined remembered Qs 2020 50. True or false question. Went something like this: Nutrional and systemic diseases have been seen

to be associated with periodontitis because of these local factors …idk I forgot the rest.

51. Bisphosphonates affectà osteoclast

52. Addisonà hypotension

53. Patient opens his mouth all the way and it deviates to the left side what is the problem and what

movement:

· Left side: rotation

· Left side translation

· Right side: rotation

· Left side: translation

54. Early sign of syncope: pallor

55. All are true of efficacy except:

· Refers to the attractiveness of the drug to its receptor

56. Sickel cell anemia comes in for extraction of 3rd molar that is indicated for extraction (has pain).

All are indication for this patient except?

· Nitrous

· Local anesthesia

· Systemic antibiotics (why give systemic antibiotics to a regular extraction? Patient

doesn’t need it, abx prophy isn’t indicated here and question didn’t state there was any

infection associated with it)

Page 162: Decembery RQS – NBDE II

Combined remembered Qs 2020

· Extraction and in histo was neutrophilsà abscess

57. Definition of nonmalefecience: do no harm

58. Dentinogenesis which stage: bell

59. Best for molding masseter: tell pt to bite down

60. Old patient comes to your office for maintanence what is something to keep on eye on to catch

one time

· Cervical caries

61. Primer all except: I don’t remember options but it was something along these lines

· It said hydrophobic/hydrophilic don’t remember which

· Something about including the smear layer or not (I think I picked this one)

· Rest I don’t remember

62. What is important in single implant tooth replacement: smooth interface, connecting the implant

to neighbouring teeth, broad contact with neighbouring teeth, I put antirotational something

63. MOA of ibuprofen

· Extrinsic pathway

· Intrinsic pathway

· Irrversible on platelet

· Reversible on platelet

64. Acron vs non acron= mand movement

Page 163: Decembery RQS – NBDE II

Combined remembered Qs 2020 65. Eldery man losing it but sometimes mentally stable, son takes care of him. He can decide for him

when he has power of attorney

66. Drug with good first pass effect: low bioavailability

67. All are anticholinergics except

68. All are immunosuppressive drugs except

69. What causes the least buccal-lingual resistance to lateral forces? Same exact rq, nothing missing

in this rq..its completely correct

A- Two 5mm diameter splinted implants

B- Two 4mm diameter splinted implants

C- One 5mm diameter implant

D- One 4mm diameter implant (I put this)

70. Levadopa mode of action:

· Blocks dopamine

· Replenish dopa

71. Endo was easy, don’t waste time studying it

72. Doing endo txt gutta-percha extruded what do you doà Observe

73. Dry socket all exceptà Abx

74. 1st, 2nd and 3rd

· 1st is fl

Page 164: Decembery RQS – NBDE II

Combined remembered Qs 2020

· 2nd is md

· 3rd is torque

75. Which is a true cyst: dermoid

76. why is important to rule out okc

a) due to potential for malignancy b) asso with nevoid basal

cell carcinoma c) recurrence rate d) infection

77. Green and oral stains: poor oral hygiene

78. Does not affect denture à buccinator

79. A question of ending of calcification of 1st molar

80. Most associated with candidiasis:

· Insufficient radiation

· Chemotherapy

81. I got ZERO questions on flaps. Still surprised.

82. Most common tooth to get blocked of out mand archà 2nd molar

83. You do a flap on complete mandible, what nerve is most likely to get damagedà mental

84. BSSOà inf alveolar nerve most common to get damaged

85. Caries in radiotherapy patients MOSTLY in

· 1 cervical

Page 165: Decembery RQS – NBDE II

Combined remembered Qs 2020 · 2 occlusal

· 3 proximal

· 4 All

86. Hardest to maintain:

· Mand 2m in 7 year old

· Mand 2m in 5

87. All are part of infraorbital except: it’s a rq look for it same one

88. I got a question of stages of gingivitis and the cells of each

1. Amalgam breaksà not deep enough

Page 166: Decembery RQS – NBDE II

Combined remembered Qs 2020 2. Erythroplakiaà dysplasia

3. Initiation of cariesà strep

4. Not initiation but developmentà Lacto (Are these like bible questions that always go?)

5. Hairy tongueà filiform

6. What age is commonly affected with Primary herpetic gingivostomatitis?

A- 1 B- 2 C- 6 D- 9

7. Osteoporosis seenà Thin trabecule

8. Delay txt of temporalà vision loss

9. Verrucous carcinomaà Warty

10. Combination syndrome all exceptà inc vdo

11. Screened annually forà tb

12. Injected didn’t work went to à mylohyoid

13. 2 questions on SLOB

14. Gingivectomy mostly forà suprabony pockets

15. Minimally attached gingivaà no gingivectomy

16. Mean boys and girlsà t test

17. Wax try in all exceptà obtain facebow

18. Chronic pericoronitis seen in xrayà flame shape distal to third molar

Page 167: Decembery RQS – NBDE II

Combined remembered Qs 2020 19. Biological widthà 2mm junctional and ct

20. Initiator of acrylicà Benz by tertiary amine

21. Trephinationà Penetration of bone

22. Levoà alpha 1

23. Rest of pharm was more towards what the medication causes so know the side effect s

Day 2: More challenging. You cant see pictures at all.

Case 1:

Patient diabetic, hypertensive, case said he wasn’t controlled since he hadn’t gone to the doctor in

several years. He had a removable partial denture in superior from lateral to lateral. He came with

Page 168: Decembery RQS – NBDE II

Combined remembered Qs 2020 chief complaint that food was going between his denture and he had bad breath. He had so many

cervical caries…I would say about 10 cervical caries…especially in superior. NOTE: missing anterior

teeth (lateral to lateral) had significant bone loss and great depression.

Question #1: Pic that they presented was something like this only on anterior though since he was

missing anterior teeth…it was a class kennedy 3 division 1. It had some red in the space of 1 of the

endetulous portions of the missing teeth. The red outline was less intense than this picture and had the

same outline as the partial.

What is the reason why patient presents this?

1. Candida (I chose this, same description as denture stomatitis)

2. Food impaction

3. Allergy to material

4. Some other crazy answer not applicable

Page 169: Decembery RQS – NBDE II

Combined remembered Qs 2020

Question #2:

What is NOT of importance when placing implants in anterior missing teeth:

1. Bone loss

2. Systemic condition

3. Caries hygiene

4. Palatal gingiva

Question 3:

They asked me what classification the patient was: Class 3 modification 1

Page 170: Decembery RQS – NBDE II

Combined remembered Qs 2020 Question 4:

They asked me what was the RL inside the canal of superior canine. They said: look at both

radiographic images taken from 2 different angles.

1. Internal resorption

2. Occlusal caries

3. Smooth surface caries

4. Amalgam

When you see it initially it looked like an internal resorption but when you see the other xray the RL

moved places. Internal resorption does NOT move with xray it stays in place. Patient did not present

with caries in occlusal on clinical picture nor on dentigram…but did have a huge cervical caries which

was seen on dentigram and clinically so I picked that.

Case 3: I don’t remember this one just remember the patient was a COPD patient. What ASA

classification is he? 3

Page 171: Decembery RQS – NBDE II

Combined remembered Qs 2020

Case 4: Ortho case a girl 10 years old. Her canines were impacted.

All will be included in the ortho informed consent except A. Ortho treatment can bend roots of the

teeth B. Caries and gums dieses can happen during ortho treatment

Why is her maxillary deviated to the right?

1. Not enough space in anterior (this was answer)

What Is the RO line that crosses her max incisors in cephalon? Palatine process

Ruler? Magnification

They asked me which of these has the highest certainty:

2. Randomized clinical trial

3. Case control

4. Case series

5. Another weird one I have never seen (it was not Meta analysis)

Page 172: Decembery RQS – NBDE II

Combined remembered Qs 2020

Her profile according to cephalo? Convex

What occlusion is she? Class 2

Case #5: this was short

She is epileptic. Patient takes Dilatin and a thiazide. Chief complaint: I don’t like my teeth they look

short. (How about your gums lady?) The picture looked like this but less severe

All of the following are treatment options except:

· Crowns

· Gingivectomy

· Gingivoplasty

· Tell patient to stop dilatin immediately (you are suppose to contact physician..shes epileptic

you cant remove the medication…only speak to physician to change it)

All are symptoms histologically of this patient except:

· Acanthylosis

Page 173: Decembery RQS – NBDE II

Combined remembered Qs 2020

Case #6: pt only had 3 teeth left in mouth. Came for a total denture.

Question:

You will place implants on anterior for more retention all are indication except:

· Sinus lift in ant

All of the following is true except:

· A partial denture will have same retention as implant

Miscellaneous day 2:

Page 174: Decembery RQS – NBDE II

Combined remembered Qs 2020 Endocrine stored? Ant pituitary

All are treatment options for this patient except: full mouth srps (majority of her probing was 2’s why

would you do that)

Ant 1/3 of tongue taste which nerve

Post 2/3 of tongue sensation which nerve

Which has the most difficult prognosis long term:

· Mand molar with class 2 furcation

· Other mand molar with probing of 5 and 6

· Tooth with mobility

· Don’t remember last option

There was a case that there was brown pigmentation of the gingiva and it said which of the following

medication cause this: no clue.

Page 175: Decembery RQS – NBDE II

Combined remembered Qs 2020

May 20

-Mmm I felt it strong in pathology And pharma 1. Why is Atropine good for Pre anesthesia? Vagal reflex inhibition

a. Atropine-a powerful anticholinergic that blocks saliva production in the salivary

glands. Other anticholinergics have a similar action.

2. Reason for Amalgam fractured at the isthmus 6 week after placement

3. Keratocyst

4. Nevoid basall cell sx

Page 176: Decembery RQS – NBDE II

Combined remembered Qs 2020

5. Gingivectomy contraindications = esthetics

a. Contraindications: infrabony pockets (defects) and lack of attached tissue.

Limitations include compromised esthetics with longer teeth, lack of access to

bony defects, and having a broad, open wound post-surgically.

6. Best way to avoid fracture In amalgam MoD or something =Beveling axiopulpal

angle

7. Alpha 1 receptor action = vasoconstriction

8. Gingival hyperplasia med And wasn’t there phenytoin calcium Blochers or cyclosporin.

I pick carbamazepine

9. Iv drug for ventricular arrythmia= lidocaine

10.Drug for lidocaine overdose = Diazepam

11.Why Acetaminophen and steroids work well together?I pick different MOA

a. Steroid control inflammation n pain

12.Best prognosis for perio-endo, endo-perio lesons primary endo or primary perio. I pick

perio primary

13.Mand molar after rct with radiolucency in the furcation whst to do? Hemisection, Root

amputation, Bone graft.

14.Kid showing signs of abuse what you do. Call local police. Talk to parent. Call to

correspondient agency

Page 177: Decembery RQS – NBDE II

Combined remembered Qs 2020

15.Patient 14 years old pregnant Tells you to not tell to their parents What you do?

16.Best treatment for root sensitivity. Plaque control

17.Days to suspend warfarin in a patien for extractions. 2 days in Mastery

18.Carbidopa moa = Peripheral inhibitory

a. Carbidopa-a drug used to treat PARKINSON'S DISEASE, but only works when

combined with LEVODOPA (treats Parkinson's Disease to replenish the brain's

supply of dopamine, which is the deficient neurotransmitter in Parkinson's.

Levodopa (L-dopa, Dopar)-in combination with carbidopa, is the precursor of

dopamine, and MAIN TREATMENT for Parkinson's Disease. Administered with

carbidopa to increase its effectiveness and reduce adverse effects.

19.Non inhibition of wall synthesis = azithromycin

20.Best flap for visualization of apex = trapezoidal

Page 178: Decembery RQS – NBDE II

Combined remembered Qs 2020

Page 179: Decembery RQS – NBDE II

Combined remembered Qs 2020

21.Srp indication =I pick >4 pockets

22.Who Invented the universal standard care something like that = Cdc

23.Opoid most common side effect = Constipation

a. drowsiness, cns depression, miosis, euphoria, dysphoria, dry mouth…

Page 180: Decembery RQS – NBDE II

Combined remembered Qs 2020

MAY 21 Had around 5 questions about diagnosing pulpal and PDL health on day one. Almost all

cases on day two gave you information on diagnosing correctly a tooth. So my advice is

know the differences between every type of pulpitis and periodontitis. It’s not difficult to know

the answer. CROSS OUT the answers you know aren’t true and you will usually only be left

with one. It’s not about having all the characteristics of the disease it’s about being the

closest to it. So please relax, read the questions well and a simple process of elimination

goes a long way.

They throw in a few occlusion questions that are somewhat confusing but don’t mind them, it

is just about 4 questions out of 500 so mark it and come back to it and give it your best guess

if anything. Lol

The files I recommend to read is caterpillar, Dan man (make sure it’s a corrected version, the

first one I got was full of mistakes) and patient management file (there is two of them one has

more information). But definitely pick your source carefully, that is the base of your prep and

you need it if you want to understand the questions given and correlate the answers.

PS: these questions are going to be worded either the same or differently but they are all

going to ask the same thing!

GOOD LUCK GUYS!

1. Bacteria that secretes collagenase, hemolysin for periodontal breakdown? - Porymorpha

other choices: Streptococus, lactobacillus.

2. Virus that causes Oral Hairy Leukoplakia: Answer?

- EBV; other choices included HPV candida and something else.

3. All are side effects of nitroglycerin?

- Headaches, lightheadedness, flushing…. Etc.

4. What immunoglobin is dominant in anaphylaxis?

- IgE (allerGE)

5. Antihistamines effects against histamine?

- Competitive antagonist

Page 181: Decembery RQS – NBDE II

Combined remembered Qs 2020 6. All these drugs can cause gingival hyperplasia except?

- Benzodiazepine. (cyclosporine, CCB and phenytoin)

7. A question was about different types of bone lesions, which one wasn’t caused by

inflammation, trauma or infection:

- Options included osteomyelitis and condensing osteitis. Don’t remember what I had put or

the last two options. (Danman file has similar question #17, it says idiopathic osteosclerosis)

8. What bone lesion is most susceptible to malignant transformation?

- Pagets disease.

9. Mechanism of NSAIDS:

- Inhibit COX.

10. Oligodontia/anadontia is common in: (list of syndromes) answer:

- Ectodermal dysplasia.

11. Had 2 questions asking about what an antagonists are characterized for:

- High affinity, no intrinsic activity.

12. What type of gold is used for a RPD? type IV

13. Advantage of penicillin:

a) low toxicity

b) high price

c) low spectrum

14. What lesion is not found in bone?

a. Nasolabial cyst

b. Others I don’t remember lol

15. Going from an 8” pid to 16” what happens to the intensity of the xray?

a. Stays the same

b. Its ½th strength

c. Its ¼th strength

d. 1/8th the strength.

16. How to reduce radiation?

a. Cone collimation

Page 182: Decembery RQS – NBDE II

Combined remembered Qs 2020

b. Cylinder collimation

c. Rectangular collimation

17. Function of the collimation in the radiograph apparatus?

- Restricts the dimensions of the x-ray beams.

18. What is the spenoccipital syndochondrosis?

a. Cartilage

b. Fibrous suture (I put this one I wasn’t sure tbh)

c. Intramembranous bone

19. Common location of supernumerary teeth:

A. Max posterior

B. Man anterior

C. Man anterior

D. Max anterior

20. Developmental anomaly of supernumerary teeth occurs in which stage of tooth

development :

A. Initiation

B. Apposition

C. Bell stage

21. Sialolithiasis is common in = Wharton's duct

22. First step in preparing an abutment tooth for an RPD?

- Preparing guide planes.

23. Most danger for an operator during xrays?

a. Scatter from xray head

b. 2a scatter from patient

24. Intrusion of primary central of 5mm in a 5 year old boy how to treat?

a. Adjust the tooth and splint

b. Extract to prevent damage to permanent

c. Ortho

25. The most important factor for reducing sensitivity after periodontal treatment?

Page 183: Decembery RQS – NBDE II

Combined remembered Qs 2020 - Plaque control

26. How long to splint an apical root fracture?

- 7-10 days

27. Where you cannot place an apical positioning graft?

- palate

28. Ectopic canines in a young patient you most commonly see?

- Gingival recession

29. During closing of the mandible, what is least important?

- Relaxation of lateral pterygoid.

30. What reflex is the most important to maintain during conscious sedation?

- Verbal

31. Percussion test tells you what?

- Pdl inflammation.

32. Class III composite with brown borders what to do?

- Replace composite

33. 1 week old composite with staining on the margins but with good internal seal?

- Remove 1 mm of composite and place more composite.

34. Primary central intruded 5 mm in a 5 year old patient what to do?

a. Reposition and splint.

b. Leave alone and allow for it to reerupt.

c. Extraction to avoid damage to permanent tooth. (I picked this one)

35. Avulsed primary tooth what to do?

a. Replant

b. No need to replant.

36. Whats the most important factor of an avulsed permanent tooth?

a. Storage medium

b. Debridement and rinse of socket

c. Time

37. Common feature of Cleidocranial dysplasia?

Page 184: Decembery RQS – NBDE II

Combined remembered Qs 2020 - Delayed eruption and supernumerary teeth.

38. Horizontal mid root fracture?

- Spline and recall.

39. Mandibular root is fractured during extraction what to use?

- East west (cryer) elevator.

40. When preparing a cast post what is the function of the key way?

- Anti-rotation.

41. Which teeth can be successfully extracted with and IAN and lingual N block?

- 1 pm and canine on the same side.

42. How to treat a fearful child?

- Modeling using sibling as an example.

43. Dentist ignoring unacceptable behavior?

- Extinction.

44. Telling a patient to replace amalgams due to toxicity of mercury is breaking what code of

ethics?

- Veracity.

45. Informed consent is a type of?

- Autonomy

46. What medicine to avoid with… (here they can place any of the natural anticoagulants)

ginger, gensing, ginkgo biloba, turmeric etc. ? they asked 2 different questions about the

same thing with different answer choices provided below.

- Aspirin or clopidrogel

47. Who’s most affected by chronic periodontitis?

- African American males

48. Modified Widman flap is a?

- Full mucoperiosteal flap coronal to the mucogingival junction.

49. Periodontitis least helped with antibiotic therapy?

- Chronic periodontitis.

50. Systemic or local antibiotic and SRP are combined treatments for?

Page 185: Decembery RQS – NBDE II

Combined remembered Qs 2020 - Aggressive periodontitis.

51. Medication of choice in status epilepticus in the dental chair?

- Any benzodiazepine especially midazolam, diazepam or lorazepam would be the answer

choice.

52. Osteomas, unerupted teeth, retained primary teeth (x ray) are signs of?

- Gardener’s syndrome.

53. What bones are underdeveloped or missing in cleidocranial dysplasia?

- Clavicle

54. Pigmentation of the lips and oral cavity plus intestinal polyps are signs of?

- Peutz Jeghers syndrome.

55. Dementia is characterized by?

- Lack of short term memory retention (short term memory loss same thing)

56. Longest acting local anesthetic in IAN block?

- Bupivacaine.

57. Which of the following dental conditions is often found in Down's syndrome patients?

A. Mulberry molars

B. Supernumeraries

C. Short, conical roots

D. Hutchinson's incisors

58. To avoid porcelain fractures in PFM the junction to metal should be?

- Right angle (90°)

59. Maxillary Premolar extraction forceps?

- 150

60. Mandibular Premolar extraction forceps?

- 151

61. Method of action of doxycycline 20 mg?

- Inhibit host response or collagenase.

62. Amalgam margins over time?

- Improves seal

Page 186: Decembery RQS – NBDE II

Combined remembered Qs 2020 63. What preparation step in amalgam offers resistance to fracture during compression?

- Rounded or beveled axial pulpal line angle.

64. With no barrier the person taking the xrays should be where?

- 6 feet away and 90-135° from the original beam.

65. Increase in the Kvp (voltage) of the xray will produce?

- Short wavelength and high energy beam.

66. Some question about a patient moving for a second during the panoramic imaging?

- It was talking about the focal trough and how it would be distorted.

67. Best radiograph image to view condyle fractures?

- Submentonvertix

68. Waters are used to view all these sinuses except?

- Sphenoidal sinuses

- The other choices were: maxillary, ethmoidal and frontal sinuses.

69. Radiographs to confirm a horizontal root fracture?

- 3 PA’s and 1 occlusal.

70. Ahh the scarring from the electrical burn victim question always asked. Scarring from an

electrical burn in a developing child will?

- Decrease VDO

71. A pregnant woman faints what do you do?

- Left lateral decubitus (turn on left side)

72. Local anesthetic safe to use in pregnant patients?

- Lidocaine

73. Best trimester to treat a pregnant patient?

- 2nd trimester (only do simple things like prophylaxis, debridement and emergencies,

elective treatments should be postponed)

74. Cleft lip occurs in what weeks during intrauterine?

- 6-9 weeks (I know I know the study material says other dates but that’s how its worded in all

the exams other choices were before 5 months or after 10 months)

75. Cleft lip more common in?

Page 187: Decembery RQS – NBDE II

Combined remembered Qs 2020 - Males

- Cleft palate alone is more common in females.

76. Internal bleaching causes?

- None of the answers said cervical resorption so the closest to it was internal resorption.

(didn’t specify root resorption).

77. A dentist did a study of caries in patients and found that the study was faulty at predicting

the disease so the study is considered?

- Low sensitivity (something like that, they are trying to confuse you by making it seem like

they are talking about sensitivity but read the questions well)

78. When receiving a pfm crown from the lab whats the first step?

- Check the color of the crown!

79. First step in seating a crown?

- Check proximal contacts.

80. Following up on a patient with SRPs and still has pockets of 5 mm what to do next?

- Open debridement.

81. Patient presents with grade 3 furcation involvement in a tooth with no additional apparent

bone loss what to do?

- Hemisection.

82. Discoloration 1 week after placing a veneer, what could be the cause?

- Microleakage.

83. Green discoloration of porcelain is caused by?

- Silver

84. All are effects of opioid except?

- Diuresis

85. Gustatory sweating?

- Frey’s syndrome

86. LD50 and ED 50 is the?

- Therapeutic index.

87. Slight anterior crowding and canines are erupting what to do?

Page 188: Decembery RQS – NBDE II

Combined remembered Qs 2020 - Stripping (enamel disking)

88. Best antibiotic to give a patient who is allergic to penicillin for endocarditis?

- Clindamycin 600 mg (child 20 mg/kg)

- If not there then azithromycin 500 mg (child 15 mg/kg)

89. Diazepam antagonist?

- Flumazenil.

90. Morphine overdose antidote?

- Naloxone.

91. New patient taking amantadine, what class of drug is it?

- Antiviral.

92. 7 year old boy, new patient walks into your office and is taking amphetamine, what is the

most likely diagnosis of the disease?

- ADHD (attention deficit hyperactive disorder)

93. Best analgesic to give a pregnant patient?

- Acetaminophen.

94. Picture of a sinus tract and a radiograph with a periapical radiolucency, what’s the lesion

on the gingiva?

- Parulis (know this term, it’s an old term for sinus tract)

95. Not a question but rather know the reason they use epinephrine/vasoconstrictors in local

anesthetics, they had a question ask all are desirable effects of the vasoconstrictor in

lidocaine except?

96. Compression rate in an adult?

- 120 x min

97. Gingival index is considered?

- Ordinal

98. Which interleukin has been found recently associated with periodontal loss?

- IL 1

99. Minimum amount of tissue needed for a 4 mm width implant?

- 7 mm (no answers said 6)

Page 189: Decembery RQS – NBDE II

Combined remembered Qs 2020 100. Critical pH for demineralization of enamel?

- 5.5

101. Critical pH for demineralization of flour remineralized enamel?

- 4.5

102. White spot on enamel, fades when moist?

- Incipient carious lesion.

103. Cheek biting in denture when?

- Inadequate horizontal overlap in molars.

104. How to correct cheek biting in a denture?

- Increase horizontal overlap in molars.

105. Kennedy class III gets support from?

- Occlusal rests (tooth borne device)

106. Thumb sucking patient has a fixed oral appliance placed, this is considered?

- Positive punishment.

107. Axial walls in MOD cavity for in cast onlay?

- Convergence towards pulpal floor.

108. Best type of allograft material?

- FDBA

109. Maximum concentration of nitrous oxide in children?

- 50 is absolute max; 30 was the answer given.

110. Cariogenic bacteria are found in?

- Non shedding surface.

111. Most common dental emergency?

- Vasovagal syncope.

112. Maximum amount of carpules for a cardiac patient with lidocaine 2% 1:50 000

- 1 (1:50 000 = .036 mg) max dose is .04

113. Oxygen is administered in all dental situations except?

- Hyperventilation syndrome.

114. Opioid overdose:

Page 190: Decembery RQS – NBDE II

Combined remembered Qs 2020 - Miosis!

115. Wavelength of color?

- Hue.

116. Prevalence of cleft lip in USA?

- 1/700

117. Patient presents with flabby and mobile anterior ridge in maxilla, how to take impression

for new dentures?

- Passive

118. Vertical angulation in radiograph produces?

- Elongation or foreshortening

- Memorize one of them and the other is the opposite.

119. Chemical disintegration of enamel?

- Erosion.

120. Reason for taking shade of tooth before placing rubber dam?

- Dehydration of tooth.

121. Irreversible index?

- DMFT

122. Most common site and cause of mucocele?

- Lip and trauma. (separate questions)

123. The mandibular division of the trigeminal nerve passes through what orifice in the skull?

- Foramen ovale.

124. Dens in dente is most commonly found in what tooth?

- Maxillary lateral incisor.

125. Property of a drug to be able to increases tendency to cross membranes?

- Non ionized/ high lipid solubility.

126. Most common cyst of the oral cavity?

- Radicular cyst

127. Strawberry tongue, fungiform papillae involvement?

- Scarlett fever

Page 191: Decembery RQS – NBDE II

Combined remembered Qs 2020 128. Fluoride to use in a patient with implants and porcelain?

- Sodium fluoride.

129. Treatment of choice of posterior crossbite in mix dentition patients?

- Fixed maxillary expansion devices.

130. Lingual flange determinant of the mandibular denture?

- Mylohyoid muscle

131. Distal extension of a mandibular denture is determined by the action of what muscle?

- Masseter.

132. Most common donor site for a gingival/ CT graft?

- Palate

133. Patient has endo lesion and periodontal involvement what is the correct order of

treatment?

- Endo first perio second.

134. The use of lateral positioning grafts (pedicle graft) is limited by?

- The amount of attached gingiva from adjacent tissues.

135. Gingival attachment of implants are given done by?

- Hemidesmosomes

136. Type of osteointegration for an implant?

- Osseous

137. The sulcular epithelium acts as a semi permeable membrane through which?

- Fluids from the gingiva seeps into the sulcus.

138. Retention for a cast crown is given primarily by?

- Axial taper

139. Calcification of permanent 1st molar occurs?

- At birth

140. MOA of carious detection dyes?

- Stains infected dentin.

141. The function of occlusal rest?

- Support

Page 192: Decembery RQS – NBDE II

Combined remembered Qs 2020 142. The depth of an occlusal rest in posterior teeth is?

- 1.5 At the marginal ridge and deeper into the fossa.

143. Treatment of choice for trigeminal neuralgia?

- Carbamazepine.

144. Pulp test for a tooth with a crown?

- Thermal testing

145. Premolar most likely with 3 canals?

- Maxillary 1 PM

146. Patient denies having an addiction to smoking, is considered to be in what phase?

- Precontemplation.

147. A graft from another species (pig) is considered a?

- Xenograft

148. Patient taking anticoagulants needs an extraction, how to control dosage?

- Consult with primary physician about altering patient dosage.

149. Occupational max dose of radiation per year?

- 0.05 SV or 50 mSV

150. Blue sclera, osteogenesis imperfecta and obliterated pulp canals associated with what

disease?

- Dentinogenesis imperfecta.

151. Best pulp test for newly erupted tooth?

- Cold test

152. Caries zone of enamel appears on xray is what zone?

- Body

153. Concentration of acidulated fluoride?

- 1.23%

154. What type of material is the receptor of the anode in the xray machine made of?

- Tungsten

155. Edentulous patient with candida treatment?

- Nystatin

Page 193: Decembery RQS – NBDE II

Combined remembered Qs 2020 156. What property of color increases with age?

- Chroma

157. Space for 2 and 3rd mandibular molars is done by?

- Bone resorption of anterior ramus.

158. Primate spaces in maxilla?

- Canine and lateral incisor (mesial of canine)

- Mandibular (distal of canine)

159. 2 patients with same amount of bone loss from periodontal disease, who has a better

prognosis?

- Older patient.

160. Treatment of alveolar osteitis?

- Saline rinse and Medicated dressing

161. Distance between 2 adjacent implants?

- 3mm

162. Hands foot and mouth disease (and herpangina) caused by what virus?

- Coxsackie virus

163. Pka of a local anesthetic affects?

- Onset of action.

164. Analgesic for an alcoholic?

- Ibuprofen

165. Stress and illness are often related. The best description of their relationship is which of

the following? Jk

a. Stress is primary cause of illness

b. Illness is adaptation to stress

c. Stress is a psychological reaction

d. They often occur together but are casually unrelated

e. Stress is contributory to illness and illness is usually stressful

166. In pursuit of what the dentist believes is best for the patient, the dentist attempts to

control patient behavior. This is known as?

Page 194: Decembery RQS – NBDE II

Combined remembered Qs 2020 a. Autonomy

b. Competence

c. Maleficence

d. Paternalism

167. HIPAA was designed to

a. Ensure the security and privacy of health information

b. Provide insurance coverage for providers

c. Increase hospital testing ability

d. Encourage employees to stay in their jobs to retain insurance coverage

168. MOST APPROPRIATE TEST TO DETERMINE THE DIFFERENCES BETWEEN TWO

MEANS?

a. CHI SQUARE TEST

b. T TEST

169. Information about subjects in a study included their ethnicity. What level of

measurement is ethnicity?

a. Ordinal

b. Nominal

c. Ratio

d. Interval

170. A moderately mentally challenged 5-year-old child becomes physically combative. The

parents are unable to calm the child. Which action should the dentist take?

a. Force the nitrous oxide nosepiece over the child’s mouth and nose.

b. Hand over mouth exercise (HOME).

Page 195: Decembery RQS – NBDE II

Combined remembered Qs 2020 c. Discuss the situation with the parents.

d. Firm voice control.

171. Which of the following is the principal nonverbal cue that two or more people can use to

regulate verbal communication?

a. Eye contact

b. Verbal communication

c. Gentle touch

d. Facial expressions

172. Which is the most important feature of systematic desensitization?

a. Muscle relaxation

b. Cognitive restructuring

c. Exposure

d. Education

173. If a dentist is stuck with a needle white treating an HIV infected patient, which should he

perform?

- Antiretroviral therapy

174. Untreated caries in permanent dentition?

- African Americans

175. Children with most prevalence of caries?

- Hispanics

176. All are ways of sterilizing in the autoclave except one which one is the exception?

- Solid metal box. (other answers included plastic pouches, bags)

177. Which term listed below measures the proportion of those without disease who are

correctly identified by a negative test?

- Specificity

178. I had a question ask about identifying the independent variable in a study. Not the same

from the files. (read the patient management files and look out for a question about a

question asking the same thing and know how to determine the variable)

Page 196: Decembery RQS – NBDE II

Combined remembered Qs 2020

General notes📖

rubber dam gives a black background appearance, affecting the Shade selection

Everything will llook whiter than it actually is

The attachment of the JE to the tooth surface can occur on enamel, cementum, or dentin.

The position of the EA on the tooth surface is initially on the cervical half of the anatomical

crown when the tooth first becomes functional after tooth eruption.

ferrule means

"a metal ring or band around a slender shaft that prevents splitting". In dentistry it basically

means a ring of solid tooth structure going around your crown prep. I believe most

prosthodontists recommend 1 mm minimum of good tooth structure in order to put a crown

on a tooth in order to have a decent prognosis (this doesn't include the build-up). I may be

wrong about the 1 mm minimum- you definitely want as much tooth structure as possible

going around the tooth. This creates what is referred to as a "ferrule effect" which improves

retention and resistance, exactly and the longer the ferrule the more the resistance to

fracture!

welcome...check mosby p 27 as well to get it..

In Davis Crowns a Post + Core is Casted where core covers 1-1.5 mm on to prepared

coronal tooth stump to prevent vertical splitting of root due to wedging effect of cast post=

Ferrule effect

*Forceps #

89# Maxillary right molar

90# Maxillary left molar

17# Mandibular molar

23# Mandibular molar roots

210# Maxillary 3rd molar

222# Mandibular 3rd molar

150A# Maxillary premolar and molars

151A# Mandibular premolars

Page 197: Decembery RQS – NBDE II

Combined remembered Qs 2020 150S#Maxillary primary teeth

151S#Mandibular primary teeth

65#Max premolars

286#Max premolars,incisors and root tips

Whats penumbra???

Smaller the focal spot area sharper the image appears

Fuzzy unclear area that surrounds a radiographic image is called as penumbra

So ↑ in source to object distance

↓ in film to object distance.

Smoking is not a contraindication for the placement of dental implants, however, failure rates

are higher in smokers. The failure rate is related to the amount of smoking on a daily basis

and the pack/years history for the patient.

Based onon angle formed by cutting edge with edge of the blade.

Away from the handle distal GMT

Towards the handle mesial GMT.

BURS-:

330- pear shaped, 34 inverted cone, 6-round, 558 straight fissure cross cut.

round- 1/4, 1/2, 1, 2,3,4,5,6,7,8,10

Inv cone- 33 1/2, 34,35,36,37,38,39, 36L, 37 L

Straight plain- 56,57,58,59,60, 57L,58L

Straight cross cut- 556,557,558,559,560,557L, 558L

Tapered fissure plain- 169, 170,171,169L,170L, 171L

taperd cross cut - 699(l)700 (l),701,702,703,

End cutting-957,958

Wheel-14

Pear- 329,330,331 331L, 332

It's dried blood. Someone mentioned on the earlier posts only dripping wet blood comes

under regulated waste; went by that explanation

y-shaped in antral. I remember someone had got this in their exam.

Page 198: Decembery RQS – NBDE II

Combined remembered Qs 2020 At low doses, nitroglycerin will dilate veins more than arteries (reduces preload, primary

mechanism of action), but at higher doses it also dilates arteries (afterload reduction) and is a

potent antihypertensive agent. In cardiac treatment, the lowering of pressure in the arteries

reduces the pressure against which the heart must pump, thereby decreasing afterload.[18]

Dilating the veins decreases cardiac preload and leads to the following therapeutic effects

during episodes of angina pectoris: subsiding of chest pain, decrease of blood pressure,

increase of heart rate, and orthostatic hypotension.

Ectopic eruption sequences: Its max first molar>max canine>mand canine>mand second

premolar>max lateral.... Most common to least

Hep C. Also associated with chronic liver disease, hepatocellular carcinoma and the no 1

cause of liver transplantation in US

Other names for alarm clock headache: Cluster headache, Horton's headache, red neuralgia,

histamine headache, and Sphenopalatine neuralgia

best post= parallel and best pin= self threading

Biostatistics in Pt management

* Validity....> should be compared to gold standard and should be high sensitive,specific and

unbiased

*Realibility....> should be reproducible and repeatable with same value means produce

similar results

* Sensitivity....> % of persons having the disease TP/TP+FN X 100%

* Specificity ....> % of persons not having the disease TN/TN+FP X 100%

* predictive value positive....> TP/TP+FP X 100%

* predictive value negative....> TN/TN+FN X 100%

Highest prevalence of caries = Hispanics

Highest DMFT = White (caucasian) (highest amount of restored teeth)

Highest untreated primary teeth = Hispanic

Highest untreated perm teeth = Black (African American)

Moderate periodontitis = Black males ( African American)

Class II caries = Whites (caucasian)

Page 199: Decembery RQS – NBDE II

Combined remembered Qs 2020 Class III caries = Blacks (African American)

Cleft lip/palate w/ Class III occlusion = Native American

Cleft lip alone = Asian

Cleft lip in USA = 1:700 to 1:800

class 2 malocclusion : whites of northern European descent

class 3 malocclusion : Asian

Caucasians have more lip cancer while African american have more oropharyngeal

carcinoma.

Anterior open bite: African American(blacks)

Deep bite: cuacasian( whites)

Cemento osseous dysplasia - black middle aged wome

so caries is generally low in down sy and periodontal disease are high in these patients. At

the same time in case of cystic fibrosis due to medication (antibacterial specially tetracycline)

and composition of saliva caries is very lowe(lower than Down sy). If you had both, then

choose CF ,I red this today in oral patho

il-8 attracting inflammatory cells, TNF similar to il-1 but less potent, mmp 8 is collagenase so

they involve in periodontal tissue destruction.

Minimum thickness of a connector is 3 mm

Mesiodistal area is shared by pontics and connectors

..In Class 2 -->Amalgam (internal line angles are rounded)

but in case of Class 2 -->Inlay (internal line angles are sharp)

.In working side interferences:BULL rule (inner inclines)

In Balancing side interferences:Mandibular Buccal Cusp (inner inclines)------->although they

are Secondary Centric Holding Cusp

we never grind Maxillary lingual Cusp (Primary Centric holding Cusp)

Supracrestal fibrotomy B is particularly for rotations..while D keep the contacts wide is for

lower anteriors so as to make sure to prevent slippage of contacts...as given by Raleigh

Williams and I think also in Garber...I ll go with D...

For Cusp Protection via GOLD

Page 200: Decembery RQS – NBDE II

Combined remembered Qs 2020 a.Capping is done in Functional cusp

b.Shoeing (Veneer) is done in Non functional cusp except Buccal cusp of MAXILLARY

PREMOLARs and MOLARs

for minikin ..its 3 mm

minim ...5 mm

cross bite:----Dentoalveolar: Inclined plane, Reverse SS Crown, Tongue Blade, Maxillary

Hawley with Z-Springs, and Posterior Bite Block Combination to discclude the anterior

dentition

Functional... check for the premature contacts.... Most likely they are primary canines which

are the culprits as they haven't attritioned with rest of the primary dentition

osseous resective surgery includes ostectomy and osteoplasty.......in which aim is to reduce

and removve supporting and non supporting bone ......in this first of osteoplasty includes

technique of fesooning which is vertical grooving or bone festooning to reduce buccal and

lingual thickness of bone interdentally........

Aldosterone release and presence of insulin causes uptake of potassium from blood stream

causing hypokalemia

Both true,etching time for primary teeth is 30-60 seconds.

patient is aware of problem but there are some barriers like time, expenses.. so it comes

under contemplating

Overextended max denture __ result in sore throat

Condylomata acuminata (6-11) HPV while 16 18 , 31 33 HPV for verrucous carcinoma

1 nd 2

High copper(no gamma 2 phase no loss of marginal integrity)

Spherical(less hg content 40-45%)

Acromegaly :Enlarged Tongue

Pieer robins:Posterior tongue displacement(glosoptosis)

Down:fissured

Does anyone have a reference for 3?

Page 201: Decembery RQS – NBDE II

Combined remembered Qs 2020 A crown's color is selected by looking at the adjacent teeth. And if it has gotten 'lighter'

means the adjacent teeth have stained. And since a crown is selected by looking at

surrounding teeth only, will we not change the crown rather then messing with all these other

teeth?

Flouride fatal doses

Fatal dose 16 mg/kg in children

2g in adults

while toxic dose 5-10mg/kg in children

Agranulocytosis, you see ragged ulcer in gingiva and palat

@Yeah Swiss cheese pattern in cribriform is seen in adeno ....and honey comb in acinic

1.5 mm between tooth and implant and 3 mm between two implants

This was discussed here a few days ago , someone wrote that serial extraction may cause

deep bite , coz of lack of support , so in case there is already deep bite present then its better

not to do it

it's written in decks that OSHA responsible for all the employees to get hep b vaccination

and it tells cdc to take care of those vaccination..I don't remember

Bonding agent contains a hydrophilic n hydrophobic strata . Hydrophilic part bonds to dental

tissues n composite should bond to the hydrophobic part... since dentin contains more

organics n H2o than enamel, the agent bonds better to dentin than enamel

GINGIVAL CYST of newborn or adult Not seen radiologically. Not to be mistaken with

Epstein’s Pearl (on median palatal raphe) and Bhon’s nodule - seen on alveolar ridge of the

newborn.

Operant conditioning (sometimes referred to as instrumental conditioning) is a method of

learning that occurs through rewards and punishments for behavior.

b, you dont remove caries when they are half way in enamel.. they are treated by flouride tx...

but when it touched DEJ you have to remove it

16 Weeks between the 1st n 3rd dose n 8 weeks between dose 2 n 3. If only dose 3 is left it

should be given immediately

Page 202: Decembery RQS – NBDE II

Combined remembered Qs 2020 For ADHD patient ask patient to have the medicine 1/2 hr before appointment, rest is all

same , keep short appointment and morning one,

Yes most is chlorphenarmine maleate then phenargan then least is diphenhydramine

Aluminum is the weakest crown material for implant abutment

pneumbra, causes indiscrete margins of x-ray's film

245- cavity prep

Prospective-->foward in time (starts with the risk factor and ends with the disease)

Retro(back)pective--->Backward in time(starts with the Disease and asking about the risk

factor in the past)

iagnostic Clues Craze lines are frequently confused with cracks, but can be differentiated by

transillumination. If the tooth is cracked, the light will be blocked, allowing only a segment of

the tooth structure to light up; if the tooth only has a craze line, the entire tooth structure will

light up,shouldn't the answer be Periapical abscess then as the tooth is intact?What is the

answer?

Ok,Periapical abscess seems appropriate as the whole tooth is intact n can be illuminated.

low grade mucoepidermoid carcinoma has best prognosis

Iopa of max canine in periapical region lateral wall of nasal fossa and anterior border of

maxillary sinus form an inverted Y known as inverted Y OF ENNIS

B, this is what I found

Most Sensitive: Blood-forming organs

Reproductive organs

Skin

Bone and teeth

Muscle

Least sensitive: Nervous system

Physical restraints for moderate retarded child at emergency ... Cos voice control and HOME

are for pts with normal mental activity.. GA not possible as it is an emergency visit...

what's the sequence of hue value chroma when we have to match a shade?

First Hue then Chroma then Value

Page 203: Decembery RQS – NBDE II

Combined remembered Qs 2020 Wavelength dependence ------> hue

Concentration dependent------->chroma

Reflection of amount of light dependent-----> value

For years, people with asthma and allergies have been told to avoid antihistamines because

they dry sinuses and cause constriction of the airways and sedation. Newer antihistamines,

including fexofenadine (Allegra), loratadine (Claritin), and cetirizine (Zyrtec) may be beneficial

for mild allergic asthma, but antihistamines are still problematic and are usually not beneficial

for moderate to severe asthma. People should not use antihistamines if a sinus or respiratory

infection is present. Used routinely, antihistamines thicken mucous secretions and can

worsen respiratory infections. It is particularly important to treat any co-existing sinus

infection in people with asthma because they might not respond to asthma treatments unless

the infection is first cleared up.

Contemplation

reason:

Contemplation . pt is thinking about quitting but not enough motivated yet..

pre-Contemplation .. pt is not even thinking of quitting

i think that denial and preContemplation are the same..

action. pt is actually working to quit.

correct me if i am wrong ...

yep... I was reading some random article and according to tht., if fluconazole isn't the given

choice then ketoconazole is the answer...for systemic anti fungal

According to A.D.A., the acceptable dimensions of a tooth brush are

(a) 1-11/4 inch long, 5/16-3/8 inch wide, 2-4 rows, 5-12 tufts per row

caries is seen on which aspect of 1st primary molar??

distal surface, occlusal,

Ans is occlusal.

Full ext—how long do you wait for max denture o 1 day

o Same day o 1 week

o T- 8 week o 16 week

Page 204: Decembery RQS – NBDE II

Combined remembered Qs 2020 6 weeks for preliminary impression and 8 weeks for final impression

The pain from the loss of

pulpal vitality is the most common presenting complaint ofpatients with combined lesions.

The symptoms reported are those most often found with pulpal disease. Thermal

pulp testing provides information relative to the status ofthe pulp, and dental radiographs

can confirm the presence of apical changes and the extent ofbone loss. Careful probing

confirms the presence and morphology ofany periodontal pocket and permits location of

the conmunication with the apical lesion.

In combined endodontic-periodontic lesions, it is generally wise to treat the endodontic

component first, because in many cases this will lead to complete resolution of the

problem.

After successful endodontic treatment, the residual periodontal pocket that remains can

be more predictably heated. The periodontal therapeutic objeclives vary with the extent

Permissive way....wid special child

decisive is when its a normal child and u know he has ro capabilities to understand the

reason

ans is pemphigoid where there is seperation of membrane and epethilium from connective

tissue whie in pemphigus there is blistering of skin due to antibodies being direted against

desmosomal adhesion molecules ..meaning only the epidermis is affacted

Pemphigoid -sub basilar cleft

Pemphigus- intra epidermal cleft

intenal: RCT, for external: if not self limitting than in most cases extraction

chi-square is the sum of the squared difference between observed (o) and the expected (e)

data (or the deviation, d), divided by the expected data in all possible categories.

From best to worst for implantation ...

D1 Ant Mand,

D2 Post Mand,

D3 Ant Max,

D4 Post Max:)

Page 205: Decembery RQS – NBDE II

Combined remembered Qs 2020 ECC locations start with the most affected

1. Ant max

2. Post max

3. Post mand

4. Ant mand --> least because tongue cover them

unbundling is done by dentists and downcoding is done by insurance

bundling : all pricedures into one -by insurance

upcoding : dentist playing with codes to get more $$

If there was any other problem the margins wouldnt have fit....whenever we are fitting a

crown the first thing u check is proximal contacts...then margins and then occlusion...so if the

margins are ok it is understood tht the proximal contacts are ok too

Its iv bisphosphonates its effect will not reduce on stopping the medication just 3 months

before that too when patient is taking from 2 years, I guess second option will be best , we

will definitely choose non invasive procedure coz its IV bisphosphonates

Normal salivation rate is : It's 1.5 L/day = 1 ml/min

a pier abutment is a natural tooth located between 2 terminal abutments for example..if u

have a missing 1st premolar and missing 1st molar,the 2nd premolar would be the pier

abutment that is located between the canine and 2nd molar.

condyle for both...only difference is second most common...symphysis is 2nd most common

in children and angle is nd most ommn in adults

To make the crown narrower , Move line angles more facially and increase interproximal

embrasure

mental the most difficult one,platal the easiest but painful,The buccal block is a successful

injection because the buccal nerve is readily located on the surface of the tissue and not

within bone

In self cure: initiator- benzoyl peroxide, activator: toluidine. In heat cure: initiator -benzoyl

perxide , activator -heat.

Working cusp for amalgam is 2.5 to 3

Page 206: Decembery RQS – NBDE II

Combined remembered Qs 2020 Non working for amalgam is 2.o

W cusp for cast gold is 1.5

Non w cusp for cast gold is 1.0

W cusp for metal ceramic is 1.5 - 2.0

Non w cusp for metal ceramic is 1.5 -2.0

I read that people with CP have a 30-50% or 35-50% of seizure development and mental

retardation/learning disability.

Working side interference: reduce de lingual incline of the buccal cusps of maxilary molars

and inner incline of lingual cusps of mandibular molars. Non working side interference: grind

only the inner incline of the buccal cusps of madibular molars. In reality u should never

reduce the primary centric holding cusps (lingual of max.) But in nbde u can reduce it if is

high in centric, protusive and lateral excursion movement.

Stieglitz pliers -- use for removing silver points

Implanted opened apex tooth regain blood, Ans is 20 days...it will be regained within 20 days

after replantation but nerve supply lags behind.

Cavicide disinfectant for dental chair

Is phenargan contraindicated in pregnancy

Animal reproduction studies have shown an adverse effect on the fetus and there are no

adequate and well-controlled studies in humans, but potential benefits may warrant use of

the drug in pregnant women despite potential risks.

CATEGORY C-pregnancy

b . cheek bite with edge to edge relationship

A dentrifices have 20-40% abrasive particles. And a abrasive paste have about 80% of

abrasive particles. Also most dentrifices (tooth pastes) have fluoride incorporated... most

tooth pastes have 1450 ppm of fluoride.

300-200/1000 = 0.1, incidence define no. Of new cases

JUNE 3

Page 207: Decembery RQS – NBDE II

Combined remembered Qs 2020 Exam was okay okay. Lets hope I PASS this exam.

1-composite monomer-options were polymethy methacrylate,UDMA,methacrylate

2-common duct for sialolithiasis-stenson,bartholin,wharton,minor salivary duct

3-Radiograph showing large radioopacity in maxi anterior tooth region-options-AOT,Fibrous

dysplasia,Ameloblastoma,condensing ostitis

4-pregnant patient in left to prevent pressure on-IVC

5-Case question in which pt has habit of smoking 30 pack cigrates per year.Options were-

precontemplatory,contemplatory,denial,action,Maintenance

6-cryer forcep used for-extraction of mandi molar root piece.

7-what doesnt get killed in steam sterilization-options were all virus and one bacillus st.

8-minimum buccolingual width needed for 4 mm implant- 6mm

9-occlusal plane of denture is limited by-fox plane registration,1/2 or 2/3 of retromolar

pad,something about tuberosity.,and other option was distal flange something like that not

sure

10-best graft for osseous defect- Autograft,allograft,alloplast,dried frozen

11-premedication for endocarditis patient.-

12-what is not recommended to place the instruments in steam sterilization-paper

packets,paper plastic packets,cloth pouches,solid metal containers,plastic bags.

13- broken file in one of the canal of molar what to do - obturate and follow up.

14-infection from which of the following space goes to mediastinum-pharyngeal

pouches,submandibular,Infratemporal,Retropharngeal.

15-Angular chelitis caused by all expect- decreased VDO,lingual inclination of mandi

Anteriors,vitamin deficiency,smoking habit

16-cells present in initial stage of periodontitis

17-question on combination syndrome but it was not straight forward to understand.

18-case and radiolucent lesion in periapical region of mandi anterior with no pain on

percussion and normal response to thermal stimulus-Periapical cemnto dysplasia.

19-muscle that is in contact with disto buccal flange of mandi denture-lateral pterygoid,medial

pterygoid,masseter,temporalis

Page 208: Decembery RQS – NBDE II

Combined remembered Qs 2020 20-radio opaque structure seen in front of C3/C4 in cephal

21-how to reduce value-add blue cyan,complementory color,adjacent color,redo

22-mean question-

23-property of GIC over composite

24-warfarin -Vit K

25-identify kennedy classification picture

26-radiograph-dentigerous cyst

27-radiograph -odontoma

28-differential diagnosis of crusted ulcer on lip-phemphigus,phemphigoid,verucous

vulgaris,HSV

29-xray projected @ 20 angularion from distal where is distal /mesial root located-something

like that -SLOB rule questions I got 2 but they were NOT easy to answer.

30-inverted Y line radiograph and how it is formed

31-identify occlusion-

32-most commonly seen occlusion in paediatric patient

33-position to treat 2year old clild

34- 2 year old child has fear from -light,sound,instrument or eugenol smell.

35-case question slurred speech cause -ischemic attack

36-bells palsy- facial nerve

37-fibroma -hyperplasia

38-what to avoid in minimum keratinised tissue- apical graft

39-least important during closure of mandible-relaxation of lateral pterygoid

40-numbness in lower lip without any anesthesia- Malignancy

41-avulsed primary tooth- do not reimplant

42-all of the following factor contributes to reduce sensitivity of root after perio treatment

except-options were 6 months maintence visit to dentist,desensitizing toothpaste, plaque

control, and use of fluoride varnish

43-appilance use for orthodontic correction of 4 maxi anteriors-hawley retainer,nance

button,lingual arch

Page 209: Decembery RQS – NBDE II

Combined remembered Qs 2020 And something else

44-for maxi anterior tooth-which block should be given along with nasopalatine block.

45-identify radiograph showing linear radiolucency above the root of 1st maxi molar.-I choose

PSA canal.

46-initial step in biofilm formation.

47-Y we avoid xray for a routine follow up appointments within 6 months

48-better prognosis-1 wall defect,2 wall defect,3 wall defect,4 wall defect

49-emergency treatment for endo infection.

50-indirect pulp capping when?

51-reasons for fracture of amalgam restoration except-

52-MO amalgam restoration Y did it fracture- narrow dovetail, rounded axiopulpal floor,wide

intercuspal distance,etc

53-feature if gracey’s scaler.

54-bells palsy cAused due to needle placed too backward,forward,superior,inferior

55-Procedure for apical closure in permanent immature non vital tooth-Apexification.

56-one more question for apexifixation but asked in different way.

57-alginante impression material placed in water -Imbition

58-Y acrylic denture is cured after 24 hours

59-if you want to work on infectious site.how do you numb? Block or infiltration

60-pt needs to take break in between procedure- what medicine-Laxis

61-fluoride in community water-75%

62-medication of status epilepticus- diazepam.

63-quetion on drug,potency,efficency ,

64-cafe e lu spots-Neurofibromatosis

65-all are feature of dementia expect-

Preservation of short term memory,lose of function,reduce intelligency,and one more.

66-evaluation of 2 groups A and B with 2 drugs for same period what type of study.

67-fetal alcohol syndrome

68-percussion test is used to check-

Page 210: Decembery RQS – NBDE II

Combined remembered Qs 2020 69-polycrystalline ceramic- Zirconia(not sure exactly what question was)

70-gun shot wound is similar to which type of fracture-simple,cominutted,complex and one

more option

71-high strength /low strength ceramic

72-HPV which is more specific in oral lesions.-6,11,18,20

73-mandible follows under which tissue growth.? Somatic,neural,lymphoid etc

74-treatment for cervical caries no GIC in option

75- flowable composite- A1,A3,B2,C

June 5-6’ 2020 RQS.

Kindly read the questions very carefully guys. What we all provide each other as RQS are not

the same what we get in REAL EXAM.. and now I understand why people can only provide

topic names in their rqs cuz its difficult to remember all with all the options.

In my exam first 2 sections were very hard to understand the questions, difficult questions

with wired options. Whatever I have provided in my rqs are no doubt the repeated question

too, I realized when I start typing them, but these are still the few I remember, EXAM is of

400 questions for day 1 so many more questions are there which make u madd. Stay strong

and pray for each other.

So please keep me in your prayers please please.

Just an advice for COVID-19 candidate.

we have to wear our mask all the time, what I did--- remove the mask from ur nose when u

sit Infront of ur exam screen. I feel suffocated so just do that if u feel the same.

DAY 1

Page 211: Decembery RQS – NBDE II

Combined remembered Qs 2020 1. Major connector- support and rigidity

2. Associated with rheumatoid heart

· heart mumurs

· Joint effusion

3. Most important detail that would most affect the outcome of a fracture tooth

· 48 hr delay of txt

· Fracture being far away from apical

· Larger than normal pocket

· Infection

1. Non odontogenic max sinus what antibiotic? Amox with clavu

2. ingredient in maurijuna ending in cannibal or THC

3. antihistamine moa: blocks histamine at receptor

4. Epi+ propranolol= Inc BP and bradycardia

5. Sickle anemia mutation- something really diff in language not glut to valine ….. don’t

know what.. missense mutation (Point mutation)

6. Amalgam what do we not do- Put it in hazards

7. Othro tension side (entirely diff French language ☹ ) but ans OESTEOBLAST.

8. CPR I put lose airmask not sealed properly as when we doing CPR we have already

clean the throat and then start CPR.. so CPR include chest compression and breathing. I

marked this.. (pray)

9. There was a question on smoking who will leave and why? precontemplation

10. Cause of furcation except – I put endo.. other options were pearls , grooves

11. Heamingioma pic

12. Fibrous dyaplsai pic

Page 212: Decembery RQS – NBDE II

Combined remembered Qs 2020

13. Angina pain which fibers – A delta

14. All of the following are considered for a demineralized lesion except:

· Texture

· color

· age

· location

15. MI treatment—diff language

Supportive care Defibrillation and Oxygen therapy Medications Anticoagulant, Antianginal, Narcotic,

Beta blocker, Statin, and ACE inhibitor Medical procedure Coronary stent and Coronary angioplasty

Surgery Coronary artery bypass surgery Ace inhibitors and Glycosides like Digitalis and Digoxin (ace

inhibitors captorpil)

16. Most common periodontal disease – chronic perio,, option were aggressive, marginal

Papillonma and fibroma diff – The fibroma is a smooth surfaced, pink, mushroom shaped growth with

a stalk that appears on the affected surface. A papilloma is also mushroom shaped but has an irregular,

white, pebbly surface. A papilloma is a growth that ears in its own, not as a result of irritation. Rx:

conservative exc

17. Synchrondosis – epiphyseal plate ,b/w vertebrae immovable.

There were many many questions on FLOURIDE.. put of the world ☹☹ Cariostatic effect of fluoride

is at calcification stage of tooth development. ■ Fluoride converts hydroxyapatite to fluorapatite. ■

Fluoride ↓ solubility of enamel. ■ Toothpaste contain 1100 ppm of fluoride.

18. LAP treatment – SRP, Abx

19. Endo bacteria – obligate anaerobes

Before rct it is strict anaerobic But after rct it is gram + facultative anaerobes

Page 213: Decembery RQS – NBDE II

Combined remembered Qs 2020

20. Addiosns disease- hypotension

21. Veryyyy diff 3 questions on antogginist – competitive: atropine and propantheline,

physiologic: epi and nitro. Naloxone : non-selective and competitive opioid receptor antagonist in

case of opioid overdose

22. Potency

Response to a drug over a given range of concentrations. Depend on dose of drug; less mg for same

efficacy has more potency

23. Mand denture closure on molding

Distobucc: masseter, distoling: sup pharyngeal constrictor, buccal shelf: buccinator

24. Radio patient cervical caries - chemotherapy

25. Many questions on dry socket- what to do what not to do.. please read the question options

carefully again and again.. I got excited whenever I read some topic of known question but

when I read options that are entrilyyyyy different --- hahahhahah.

Dry socket treatment: no abx, no curettage, warm saline and sedative dressing alveogel

Dry socket can occur three to four days after an adult tooth is removed. The blood clot that should form

after removal is dislodged or dissolved before the wound heals, exposing underlying bone and nerves

26. What is not a true cyst. stafne

27. What is a true cyst. dermoid

28. Nevoid basal cell carcinoma symptoms

Minor Features:

· Medulloblastoma.

· Increased head size and large forehead.

· Cleft lip or palate, extra fingers or toes.

· Abnormal shape of the ribs or spinal bones.

· Eye problems such as cataracts, small eyes, or tumors in the iris.

Page 214: Decembery RQS – NBDE II

Combined remembered Qs 2020

· Fibromas, meaning benign fibrous tumors, of the ovaries or heart.

· Abdominal cysts.

Gorlin-Goltz syndrome is an autosomal dominant disorder with a high degree of penetrance[1] and

variable expressivity. [2] It is characterized by basal cell carcinomas, odontogenic keratocysts, palmar

and/or plantar pits, and ectopic calcifications of the falx cerebri.

29. BSSO nerve damage IAN..

30. Anterior cross bite treat immediately

31. Cross bite. manyyy diff questions on it.

A: applianceS to correct anterior cross bite:

1) tongue blade

2) palatal spring

3) Z-spring

4) finger spring

applianceS to correct posterior cross bite:

1) jack screw

2)hyrax

3)haas

4) hawley

5)quad helix

6) transpalatal arch

32. Amalgam fracture why? Inadequate prep

33. Amalgam low modulus of elasticity

34. Gingivectomy – supra bony , adequate gingiva

Page 215: Decembery RQS – NBDE II

Combined remembered Qs 2020

35. What not to do where there is less keratinized gingiva specifically on distal surface of last

tooth.. option apical position flap, and distal wedge. I picked distal wedge as distal to last

molar is distal wedge not apical reposition on distal.

36. No perio questions on flapsssssss

37. No opertiveeeeeee amalgam composite,

38. No classs cavity prep questions

39. Lots of patient management – trust me that’s ur gut feeling when u pick that

40. Whats the things you will do when a patient visited to u and worried about his/her

condiotin.. show empathy and concern- It seems like you r worried about your condition, but

we will look into it (something similar question in running in our old rqs)

41. Maleficence and non-male definitions with diff wordings. – understanding is important (no

similar explains examples like we did)

42. There was a question on dopamine and Parkinson’s.

43. Biological width – JE + CT

44. Propranolol is the Prophylactic medication to angina not nitroglycerin.

45. C4 is the hyoid bone not C3 pleaseee keep this in mind (these 2 questions have confused

options for us in our previous discussions.

46. Question of VRF SYMPOTOMS—SYMTPOM IS COLD SENSITIVITY , sSIGN is

pain on biting .. I choice cold

47. QUESTION: What does percussion test?

a. Presence of inflammation in PDL or not

b. Spread of inflammation to periodontium from PDL or not

c. Responsiveness

d. Pulp vitality

48. avulsion and splint ? how many days – flexible splint, 7-10 days

49. horizontal fracture and splint- just the duration of it, no type of forizaintal root fracture was mentioned –

Horizontal root fractures—-> Rigid Splint, 3 months (12 weeks) . Dx: take multiple vertical X-rays.

3 vertical and 1 occlusal

Coronal is 3 months

Middle is 3 weeks

Page 216: Decembery RQS – NBDE II

Combined remembered Qs 2020 Apical 2 weeks or leave

50. mastoid process red spot and periorbital echymosis, which typoe of fracture? Lefore 1 , 2 , base of skull

fracture,( so its lefort 3 base of skull fracture and mastoid process red spot means bleeding there)

51. nutrient canals xray

52. worst solution for avulsed tooth to dip--- water

53. QUESTION: Advantage of a direct composite vs. a veneer? Direct composite is only 1 appointment vs.

veneer is at least 2

54. supra gingival antibiotics effective for removal of plaque? T/F – False

Gracey curette shape and details . every option is mixing read it slowly please----Semi circular, more then 45 and less than

90 degree angle. semicircular.. enter at 0 then perpendicular 45 but less than 90 the blade of the Gracey

curette is only offset by 70 degrees, giving the blade a lower cutting edge and an upper non-cutting edge

55. extraction of deflected divergent roots of upp molar, how to extract? --- section the tooth , others options

were flaps and more aggressive ..

56. ortho treatment with some veener placement on anteriors EXCEPT

-move some teeth after the veneer

-move some teeth before veener ( I put this considering the EPOSOP as ortho before prostho)

-just veener

57. one question on pins EXCEPT , which I was discussing last days whats wrong in pin statement and ans was

pins provide strength to amalgam as an EXCEPT question, it provides retention does not strengthen the

amalgam)

58. leakage from rubber dam—holes too close.

59. some questions on radio therapy and its symptoms after that.. erythema was not in option :D and I don’t

remember too please read this

dryness, itching, blistering, or peeling. Fatigue

60. QUESTION: Three factors that affect caries initiation: Substrate, bacteria, host susceptibility but options

were not these .. im sorry I forget the options.

61. something about the PH of enamel which is not correct.. and some long statement was given was 6.5 and I

chose that .. enamel: 5.5, dentin: 6.5

Page 217: Decembery RQS – NBDE II

Combined remembered Qs 2020

62. non working side cusp was related to this and best I think was this-- mand buccal cusp lingual inclines, max

lingual cusp buccal inclines

63. what movement dis occludes full posterior teeth on movements---- mutually protected (canine guidance)

64. QUESTION: Two things that account for a successful posterior composite restoration? The type of resin and

the type of preparation

65. few questions on ENDO 2 I think.

66. I had many short cases on day 1 too and those are time consuming too. Please manage accordingly.

67. inflamtory resportion which condition it happens?

68. Bowl-Shaped Resorption (inflammatory resorption) – involves dentin and cementum; tx –

immediate RCT; CaOH every 3 mo and after 1 yr, obturate with CaOH sealer; necrotic pulp

69. ankylosis replacement resportion.

Replacement Resorption – resorption of root surface and bone causing ankylosis; often seen in

replant cases; accompanies dento-alveolar ankylosis, characterized by progressive replacement

of root by bone (no pdl); signs: no mobility, metallic percussion sound, and infraocclusion.

DAY 2:

Case 1:

Tonsillitis case:

GIRL having halitosis and caries in mouth, hr class mate make fun of her due to bad breath..

Q1-common question what the cause of bad breath and I marked-- chronic tonsillitis (there were

not much caries like rampant caries in mouth of more than half of her teeth are involved as carious

lessions- just 5 – 6 I think carious teeth.. so I don’t think this much caries can cause bad breath.. so

I picked CHRONIC tonsillitis the case of bad breath

Q2- what is he best oral treatment u will advise – maintaining good oral hygiene.

CASE 2- asthma patient with inhaler presented in clinic

Q1- patient had diff in breathing in start as he enters the clinic, what will you do for treatment

management, -

-STOP THE TREATMENT UNTIL HE IS RECOVERED with the asthma ( I marked this, as he is

asthmatic patient and attack might be due to anxiety and follow this SPORT)

- Continue treatment

Page 218: Decembery RQS – NBDE II

Combined remembered Qs 2020

- delay treatment

Q3- atenolol side effects- staining in teeth , xerostomia( THIS)

Case 3: male 45 years patient has lots of missing teeth and not happy with smile, and what to

replace all his teeth (didn’t mention he want CD or fixed as implants after removal ) he’s just not

happy with his teeth, need removal of all and

Q 1- what would u advise him for treatment plan EXCEPT.

- CD (THIS)

- PD with fixed in posteriors

- IMPLANT WITH PD in posteriors of lower

Q2- considering the patients demand (removal and replacement ) what is included in treatment

option for upper arch EXCEPT

- Removal of 2, 5 (as these 2 teeth were only present in the upper arch)

- Sinus lift -----------sinus lifting will be recommended if patient wants fixed implants as the

presented Pano had left sinus and bone level almost sinus was so deep towards bone)

- Alveoloplasty (THIS I choice not sure)

- One more I forget

Case 4 , 5

2 CASES ON ORTHO and CORSS BITE .

Q-1WHATS not included in ortho treatment plans

Q2-If we do the treatment and doesn’t included patients view which principal u r breaking.

- Beneficence

- Non male

- Informed consent (I choice this as informed consent is in principal of autonomy and here, we

are breaking autonomy of patient if not taking informed consent)

- Paternalism

ALL THE BEST AND JUST A SMALL PRAYERS FOR MY PASS NEEDED. Jazakallah

It’s a PASS guys! Thank you so much for your help!

My exam was on June 5 and 6!

Page 219: Decembery RQS – NBDE II

Combined remembered Qs 2020 Rqs Paola Alvarez. Good luck everybody!

Part 2 is longer than part 1, I would advice you to get some rest the night before of your

exam and the night between day 1 and day 2, I felt day 2 was way longer and it took me half

of the time so be ready for that!! Overall the exam is doable, most of my questions came from

recent files (Strawberry, DanMan, Canine, El Maestro).

For day 2 there is a file called masterday2 and unicorn. Those are important, the questions

wont be exactly the same but you will be asked about certain pathologies that are mentioned

in those files.

1) Question about behavior shaping. Which one is not a behavior shaping tecnique.1 of the

options was “Carrot-Stick tecnique” thats the one I marked but after the exam I found out that

the carrot-stick tecnique actually exists!!

2) Multiple questions about endo. They will tell you the patients symptoms and you have to

diagnose if the pulp is vital or not and what procedure you would perform, I didnt find them

too complicated.

3) I had one picture on day 1, it was a Lichen.

4) What does the "W" mean for clamps-- Wingless.

5) What forceps for a max premolar. 150

6) How long you have to wait after a bleaching to place a composite. 1 week

7) Mechanism of action of fluoride. Make enamel stronger.

😎 What does fluoride do. Replaces hydroxil

9) A patient comes back to the office for a first follow up, he tells you the bottom denture you

gave him keeps falling out when he talks. Probable cause? --Overextended borders--

10) How many pins to replace cusps with a onlay amalgam? 1 per cusp?

11) Mechanism of action of Sulfonamides? PABA

12) Implant placement: High torque, Low speed

13) Opioids antagonist: Naloxone

14) They asked me 3 times this about antagonists: High affinity, no intrinsic activity.

15) Axillary nodules: Neurofibromatosis

16) Amantadine: medication for parkinson

Page 220: Decembery RQS – NBDE II

Combined remembered Qs 2020 17) What is not a sign of opioid overdose: Mydriasis. I feel most of the questions were about

the exception..

18) Angles classification: like 3 or 4 questions on that with Xrays and or Pictures and then,

whats the patients profile.

19) Ear Lobe, max sinus and zygoma on Xrays

20) Medication for status epilepticus: Diazepam was not in the options so I put Midazolam

21) How many permanent teeth does a 9year old have in the mouth. 9-12?

22) Only 1 question about how much anesthetic for a 20kilos boy. 2 was not in the options so

I put 1

23) There was a picture of a bridge and the gingiva surrounding one of the retainers was very

swollen.. what its not the probable cause? Shade selection

24) I got like 6 questions on Hue, Value and Chroma so make sure you know those well.

25) Most important thing when placing an implant? Silly options so I put stabilization when

placing it

26) Patient that has been smoking for 30 years.- Precontemplation, Action, Contemplation,

Maintenance? I put precontemplation

27) Questions on Perio.. Whats the recommended time for maintenance appointments?

Every 6 months??

28) What is true about Codeine?

29) Treatment for Osteitis: Dressing and gentle wash.

30) Multiple questions about Cleidocranial dysplasia: Supernumerary and Ectodermal

dyplasia: Oligodontia

31) You want to report a drug? FDA

32) Whats the first thing a dentist have to do? Stablish rapport

33) Who is more prone to Chronic Periodontitis: Black males

34) Which one is malignant? Pagets disease

35) First thing that will happen after high dose of radiation? Erythema

36) Multiple Osteomas? Gardners

37) Leukoedema (Picture) It was bilateral, Blue/grey lesion

Page 221: Decembery RQS – NBDE II

Combined remembered Qs 2020 38) Epulis hystologically? Granuloma... Clinically: Fibroma

39) Disadvantage of oral sedation: No tritation

40) 1 month after placement of amalgam and its pigmented.. microleakage??

41) You were about to put anesthesia and pt starts wheezing (Pt was asthmatic)? Asthma

attack

42) What is not a sign of Parkinson? Good short term memory

43) What drugs cause dry mouth... Weird options

44) % of HPV? 85%

45) Distance between 2 implans? 3 mm

46) Pt taking baby aspirin(81mg)? We dont have to stop the medication

47) Whats the O in PICO questions? Outcome

48) What nerve does not give sensation to the tongue? Hypoglossal

49) What causes Osteonecrosis

50) Inform consent: When to get it? After explaining treatment plan

51) Prevalence/Incidence questions. Prevalence: Existing cases. Incidence: New cases

52) Biopsy after 2 weeks.

53) A picture of a big white lesion on the lateral border of the tongue, what biopsy: Incisional

54) Pt with CHF, what not to do.. Position him horizontally

55) Pregnant patient: IVC

56) Most potent LA? Bupivacaine

57) Patient is very anxious, you want to help him, what to do? Explain the treatment plan

carefully.. prescribe him a Midazolam

56) 5mm intrusion, primary tooth? let it re-erupt

57) Epi and histamine. Physiologic antagonism

58) pKa: Onset

59) What lesion is radiolucent? OKC

60) Drug is more efficient when its free in blood

61) Ludwigs Angina? Not in retropharyngeal space

62) Pt with pigmentation on the face, heart and kidney problems? Lupus

Page 222: Decembery RQS – NBDE II

Combined remembered Qs 2020 63) What antifungal can be given orally? Clothrimazole

64) How long to splint a tooth with a mid root fracture? 7-10 days

65) Photo initiator for composite? Camphoroquinone

66) Bone resorption IL? IL1

67) What fibers are stimulated when using Electrical test? A-Delta fibers

68) What type of radition on MRI?

69) Posterior border max denture? Coronoid process

70) Posterior border mand denture? Masseter muscle

71) Muscle that wont allow the pt lift his upper lip? Orbicularis oris

72) 1996 children with no healthcare.. something like that? I put medicaid

73) Pt with Down syndrome: No rampant caries

74) Best prognosis for a GTR? 3 walls defect

75) Dentinogenesis imperfecta and Amelogenesis Imperfecta are not related

76) Daughter bringing her dad to the dentist, she was his legal guardian. How to handle that

situation

77) Picture of a metal frame work, the question said "the lab rejected the design" what to

do?-- tell the lab to send the cast back to re-design

78) What not to give to a pt taking Gingseng? Aspirin

79) Whats not an adverse effect of lisinopril? Cough, high pottassium levels, interaction with

Nsaids or orthostatic hypotension

80) Antihistamine MOA: Block receptors

Physiological antagonist are drugs which are not effect directly on the same receptors

although they have different effects. They do not attach to the same receptors. But antihistamine

drugs attach to the receptors. 81) Collimation: Reduces exposure

82) Highest absorption of radiation? Scattered from pt

83) Picture of a pt with cervical caries on central and lateral incisors? biggest concern?

possibility of isolation

84) Picture of abrasion and how to restore.. I chose composite

Page 223: Decembery RQS – NBDE II

Combined remembered Qs 2020 85) Pt who is 25 years old, no history of extractions and no 3rd molars.. cause? Agenesia

86) Xray of a 9 year old.. 2 radiolucencies on areas of 2nd molars.. what are they? lesions? I

put 2nd molars formation

87) Picture of an amalgam and a grey shade surroinding it.. what is it? I put caries.. how to

restore? I put crown and possible post and core! It was big and near the pulp

88) Picture of a pt with atrophic candidiasis.

89) Basal membrane lesion? Pemphigus

90) Pt complaining that she shows too much gum when she smiles.. how to fix that? Lefort I

91) What includes a Lefort I? Max sinus

92) Califlower lesion? Papilloma

93) Picture of a pt missing some teeth. What is the concern if you are planning on giving him

an RPD? He didnt have space on the right side for RPD

94) Pt with no attached gingiva on 2nd molar. What not to do.. Distal wedge

95) Xray of pneumatization of max sinus and if its true that it happens when the pt has

chronic sinusitis

96) Sealants? Mechanical retention

97) Systemic fungal infection? I put histoplasmosis

98) Pt with pulp necrosis and sinus tract, what to do after RCT? no further tx needed.

99) Apexification apexogenesis questions

100) Not true about Sodium Hypochlorite? Chelating agent

101) Trauma to PDL? Intrusion

102) Least chance of creating a ledge? Short root, long root, curved root. I put short root.

103) Critial PH of enamel? 5.5

104) Preparation for an RPD.. Guide planes first

105) What pontic design is not recommended for the anterior area? Hygienic

106) What gives composite the radiopacity? I put the matrix

107) About 3 questions on trigeminal neuralgia

108) 3 questions on racial pigmentation of the gums

109) Pontic should? Gently rest on tissue

Page 224: Decembery RQS – NBDE II

Combined remembered Qs 2020 110) There was a picture of a resin base partial picture and the question was, whats the

difference between this partial and a cast metal with acrylic partial. I put they dont act the

same during vertical forces

111) You tell a pt he has to replace his amalgams for composites.. what principle? Veracity

112) Cohort study

113) Unbundling

114) High blood pressure.. Make sure you know when its ok to treat and when its not

115) Dentist ignores pt behavior? Extinction

116) Tell show do tecnique

117) Epiphyseal plates? Synchondrosis

118) Cleft lip how many weeks in utero? 6-9

119) Hyoid bone twice on Xrays

JUNE 15 1. Collimation- reduces xray beam size and reduces low energy radiation we’re both in the options

○ Filtration obstruct low energy wavelength

2. Why do you Bead a max denture? Options were Support, strengthen base, retention, form a finish line, improve tissue contact

Page 225: Decembery RQS – NBDE II

Combined remembered Qs 2020

So for rpd tissue contact and for max denture retention

3. What is in sealer that prevents resin polymerization? Eugenol

4. You have a Retruded tongue. How does this affect dentures? Mastication, speaking, not being able to

stabilize mandibular as well

5. Root amputatation how to do it exactly

6. Root amputation - max 1 m, max 2 m, mand molars which ?

If the tooth has

Tongue blade - know anterior versus posterior and if it’s for one tooth versus bilateral ans: one tooth ant cross

bite

7. Emphysema: is it proximal or distal airspace and is it constriction or dilation

8. Histamine - know it’s role bronchiole constriction, precursor is histidine, high amount in skin? It’s all

except question and they all seemed correct so just look into what histamine does

Page 226: Decembery RQS – NBDE II

Combined remembered Qs 2020

9. Intrapulpal injection for the pulp? Subperiosteal, Supraperiosteal, intraosseous

10. Tramadol was answer to opioid abuser question

11. What is unique about opioids?

a. Diarrhea

b. I put can inc CNS depression when mixed with other drugs

12. Trigeminal neuralgia?

a. Flushing,

b. certain trigger points

Page 227: Decembery RQS – NBDE II

Combined remembered Qs 2020

13. pen and tetra?

14. High amount in GCF? Doxy or mino wasn’t in options, I put tetra

15. Most soluble anesthetic?

a. lido

b. articaine

c. prilocaine

16. Match the drug to what it does EXCEPT: zidovudine and varicella (Zido is for HIV)

17. Kaposi or syphilis associated w

both transmitted through sex . saliva and blood, Kaposi HHV8

18. What anesthetic for pregnant lady? I was between meperidine and lido

I think it’s lido

19. Hypercementosis? Pagets

20. Amellogensis imperfecta is in the histodifferntiation phase - but some sources say apposition so look it

up

21. Ectodermal dysplasia - oligodontia BUT options were tricky. They all sounded correct. I even think

congenitally absent teeth was an option.

22. Hypodontia affects what ? Maxilla, mandible, midfacial, alveolus

23. What affects hair thickness? Thyroid

24. What is not associated w cleft palate? All seemed correct: microdontia, ectopic erupted teeth,

congenitally missing teeth. Idk what right answer was look into it

25. Same thing that causes herpes?

a. Measles,

b. mumps,

c. chicken pox,

d. herpangina

Page 228: Decembery RQS – NBDE II

Combined remembered Qs 2020

26. If you have HIV you get ? Viral, bacterial, infection

27. Some weird about rubella (it had a second name like encephalo) and if it was chromosomal, genetic,

viral, congenital (? Idk if that was an option)

28. What’s an advantage of resin?

29. Humulin r question about how much meds and food to give

a. only fluid and half dose of insulin

30. Why not put a child lock on old person medication? Physical barrier ?

31. Affected and infected which is true? Very confusing answers but I put that there is bacteria present in

infected. Another option was that infected has no remineralization potential

32. Most likely to cause ear pain?

a. Max 1 molar

b. mand molar

c. max pre molar

I also had the one where if IANB doesn’t work like the lip isn’t numb but everything else is ? I put mental

Scaling and root planing? To remove calculus and clean root or to remove cementum and expose collagen

Incidence

Lingual Groove that commonly has caries common in? Incisors, canine, PM, molar

Page 229: Decembery RQS – NBDE II

Combined remembered Qs 2020 Which of the following is not for shock? Hypertension?

IV Dextrose is patient is unconscious

You gave oxygen to patient having heart attack. They start to get better. What do you do? Add more, continue

treatment, send them to the doc

3rd molar disappears ? Caldwell or take a radiograph to visualize

Endo bacteria? Strict anaerobes

Arrested caries? Brown and soft or brown and shiny ?

PSA is branch of maxillary from Infratemporal

You have a first max molar coming Down (extruded) and a missing opposing molar you’re going to do a bridge

on. Why would you first fix the max molar before bridge? Level occlusal plane or properly distribute occlusal

forces

Why do you do a suture? I put to reduce granulation tissue

3 qs on hemisection and root amputation very specific

11.Ingredients of marijuana? Tetrahydrocannabinol

12.Indirect retainers prevent? Portions were towards the tissue ward and other way.

13.Pupal and periapical differences( they use different words and ask many questions on it)

14.Mandi nerve enter cranium? infratemporal fossa

15.Mandi nerve exit cranium?foramen ovale

16.Apex locator what not do? Perforation?

17.2 Implant distance to implant is 3 … to tooth is 1.5mm

18.Sterilization controls? FDA

19.Which does not Monitor sterilization unit?options are Chemical, electrical, biological, mechanical.

20.Bisphosphonates acts on which cell?options osteclast, osteoblasts.

21.Trisomy 21 infrequently seen? Options Rampant caries, others all symptoms of trisomy 21.

22.distal border extension of denture(didn’t mention which one upper or lower)? Options mylohyoid line,

Hamilcar botch, mental foramen, incisive foramen.

23.benzodiazapene reversal?

24.hydrocodone reversal?

25.pregnant pt have acute abscess event to treat? Options 1,2,3trimeter, immediately when she have.

26.cantilever bridge what not true.options difficult to clean and others I don’t remember.

27.recent study show relation of periodontitis with what? Options cardiovascular, hypothyroidism, hepatitis c,

cancer.

28.differential wbcs count diagnosis of? Eosinophilia, anemia, spherocytosis, thrombocytopenic purpura.

Page 230: Decembery RQS – NBDE II

Combined remembered Qs 2020 29.which two factor or variable helps to diagnosis?options clinical diagnosis and history, radiographs, biopsy,

lab results.

30.Lot of prostho questions related to occlusion. No questions on materials was asked in my exam.

That’s all I remember for day1. But Day 1 was little hard and had lot many tricky questions.

Day2 was simple and good. Cases were doable. Make sure to check time.

All the best to all and hope we all pass😊